Você está na página 1de 304

Qual a relação do septo

nasal com os sistemas


carotídeos externo e
interno?

1.1 ANATOMIA DO NARIZ


1.1.1 Anatomia externa
1.1.1.1 Pirâmide nasal

É a estrutura externa do nariz, tendo na face inferior 2 aberturas – as


narinas.
A pirâmide é dividida em 2 porções:
1. Óssea: composta pelos ossos próprios do nariz e processos nasais
da maxila e do osso frontal;
2. Cartilaginosa: 2 terços inferiores são cartilaginosos, com 2
cartilagens laterais superiores e 2 laterais – inferiores ou alares.

O vestíbulo nasal é a região de entrada do nariz. É revestido


internamente por pele e pelos com função protetora, as vibrissas.
1.1.1.2 Válvula nasal
Localizada logo após o vestíbulo nasal, é uma projeção intranasal da
união das cartilagens laterais inferior e superior. Quando estreitada,
essa região pode ocasionar obstrução nasal.
Figura 1.1 - Anatomia da pirâmide óssea
Legenda: (A) osso nasal; (B) osso frontal; (C) processo frontal da maxila; (D) cartilagem
lateral superior; (E) cartilagem lateral inferior; (F) cartilagens alares menores; (G) região da
válvula nasal.
Fonte: Ilustração Claudio Van Erven Ripinskas.

Quadro 1.1 - Estruturas nasais externas

1.1.2 Fossas nasais


1.1.2.1 Parede medial – septo nasal

O septo nasal é uma estrutura laminar que separa as 2 fossas nasais.


Apresenta uma porção cartilaginosa, anterior, e uma porção óssea,
posterior, formada pelo vômer, pela crista maxilar e pela lâmina
perpendicular do osso etmoide.
A drenagem venosa do septo ocorre em direção à face intracraniana,
favorecendo a disseminação facial e meníngea de focos infecciosos
dessa estrutura (Figura 1.2). A região anteroinferior do septo é
chamada de zona de Kiesselbach, apresenta grande vascularização e
é o principal foco de epistaxe em adultos e crianças. Seu terço
anteroinferior é importante pela presença de um plexo
arteriovenoso, o que torna essa região a mais propensa a
sangramentos, principalmente pós-traumáticos e em rinites. Essa é
também a região de origem de veias que drenam em direção
intracraniana (Figura 1.2).
Comumente, o septo nasal está desviado da linha média. Isso decorre
de desvios isolados da cartilagem ou nas regiões de junção
osteocartilaginosa. O tipo e o grau do desvio são variáveis, podendo
chegar a obstruir o fluxo aéreo nasal.
#IMPORTANTE
A disseminação hematogênica para a meninge
é uma via notavelmente importante (Figura 1.2).

Figura 1.2 - Via de disseminação bacteriana intracraniana a partir da pirâmide nasal


Fonte: elaborado pelos autores.

a) Estruturas anatômicas do septo nasal

1. Cartilagem septal: anterior;


2. Vômer: posterior;
3. Crista maxilar: inferior;
4. Lâmina perpendicular do etmoide: superior.

Quadro 1.2 - Estruturas importantes na vascularização do septo


1.1.2.2 Parede lateral do nariz

a) Conchas – ou cornetos

São projeções osteomucosas que contribuem para as funções de


aquecimento, umidificação e filtragem do nariz.
A concha inferior ocupa o terço inferior das fossas nasais. Quando
aumentada de volume, é uma das grandes causadoras de obstrução
nasal. A concha média tem anatomia mais complexa e se estende de
forma vertical e oblíqua.
A concha superior localiza-se anteriormente ao esfenoide e tem
pouca relação com o fluxo aéreo nasal.
b) Meatos

Entre as conchas, encontram-se os meatos, denominados de acordo


com sua localização:
1. Superior, entre as conchas superior e média: região de
drenagem das células etmoidais posteriores;
2. Médio, entre as conchas média e inferior: região de drenagem
dos óstios do seio maxilar, das células etmoidais anteriores e do seio
frontal. É a principal região acometida pelas patologias nasais. Esse
meato tem especial importância, pois as patologias que acometem
essa região geralmente cursam com obstrução dos óstios dos seios e
geram sinusites;
3. Inferior, abaixo da concha inferior: localiza-se inferiormente à
concha inferior. Nesse espaço, encontramos o óstio nasal do ducto
nasolacrimal;
4. Recesso esfenoetmoidal: localizado posteroinferiormente ao
meato superior. Local de drenagem dos seios esfenoidais.

Figura 1.3 - Visão sagital da parede lateral do nariz

Nota: observar as conchas inferior, média e superior. Estão representados estiletes


introduzidos nos óstios dos seios e do ducto lacrimonasal.

Fonte: Ilustração Claudio Van Erven Ripinskas.

Quadro 1.3 - Meatos e estruturas que drenam


1.1.2.3 Limites do nariz

1. Anteriormente: narina;
2. Superiormente: o teto nasal é formado por uma fina placa óssea,
denominada lâmina crivosa, ou cribriforme. Nessa região, emergem as
terminações nervosas do nervo olfatório – I par craniano. Essa fina
placa óssea é vulnerável a fraturas em casos de trauma
cranioencefálico, sendo, por isso, uma das principais regiões de
fístulas liquóricas pós-traumáticas; geralmente está associada a
hiposmia por lesão das fibras nervosas;
3. Lateralmente: separando a fossa nasal da órbita, encontra-se a
lâmina papirácea. Por ser muito delgada, é a principal via de
disseminação de infecções dos seios da face para as órbitas,
principalmente em crianças;
4. Posteriormente: cóana, onde ocorre a transição com a faringe.
Pode ser sítio de malformações congênitas – imperfurações coanais.

Quadro 1.4 - Limites do nariz


1.1.3 Seios paranasais
1. Seios frontais: localizados na região frontal e supraorbitária.
Drenam para o meato médio;
2. Seios maxilares: estão abaixo da órbita e acima da cavidade bucal
(palato duro). Drenam para o meato médio; a região afunilada onde se
encontra seu óstio é chamada infundíbulo (Figura 1.4);
3. Células etmoidais: situadas medialmente à órbita, em contato
íntimo com a lâmina papirácea, existem em número variável e
apresentam grande variação anatômica. A partir das células etmoidais,
a sinusite se dissemina para a órbita. A concha média divide essas
células etmoidais em anteriores e posteriores;
4. Seio esfenoidal: localizado na região mais posterossuperior da
fossa nasal, tem contato íntimo com a base do crânio. Em seu interior,
há lateralmente a projeção dos canais ósseos da artéria carótida
interna (inferior) e do nervo óptico (superior – Figura 1.5).

É importante observar que o assoalho maxilar é


a região de inserção dos dentes superiores.
Infecções dentárias podem supurar e drenar
para o seio maxilar, causando sinusite
odontogênica.

Figura 1.4 - Relações topográficas do seio maxilar


Nota: observe a relação do assoalho maxilar com a dentição. Observar a comunicação do
seio maxilar com o meato médio. A região de comunicação também é denominada
infundíbulo maxilar.

Legenda: (A) concha inferior; (B) seio maxilar; (C) órbita; (D) concha média.

Fonte: Ilustração Claudio Van Erven Ripinskas.

Figura 1.5 - Meato médio, visualizado após remoção das conchas média e inferior

Legenda: (A) ducto frontal drenando para o meato médio; (B) bula etmoidal (célula
etmoidal anterior); (C) seio esfenoidal; (D) projeção do ducto lacrimonasal; (E) hiato
semilunar, região de drenagem dos seios maxilar, frontal e etmoidal anterior.

1.1.4 Cóana
Trata-se da abertura posterior da fossa nasal, delimitada pelo septo
nasal medialmente, assoalho nasal inferiormente e toro tubário
lateralmente.
1.1.5 Vascularização nasal
A vascularização nasal provém de ramos das carótidas externa e
interna.
1. Artéria carótida externa: fornece a maior parte da irrigação nasal
por meio de 2 ramos:
a) Artéria esfenopalatina: penetra as fossas nasais
posteriormente à concha inferior e vasculariza boa parte das
fossas nasais;
b) Artérias angular e labial superior: seus ramos terminais
vascularizam principalmente o vestíbulo e a pirâmide nasal.
2. Artéria carótida interna: fornece irrigação complementar das
fossas nasais por meio de 2 ramos – artérias etmoidais anteriores e
posteriores.

A anastomose dos 2 sistemas carotídeos


acontece na zona de Kiesselbach.

Figura 1.6 - Origem das artérias nasais e convergência para zona de Kiesselbach

Fonte: elaborado pelos autores.


1.1.6 Inervação
A inervação sensitiva da parede lateral é feita por meio do primeiro e
segundo ramos do nervo trigêmeo. O primeiro ramo – oftálmico –
dá origem ao nervo ciliar. O ramo maxilar dá origem ao nervo
nasopalatino. Esses nervos têm terminações livres espalhadas por
toda a mucosa nasal e são estimulados por substâncias irritantes,
como pimenta, pó e amônia; podem iniciar um reflexo protetor com
espirros, lágrimas ou secreções.
A inervação olfatória é dada pelo I par craniano – nervo olfatório –,
cujas fibras se localizam principalmente no terço superior da
mucosa nasal – parede lateral e septo. Essas fibras direcionam-se
superiormente e cruzam a base do crânio na lâmina cribriforme até
atingir o bulbo olfatório.
A inervação olfatória é dada pelo I par craniano
– nervo olfatório –, cujas fibras se localizam
principalmente no terço superior da mucosa
nasal – parede lateral e septo.
Quadro 1.5 - Inervação do nariz

Figura 1.7 - Principais nervos das fossas nasais


Nota: observar que a inervação da região anterossuperior é dada pelo ramo oftálmico NC
VI e a da região inferoposterior, pelo ramo maxilar NC VII.

Fonte: site StudyBlue.

Figura 1.8 - Bulbo olfatório e distribuição de suas fibras pela cavidade nasal
Fonte: site Human Anatomy Body.

1.1.7 Rinofaringe
Localizada posteriormente às cóanas e anteriormente à coluna
cervical, já não faz mais parte da cavidade nasal. Três estruturas
destacam-se nessa região: a adenoide, ou tonsila faríngea, o óstio
tubário e o recesso retrotubário, ou fosseta de Rosenmüller (Figura
1.9).
1.1.7.1 Estruturas da rinofaringe

1. Adenoide: órgão linfoide secundário localizado na região superior


da rinofaringe. Histologicamente é semelhante às tonsilas e tem
especial importância na infância, pois, uma vez hipertrofiada, pode
gerar obstrução total ou parcial do fluxo aéreo nasal, com consequente
respiração bucal e suas complicações;
2. Óstio tubário: a tuba auditiva tem sua abertura nasal na região da
rinofaringe, posteriormente à cauda da concha inferior. Tem relação
muito próxima com a adenoide;
3. Fosseta de Rosenmüller: localiza-se posteriormente ao óstio
tubário, é o principal sítio de origem dos tumores de rinofaringe.

Figura 1.9 - Rinofaringe e parede lateral do nariz


Legenda: (1) óstio faríngeo da tuba auditiva; (2) adenoide – atrófica; (3) toro tubário –
porção superior; (4) fosseta de Rosenmüller; (5) toro tubário – porção posterior.

Fonte: OtoRhinoLaryngology Portal.

1.1.8 Fisiologia das fossas nasais


1.1.8.1 Batimento mucociliar

A mucosa do nariz e dos seios é composta por 80% de células


pseudoestratificadas ciliadas (Figura 1.10 – A e B) e cerca de 10% por
goblet cells, responsáveis pela produção e secreção de muco. Tal
muco é composto por 2 camadas. A mais externa, de maior
viscosidade, é chamada de fase gel; e a mais interna, de menor
viscosidade, que fica em contato íntimo com as células, corresponde
à fase sol.
Os cílios têm importante função de clareamento. Agem realizando
uma batida rápida para frente – na fase gel – e um retorno lento –
batida de recuperação – na fase sol, movendo em fluxo unidirecional
partículas sólidas ali presentes.
Patologias que acometem o batimento mucociliar, como fibrose
cística e discinesia mucociliar primária, são fatores importantes na
gênese de sinusites crônicas, pois comprometem o clareamento e
facilitam a infecção local.
Figura 1.10 - Ciclo de batimento do cílio
Legenda: (A) célula ciliar; (B) microscopia eletrônica de células ciliadas de seios da face.

Fonte: (A) Ilustração Claudio Van Erven Ripinskas; (B) Acquired cilia dysfunction in chronic
rhinosinusitis, 2016.

1.1.8.2 Fluxo aéreo nasal

O ar inspirado segue uma trajetória turbilhonar, tendo como pontos


mais baixos a narina e a cóana e o meato médio como ápice. Dessa
forma, a perviedade do terço inferior das fossas nasais é crucial para
um bom fluxo aéreo nasal. A concha inferior hipertrófica, rinite, é o
principal sítio de obstrução nasal em adultos.
O fluxo turbulento é importante para filtrar, aquecer e umidificar o
ar inspirado. O fluxo aéreo adequado em ambas as cavidades
permitirá ao indivíduo adequado desenvolvimento craniofacial
durante a infância e a juventude.
1.2 ANATOMIA DA ORELHA
1.2.1 Orelha externa
Compreende o pavilhão auricular e o Conduto Auditivo Externo (CAE
– Figura 1.11).
Quadro 1.6 - Estruturas da orelha externa
A orelha externa é recoberta por pele e anexos; os pelos estão
presentes apenas no terço externo. A pele contém glândulas
ceruminosas.
Figura 1.11 - Orelha externa
Legenda: (A) pavilhão auricular; (B) conduto auditivo externo; (C) tímpano.

Fonte: adaptada de sciencepics.

1.2.2 Orelha média


Composta por: caixa ou cavidade timpânica, tuba auditiva e células
mastoideas.
1.2.2.1 Tuba auditiva

A tuba auditiva é um canal osteocartilaginoso envolto por músculos


e que conecta o ouvido médio com a nasofaringe. A porção óssea,
posterolateral, se abre na cavidade timpânica, e a
musculocartilaginosa, anteromedial, na nasofaringe – óstios
tubários – e tem íntima relação com a adenoide.
Fisiologicamente, a tuba permanece fechada. Sua abertura ocorre de
forma ativa utilizando-se de 3 músculos: tensor do véu palatino, o
mais importante, elevador do véu palatino e salpingofaríngeo. Esses
músculos estão intimamente ligados à deglutição e à movimentação
palatal. O fechamento da tuba se faz de maneira passiva.
Uma das funções da tuba auditiva é equalizar a pressão do ar dentro
da orelha média com a pressão do ar ambiente – fossas nasais. Por
isso, a deglutição gera a abertura transitória da tuba auditiva e
consequente equalização da pressão.
Disfunções no funcionamento da tuba auditiva refletem-se na orelha
média, com consequentes otites e perdas auditivas condutivas.
Figura 1.12 - Diferenças anatômicas da tuba auditiva
Legenda: (A) crianças menores e (B) crianças maiores e adultos.

Fonte: site do Curso ENARM.

Figura 1.13 - Equalização da pressão na orelha média e na fossa nasal após a abertura da
tuba auditiva
Nota: à esquerda está representada a orelha média e à direita, a fossa nasal. Observar
que, na situação inicial, a pressão na cavidade nasal era maior do que a pressão na orelha
média.

Fonte: elaborado pelos autores.

1.2.2.2 Células da mastoide

A mastoide é a região posterolateral do osso temporal; é formada por


diversas pequenas cavidades – células – de tamanho variável que se
comunicam entre si. Dessa forma, apresenta-se como um osso
aerado. A maior célula, e a primeira a surgir na criança, é o antro da
mastoide. A partir dela se desenvolvem as demais células.
1.2.2.3 Caixa timpânica

A caixa timpânica é uma pequena cavidade cuboide localizada entre a


tuba auditiva, anterior, e as células da mastoide, posterior. Tem 6
paredes e, no seu interior, ossículos, músculos e ligamentos.
a) Limites da caixa timpânica

1. Superior – tegmen tympani: assoalho da fossa cerebral média;


2. Inferior: relação com a veia jugular;
3. Lateral: membrana timpânica;
4. Medial: diversas saliências, como o promontório, que corresponde
ao giro basal da cóclea, a abertura da janela oval ou do vestíbulo, onde
se aloja o estribo, e a janela redonda ou da cóclea;
5. Anterior: relação com a carótida interna e a tuba auditiva;
6. Posterior: antro mastoide.

1.2.2.4 Membrana timpânica

A Membrana Timpânica (MT) localiza-se no extremo medial do CAE.


Suas funções básicas são separar a orelha média da externa e
transmitir as vibrações sonoras para a cadeia ossicular. Possui 3
folhetos celulares fundidos entre si:
1. Externo: formado pela pele contínua com a pele do CAE;
2. Intermediário: formado por tecido fibroso;
3. Interno: formado pela mucosa da orelha média.

Divide-se em porção tensa, inferior, e porção flácida, superior. Essa


divisão é feita pelos ligamentos maleolares anterior e posterior.
Ligado à MT está o martelo, que faz na membrana uma umbilicação
na região superior. Dessa umbilicação partem os 2 ligamentos em
direção à periferia da membrana.
Os colesteatomas, compostos de epitélio escamoso estratificado e
preenchidos por acúmulo de queratina esfoliada localizada dentro da
orelha média, originam-se na pars flácida da membrana timpânica
devido à ausência da lâmina intermediária.
a) Representação esquemática das porções timpânicas

1. Porção flácida – superior:


a) Folheto interno: mucoso;
b) Folheto externo: pele.
2. Porção tensa – inferior:
a) Folheto interno: mucoso;
b) Folheto médio: fibroso;
c) Folheto externo: pele.
3. Ligamentos maleolares.

1.2.2.5 Cadeia ossicular

A Cadeia Ossicular (CO) é composta de 3 ossículos:


1. Martelo: composto de cabeça, colo e manúbrio, está em contato
com a membrana timpânica;
2. Bigorna: composta de corpo, apófise curta e longa, que se articula
com o estribo;
3. Estribo: composto por 2 cruras, ou “pernas”, e a platina, ou base.
Faz articulação com a janela oval por meio da platina, transmitindo os
impulsos vibratórios da CO para os líquidos do ouvido interno.

Os ossículos do ouvido médio estão suspensos por seus músculos e


ligamentos.
São músculos do ouvido:
1. Músculo tensor do tímpano: liga-se ao martelo;
2. Músculo estapédio: liga-se ao estribo.

Ambos, ao se contraírem, provocam rigidez da CO, protegendo o


ouvido interno da transmissão de sons potencialmente lesivos. A
vascularização do ouvido provém de ramos das artérias carótidas
externa, interna e vertebral.
Figura 1.14 - Orelha externa, média e interna
Nota: observar as inter-relações entre as estruturas descritas.

Fonte: adaptado de wiseGEEK.

1.2.3 Orelha interna


Está localizada na profundidade do osso temporal e é formada por
um arcabouço ósseo, o labirinto ósseo, dentro do qual se encontra o
labirinto membranoso (Figura 1.15).
Figura 1.15 - Orelha interna
Legenda: (A) labirinto ósseo: observe a cóclea, o vestíbulo e os canais semicirculares; (B)
labirinto membranoso, que preenche o labirinto ósseo.

Fonte: Ilusmedical.

1.2.3.1 Labirinto ósseo

O labirinto ósseo (Figura 1.16) dá suporte ósseo e formato ao


labirinto membranoso e se divide anatomicamente em canais
semicirculares (CSCs), vestíbulo e cóclea.
Figura 1.16 - Labirinto ósseo

Fonte: adaptado de Ilusmedical.

1.2.3.2 Labirinto membranoso

O labirinto membranoso é preenchido por endolinfa – composição


semelhante ao intracelular: maior concentração de potássio e menor
de sódio – e encontra-se imerso na perilinfa, de composição iônica
semelhante à plasmática – rica em sódio e pobre em potássio.
1.2.3.3 Estruturas do labirinto

a) Canais semicirculares

São canais dispostos nos 3 planos dimensionais, com angulação de


cerca de 90° entre eles. São denominados lateral, superior e inferior
– ou posterior. Em uma das extremidades, cada canal apresenta uma
pequena dilatação denominada ampola, que é a região inervada pelo
nervo vestibulococlear NC VIII. Esses canais têm a função de detectar
os movimentos rotacionais da cabeça.
b) Vestíbulo: utrículo e sáculo

São dilatações na região intermediária entre os CSCs,


posteriormente, e a cóclea, anteriormente, comunicando-se com
essas estruturas. São responsáveis pela detecção de movimentos
lineares da cabeça, como andar para frente ou subir de elevador.
c) Canais e tipos de movimentos detectados

1. CSCs: movimentos rotacionais;


2. Sáculo e utrículo: movimentos lineares.

d) Cóclea

A cóclea é um órgão em formato de espiral ao redor de um cone


ósseo central, chamado modíolo. Deste partem lâminas ósseas que
subdividem a cóclea em subcanais, que são a rampa vestibular,
rampa média e rampa timpânica.
As rampas vestibular e timpânica são preenchidas por perilinfa e
comunicam-se no ápice da cóclea. A rampa média é preenchida por
endolinfa, e, em seu interior, está o órgão de Corti, responsável pela
detecção dos estímulos auditivos.
e) Órgão de Corti
O órgão de Corti situa-se ao longo de toda a rampa média, o labirinto
membranoso. Em sua base está a membrana basilar, sensível às
vibrações das rampas vestibular e timpânica. Sobre a membrana
basilar, há as células ciliadas internas e externas, que recebem fibras
neurais e são capazes de transformar os estímulos mecânicos
provenientes da vibração da membrana basilar em estímulos
neurais, que atingem o Sistema Nervoso Central (SNC) por meio do
NC VIII.
As perdas auditivas tipo presbiacusia e induzida por ruído iniciam-
se em frequências agudas. Acredita-se que isso ocorra devido ao uso
excessivo do trecho inicial da membrana basilar, pois todos os sons
devem passar por essa região para chegar à sua região de estímulo.
1.2.4 Inervação
1.2.4.1 Nervo vestibulococlear

O ouvido interno é inervado pelo NC VIII, que emerge da ponte, entra


por um canal ósseo, o Conduto Auditivo Interno (CAI) e chega ao
labirinto. Divide-se, então, em ramo anterior, coclear, responsável
pela inervação da cóclea, e ramo posterior (vestibular), responsável
pela inervação do vestíbulo e dos CSCs.
a) Divisões do VIII nervo craniano: nervo vestibulococlear – VIII par
craniano

1. Ramo anterior: inervação da cóclea;


2. Ramo posterior: inervação dos CSCs e vestíbulo.

Figura 1.17 - Vias percorridas pela estimulação sonora até atingir o sistema nervoso
central
Fonte: elaborado pelos autores.

1.2.4.2 Nervo facial

O nervo facial NC VII é um nervo de especial importância na


anatomia do ouvido. Ele emerge do SNC no ângulo pontocerebelar e
entra pelo CAI junto com o NC VIII. Ao emergir do CAI, tem relação
anatômica com a orelha média, o CSC lateral, a bigorna e a parede
posterior do CAE. Seus principais ramos na orelha média são:
1. Nervo estapédio: responsável por ativar o músculo estapédio
(reflexo do estapédio) e proteger a orelha interna de trauma acústico;
2. Nervo corda do tímpano: responsável por parte da inervação
gustativa da língua. A paralisia facial pode cursar com sensação de
gosto metálico na boca por acometimento desse ramo.
Na orelha média, o NC VII corre através de um canal ósseo cujas
paredes são estreitas. Dessa forma, a ocorrência de otites médias
pode estender o processo inflamatório através de tais paredes e
acometer o nervo facial, gerando edema neurite e consequente
paralisia periférica. Outras patologias que acometem o ouvido, como
tumores ou colesteatomas, também podem cursar com paresia ou
paralisia facial do tipo periférica.
O nervo facial emerge do osso temporal por meio do forame
estilomastóideo e se subdivide em 5 ramos principais, responsáveis
pela inervação motora da face. Dessa forma, a paralisia facial
periférica cursa com paralisia de toda a hemiface.
Figura 1.18 - Nervo facial e seus ramos

Nota: observar os ramos estapédio e corda do tímpano, que emergem intratemporalmente;


o NC VII emerge do osso temporal pelo forame estilomastoideo e se divide em seus 5
ramos motores.

Fonte: Clinical Gate.

Figura 1.19 - Ramos extratemporais do nervo facial


Nota: observar sua distribuição e inervação de toda a hemiface.

Fonte: Clinical Gate.

#IMPORTANTE
O nervo facial deve ser identificado durante
cirurgias de ouvido, de forma a evitar lesão e
paralisia facial periférica.

1.3 ANATOMIA DA FARINGE, DA


LARINGE E DA CAVIDADE ORAL
1.3.1 Faringe
A faringe divide-se, anatomicamente, em nasofaringe, orofaringe e
hipofaringe (Figura 1.20).
Figura 1.20 - Faringe e suas divisões

Fonte: adaptado de Teguh Mujiono.

1. Nasofaringe: localiza-se superiormente, tendo o esfenoide como


limite superior, a coluna cervical posteriormente, a cóana anteriormente
e os óstios tubários lateralmente. Suas principais estruturas são a tuba
auditiva, a adenoide, que, quando hiperplásica, pode cursar com
obstrução nasal e as fossetas de Rosenmüller, região retrotubária,
principal região de origem de carcinomas de rinofaringe. A adenoide de
crianças pode estar aumentada de tamanho, de forma a obstruir até
100% do espaço faríngeo. Pode ocluir a abertura da tuba auditiva e
gerar patologias da orelha média: otites secretoras, otites crônicas ou
secretoras;

Figura 1.21 - Cavidade nasal e rinofaringe


Nota: observar a localização anatômica da adenoide, tuba auditiva e fosseta de
Rosenmüller.

Fonte: site Human Anatomy Body.

2. Orofaringe: anteriormente é limitada pelo palato mole, com seus 2


pilares – o anterior, músculo palatoglosso, e o posterior, músculo
palatofaríngeo, que representam a transição entre a cavidade oral e a
orofaringe. Entre esses pilares está a amígdala palatina. O espaço
virtual entre os pilares amigdalianos e a amígdala é chamado cápsula
amigdaliana. Nesse espaço formam-se os abscessos periamigdalianos
(Figura 1.22) devido aos ligamentos da amígdala serem mais frouxos
nessa região. A parede posterior da orofaringe está em contato íntimo
com a coluna cervical. Essa região é formada pelos músculos
constritores da orofaringe, de especial importância na fonação e na
deglutição (Figura 1.24);

Figura 1.22 - Orofaringe


Legenda: (A) pilar amigdaliano posterior; (B) pilar anterior; (C) úvula; (D) amígdala; (E)
língua.

Fonte: Ilustração Claudio Van Erven Ripinskas.

Figura 1.23 - Anatomia da amígdala


Nota: observar os pilares anterior e posterior “envolvendo” a amígdala. A linha tracejada
mostra esquematicamente a região ocupada pela amígdala entre os pilares.

Fonte: adaptado de Klem, 2007.

Figura 1.24 - Músculos constritores superior, médio e inferior da faringe


Fonte: Ilustração Claudio Van Erven Ripinskas.

3. Hipofaringe: nessa região localiza-se a transição da faringe com o


esôfago, posteriormente, e com a laringe, anteriormente. Sua
visualização é feita apenas de forma indireta, com espelho de Garcia
ou laringoscópio.

O anel linfático de Waldeyer é composto por estruturas linfoides


capazes de reconhecer antígenos inalados ou ingeridos, gerando
resposta imunológica ao agressor. Esse anel é composto por:
1. Adenoide ou tonsilas faríngeas: localizada na rinofaringe;
2. Amígdalas ou tonsilas palatinas: localizadas lateralmente na
orofaringe;
3. Amígdalas ou tonsilas linguais: localizadas posteriormente na
língua;
4. Tonsilas tubárias: pequenas estruturas localizadas no toro tubário.

Essas estruturas desempenham papel de “barreira” imunológica,


porém podem se encontrar aumentadas –hiperplásicas, por razões
ainda não totalmente elucidadas (Figura 1.25).
Figura 1.25 - Estruturas do anel linfático de Waldeyer

Fonte: adaptado de WebMD.

1.3.2 Laringe
A laringe localiza-se na linha média cervical, anteriormente ao
esôfago, superiormente à traqueia e inferiormente à hipofaringe.
Suas funções são proteção da via aérea, respiração e fonação.
1.3.2.1 Cartilagens da laringe

Ao todo, são 9 cartilagens:


1. Pares: aritenoide, cuneiformes e corniculadas. As aritenoides estão
acima da cricoide, e nelas se inserem as pregas vocais;
2. Ímpares: epiglote, tireoide e cricoide.

1.3.2.2 Musculatura intrínseca

A musculatura intrínseca da laringe é a principal responsável pela


fonação. Os músculos são:
1. Adutores das pregas vocais: tireoaritenoides – que compõem as
pregas vocais –, interaritenoide, cricoaritenoide lateral e cricotireoide;
2. Abdutor: cricoaritenoide posterior.

1.3.2.3 Musculatura extrínseca

Os músculos extrínsecos da laringe têm origem e inserção em


estruturas externas à luz laríngea e permitem a movimentação da
laringe durante a deglutição, fonação e respiração. Os músculos são:
1. Infra-hióideos: omo-hióideo, esterno-hióideo, estilofaríngeo e
esternotireóideo, que movimentam inferiormente o hioide e a laringe;
2. Supra-hióideos: estilo-hióideo, digástrico, milo-hióideo e gênio-
hioide, que movimentam superiormente o hioide e a laringe.

1.3.2.4 Inervação da laringe

1. Motora: o nervo laríngeo inferior, recorrente, inerva todos os


músculos, exceto o cricotireoide, inervado pelo laríngeo superior;
2. Sensitiva: nervo laríngeo superior. Todos esses nervos são ramos
do nervo vago – NC X.

O músculo cricotireoide é o único inervado pelo laríngeo superior;


todos os demais são inervados pelo laríngeo recorrente.
Considerando que a laringe é inervada pelo nervo vago, inervações
sensitiva e motora, podemos dizer que eventuais lesões desse nervo
podem cursar com alteração na motricidade laríngea ou na
sensibilidade. Tumores comprimindo o nervo ou aneurismas de
aorta podem cursar com esse tipo de lesão no nervo. Cirurgias de
tireoide podem cursar com lesão do nervo laríngeo recorrente e
consequente paralisia (uni ou bilateral) laríngea.
1.3.2.5 Fonação

Ocorre pela vibração das pregas vocais quando o ar atravessa a


laringe durante a expiração. Essa vibração das pregas vocais e da
mucosa gera uma onda sonora de acordo com a tensão, a massa e a
posição das pregas.
Figura 1.26 - Fonação

Legenda: (A) abdução das pregas vocais no movimento inspiratório; (B) adução das
pregas durante a fonação (expiração) – (1) glote; (2) pregas vocais; (3) epiglote; (4)
comissura anterior; (5) aritenoides; (6) comissura posterior.

Fonte: Ilustração Claudio Van Erven Ripinskas.

1.3.2.6 Glândulas tireoide e paratireoides


São responsáveis pela produção de tiroxina (T4), triiodotironina
(T3) e por calcitonina. Estão localizadas na região anterior do
pescoço, posteriormente aos músculos extrínsecos da laringe.
Possuem 2 lobos laterais unidos na linha média por um istmo e têm
estreita relação com a cartilagem cricoide, esôfago e os primeiros
anéis traqueais. A relação clínica mais importante é com o nervo
laríngeo recorrente, que percorre o espaço entre a tireoide e o
esôfago e traqueia posteriormente. Em tireoidectomias, pode ser
lesado e causar paralisia laríngea.
A irrigação arterial é feita pelas artérias tireóideas superiores
(primeiros ramos da artéria carótida externa) e inferiores (ramos
dos troncos tirocervicais).
Figura 1.27 - Glândula tireoide
Nota: observe a relação anatômica com a laringe, traqueia e esôfago posteriormente.

Fonte: Nerthuz.

1.3.3 Cavidade oral e glândulas salivares


A cavidade oral é delimitada anteriormente pela rima bucal –
músculo orbicular da boca –, lateralmente pela região jugal,
superiormente pelo palato duro e mole e inferiormente pelo
assoalho da boca.
1. Palato: formado anteriormente pelo osso maxilar, palato duro, e
posteriormente pelo músculo palatofaríngeo, palato mole. Apresenta-
se lateral e anteriormente à arcada dentária. Essa estrutura tem íntima
relação com o assoalho do seio maxilar. Essa relação favorece a
disseminação de bactérias para os seios maxilares durante processos
infecciosos ou após cirurgias dentárias, as chamadas sinusites
odontogênicas;
2. Assoalho da boca: formado pelos músculos gênio-hióideo, milo-
hióideo e digástrico. Apresenta anteriormente os óstios de drenagem
das glândulas salivares sublinguais e submandibulares – região de
carúncula lingual;
3. Língua: composta por musculatura intrínseca, fibras musculares
dispostas em diversas direções, e extrínseca, genioglosso,
estiloglosso, hioglosso, palatoglosso. Em sua superfície, apresenta as
carúnculas linguais e as papilas gustativas;
4. Glândulas salivares maiores: 3 pares principais, sendo eles:
a) Parótidas: localizadas lateralmente ao ramo da mandíbula e
anteriormente ao pavilhão auricular. Têm relação íntima com o NC
VII, que passa em seu interior e emerge em seu terço anterior. O
ducto de drenagem da glândula parótida localiza-se na mucosa
jugal, na região do segundo molar superior;
b) Submandibulares: localizam-se inferiormente à mandíbula.
Seu ducto de drenagem corre inferiormente à língua até a
carúncula lingual;
c) Sublinguais: localizadas inferiormente à língua, drenam em
conjunto com as submandibulares (Figura 1.28).

Figura 1.28 - Glândulas salivares maiores e suas vias de drenagem


Nota: observar a drenagem do ducto parotídeo na altura do segundo molar superior; o
ducto das submandibulares drena na carúncula lingual, enquanto os ductos dos sublinguais
drenam lateralmente à língua.

Fonte: adaptado de WebMD.

1.4 ANATOMIA DOS ESPAÇOS


CERVICAIS PROFUNDOS
A musculatura cervical, bem como as fáscias musculares e viscerais,
delimitam espaços virtuais que se comunicam entre si e servem
como via de disseminação e expansão dos abscessos cervicais
profundos. Tais espaços estendem-se desde a base do crânio –
espaço parafaríngeo – até a coluna lombar, anteriormente à coluna
vertebral; a extensão de um abscesso para a região mediastinal
apresenta mortalidade de cerca de 50%.
Anatomicamente, os espaços cervicais profundos são subdivididos
em supra-hióideos, infra-hioideo e pancervicais. São eles:
1. Pancervicais: retrofaríngeo; danger space; pré-vertebral; visceral
vascular;
2. Supra-hióideos: parafaríngeo; submandibular; parotídeo; temporal;
mastigatório; peritonsilar;
3. Infra-hióideo: visceral anterior.

1.5 ANATOMIA DOS NÍVEIS CERVICAIS


LINFONODAIS
A drenagem linfática da face e estruturas cervicais corre para
linfonodos distribuídos principalmente na região anterolateral do
pescoço. Muitos tumores de cabeça e pescoço têm como primeiro
sinal a presença de um linfonodo aumentado – metastático. A
metástase linfonodal também ocorre, em geral, seguindo um padrão.
Anatomicamente, os linfonodos estão distribuídos em áreas ou
níveis cervicais, delimitados por estruturas anatômicas, conforme
resumem o Quadro 1.7 e a Figura 1.29.
Quadro 1.7 - Limites anatômicos dos níveis cervicais
Figura 1.29 - Níveis cervicais
Fonte: Ilustração Claudio Van Erven Ripinskas.
Qual a relação do septo
nasal com os sistemas
carotídeos externo e
interno?
Conforme discutido na seção de irrigação nasal, vimos que
na região anterior do septo nasal, a zona de Kiesselbach, há
confluência do sistema carotídeo externo – artéria angular
e labial superior – e do interno – artérias etmoidais
anterior e posterior. Ou seja, o septo nasal é uma área de
anastomose entre o sistema carotídeo interno e o externo.
Em uma audiometria, por
que o limiar sonoro por via
aérea deve ser maior ou
igual ao limiar medido por
via óssea?

2.1 EXAMES DE IMAGEM


2.1.1 Radiografia de cavum
Trata-se de radiografia simples da nasofaringe em perfil. Estando o
paciente com a boca aberta e fechada, é possível a avaliação indireta
do tamanho da adenoide e do grau de obstrução da rinofaringe
causado por ela.
2.1.2 Radiografia de seios paranasais
Muito utilizada no passado para a avaliação de rinossinusites, tem
baixa sensibilidade e especificidade para a avaliação de quadros
agudos, já que não diferencia processos inflamatórios de infecciosos
(Figura 2.1).
Figura 2.1 - Radiografia de seios paranasais
Fonte: Inter-Observer and Intra-Observer Variability in the Assessment of The Paranasal
Sinuses Radiographs, 2005.

2.1.3 Radiografia de ossos nasais


Utilizada para avaliar os ossos nasais, principalmente na suspeita de
fratura, quando realizada em perfil permite a avaliação da
integridade e do alinhamento dos ossos nasais (Figura 2.2).
Em crianças, aparentes linhas de fratura nos
ossos nasais podem corresponder à região de
fusão óssea.

Figura 2.2 - Ossos nasais em perfil


Nota: observar a fratura e o desalinhamento.

Fonte: Fracture Nasal Bones, 2013.

2.1.4 Tomografia computadorizada de seios


paranasais
Na Tomografia Computadorizada (TC) de seios paranasais, as
imagens mais utilizadas são as de cortes axiais e coronais,
principalmente em janelas de partes ósseas. Em geral, espera-se que
as fossas nasais e os seios paranasais estejam preenchidos por ar.
Seu velamento indica a retenção de secreções ou a presença de lesão
ocupando a luz das estruturas. As principais estruturas visualizadas
e suas alterações são:
1. Septo nasal: presença de desvios;
2. Cornetos inferiores: hipertrofia ou atrofia;
3. Concha média: alterações anatômicas, concha bolhosa –
preenchida por ar;
4. Meato médio: infundíbulo – região de drenagem do seio maxilar –,
seios etmoidais anteriores, drenagem do seio frontal; o velamento
dessa região geralmente ocorre em sinusites, ou secundariamente a
polipose e papilomatose nasal;
5. Seio maxilar: velamento que indica processo patológico;
6. Seios etmoidais: em geral, acometidos com a região do meato
médio;
7. Seio frontal e esfenoide: segue os princípios dos demais seios;
8. Órbita: avaliação da lâmina papirácea e seu acometimento em
sinusites;
9. Base do crânio: avaliação da lâmina cribriforme ou crivosa.

As imagens obtidas com janelas para partes moles são úteis para a
visualização de tumores e patologias não ósseas da região. No
entanto, fornecem menos detalhes anatômicos. Os cortes sagitais
auxiliam na visualização do seio frontal e de seu óstio de drenagem.
Figura 2.3 - Tomografia computadorizada de seios paranasais normal
Legenda: (1) órbita/globo ocular; (2) seio maxilar; (3) septo nasal; (4) corneto inferior; (5)
infundíbulo maxilar; (6) célula etmoidal infraorbitária (Onodi); (7) concha média; (8) etmoide;
(9) lâmina cribriforme; (10) lâmina papirácea.

Fonte: site Corner Stone – Ear, Nose and Throat.

Figura 2.4 - Tomografia computadorizada de seios paranasais mostrando velamento


maxilar esquerdo por sinusite infecciosa
Fonte: Variações anatômicas e sinusopatias: estudo por tomografia computadorizada,
2002.

As principais patologias que alargam o infundíbulo maxilar


(comunicação do seio maxilar com meato médio) são o pólipo de
Killian, o papiloma invertido, a mucocele maxilar e a sinusite
fúngica.
Figura 2.5 - Pólipo de Killian, ou antrocoanal
Fonte: Adenoid cystic carcinoma of palate: A case report and review of Literature, 2013.

Figura 2.6 - Papiloma invertido


Fonte: Inverted papilloma: Evaluation with CT and MRI, 2007.

Figura 2.7 - Mucocele maxilar


Fonte: Endoscopic sinus surgery for maxillary sinus mucoceles, 2006.

Figura 2.8 - Sinusite fúngica


2.1.5 Tomografia de ouvidos
Usam-se predominantemente as imagens em janelas de partes
ósseas, uma vez que se avalia a região do osso temporal.
Normalmente, cavidade timpânica, o mastoide e o Conduto Auditivo
Externo (CAE) estão bem aerados. As principais estruturas avaliadas
são:
1. CAE: integridade óssea, tumorações, erosões;
2. Cadeia ossicular: integridade e continuidade; possivelmente
erodida;
3. Cavidade timpânica: velamento e erosões. Observar a região de
tegmen tympani – base do crânio;
4. Esporão de Chaussé: correspondente à projeção da parede
superior do CAE no interior da caixa timpânica; erosão sugestiva de
colesteatoma;
5. Mastoide: velamento e aeração; mastoides ebúrneas – poucas
células aeradas – que indicam processos crônicos.
#IMPORTANTE
Coalescência, destruição do trabeculado ósseo,
ocorre em otomastoidites de rápida evolução.

2.1.6 Tomografia de pescoço


É utilizada para diversos fins. Os cortes axiais em janelas de partes
moles com uso de contraste permitem melhor visualização dos vasos
sanguíneos e são os que normalmente fornecem maior número de
informações. Esse é o principal método utilizado para avaliar a
extensão de abscessos, como os periamigdalianos. As principais
estruturas são:
1. Palato: deve apresentar-se na linha média, simétrico e sem
abaulamentos;
2. Rinofaringe, orofaringe e hipofaringe: devem seguir os princípios
descritos. Atenção especial deve ser dada às fossetas de Rosenmüller;
3. Cavidade oral: avaliar língua e estruturas relacionadas;
4. Glândulas salivares: presença de tumorações, abaulamentos e
cálculos em seus ductos;
5. Laringe: espaço aéreo, integridade das cartilagens e presença de
tumorações;
6. Tireoide: posição, textura e tamanho. Verificar a presença de cistos
ou nódulos;
7. Vasos: os principais vasos são as artérias carótidas internas e
externas, além de veias jugulares internas, externas e anteriores;
8. Linfonodos: avaliar tamanho, número, forma e presença de
necrose no interior;
9. Espaços cervicais profundos: em vigência de abscessos
cervicais, avaliar a extensão e as estruturas relacionadas.

Figura 2.9 - Tomografia computadorizada de pescoço, corte axial, janelas de partes moles,
com uso de contraste intravenoso
Legenda: (1) corpo da mandíbula; (2) músculo gênio-hióideo; (3) músculo milo-hióideo; (4)
corno menor do osso hioide; (5) glândula submandibular; (6) corno maior do osso hioide;
(7) veia jugular externa; (8) artéria carótida externa; (9) artéria carótida interna; (10) veia
jugular interna; (11) corpo de C4; (12) lâmina de C4; (13) espinha de C4; (14) pilar articular;
(15) forame intervertebral; (16) músculo trapézio; (17) músculo esternocleidomastóideo;
(18) faringe; (19) epiglote; (20) valécula; (21) prega glossoepiglótica medial; (22) corpo do
osso hioide.

Fonte: site About Cancer.


2.1.7 Ressonância magnética de mastoides-
ouvidos
A Ressonância Nuclear Magnética (RNM) de mastoides-ouvidos é
usada para o diagnóstico e o estadiamento de tumores locais –
glomo, neurinomas –, a avaliação pré-implante coclear e a suspeita
de fístulas liquóricas otogênicas.
Figura 2.10 - Ressonância nuclear magnética de ouvido interno – sistema nervoso central

Nota: observar o conduto auditivo interno, setas vermelhas, e os canais semicirculares


laterais, setas amarelas.

2.1.8 Ressonância de nariz e seios da face


É usada, geralmente, para o diagnóstico do estadiamento de tumores
locais, como estesioneuroblastoma e carcinomas espinocelulares e a
avaliação de extensão intracraniana de sinusites.
2.1.9 Exame ultrassonográfico
Método de escolha para avaliação de estruturas cervicais
relativamente superficiais, sem necessidade de exposição a radiação
ou contraste. É utilizado principalmente para avaliação de:
1. Glândulas salivares:
a) Lesões tumorais bem ou mal definidas, com ou sem sinais de
invasão periférica;
b) Processos inflamatórios crônicos ou agudos: aumento da
vascularização e fluxo sanguíneo, alterações parenquimatosas;
c) Abscessos: coleção líquida com sinais de processo inflamatório
periférico;
d) Sialolitíase: presença de imagem hipoecogênica, geralmente
com sombra acústica.
2. Linfonodos: avaliação da quantidade e aspecto de linfonodos
cervicais e intraórgãos, principalmente glândulas salivares.
a) Linfonodos normais: hipoecoico, aspecto oval, medindo até 1
cm por 0,5 cm no maior e menor eixo, respectivamente. Possível
visualização de hilo linfonodal;
b) Linfonodo inflamatório ou reativo: geralmente há aumento do
fluxo através do hilo, aumento em tamanho, com preservação do
hilo, parênquima hipoecoico e formato ovoide. Em casos severos
pode haver sinais de supuração ou abscessos;
c) Linfonodo maligno: geralmente há sinais de neovascularização
em área capsular, vasos hilares aberrantes e possível sinal de
necrose intranodal.
3. Tireoide: tem uma aparência homogênea, a cápsula pode aparecer
como uma linha hiperecoica fina. Volume de 10 a 15 mL para mulheres
e 12 a 18 mL para homens. Alterações da ecogenicidade do órgão
podem sugerir processos inflamatórios crônicos ou agudos;
4. Nódulos: deve-se avaliar tamanho, posição, ecogenicidade,
calcificação, margens, halo e vascularização. Baseados nessas
características, podemos sugerir malignidade dos nódulos.
Em casos de lesões suspeitas à ultrassonografia, frequentemente se
realiza a punção aspirativa por agulha fina (PAAF) guiada pela
ultrassonografia. Esse teste permite avaliação da celularidade. Em
casos inconclusivos, pode-se realizar a punção com biópsia,
removendo-se fragmentos da lesão.
2.2 EXAMES ENDOSCÓPICOS
2.2.1 Nasofibrolaringoscopia
A nasofibroscopia fornece uma avaliação de resolução menor do que
a endoscopia rígida. Com um aparelho de fibra ótica flexível,
introduzido pelas narinas, podem-se examinar detalhadamente:
fossas nasais – cornetos, meatos, óstios de seios, septo nasal;
rinofaringe – óstios tubários, fossetas de Rosenmüller e palato
mole; e laringe – base da língua, epiglote, aritenoides, pregas vocais
e infraglote.
Figura 2.11 - Nasofibroscopia de nariz
Legenda: (A) concha nasal inferior; (B) concha nasal média; (C) septo nasal.

Fonte: adaptado Ressecção endoscópica de nasoangiofibroma, 2006.

2.2.2 Telescopia laríngea rígida


É realizada sob anestesia tópica com spray de lidocaína na
orofaringe, com introdução do aparelho pela cavidade oral. O
endoscópio laríngeo rígido fornece imagem com mais detalhes do
que o flexível, mas tem mobilidade reduzida e desencadeia maior
reflexo de náuseas.
Figura 2.12 - Endoscopia laríngea com aparelho rígido

2.2.3 Telescopia laríngea com estroboscopia


Usa-se a mesma técnica descrita pela telescopia laríngea rígida,
porém associada a uma fonte de luz estroboscópica. Essa luz é
sincronizada com a frequência fundamental da voz e permite avaliar
a vibração das pregas vocais com grande riqueza de detalhes,
semelhante à câmera lenta. Esse método permite maior acurácia no
diagnóstico de alterações estruturais mínimas de laringe e de outras
pequenas lesões (Figura 2.13).
Figura 2.13 - Visualização estroboscópica da vibração das pregas vocais, na qual pode-se
avaliar tal processo com grande riqueza de detalhes

Fonte: Видеоларингоскопическая картина голосового аппарата у пациентов с


различными формами хронического ларингита [imagem videolaringoscópica da voz em
pacientes com várias formas de laringite crônica (tradução literal)], 2016.

2.2.4 Videoendoscopia da deglutição –


videodeglutograma
O exame é usado em casos específicos e avalia a função de deglutição
por meio da nasofibrolaringoscopia, realizada enquanto o indivíduo
ingere alimentos corados. É possível observar a movimentação da
língua levando o bolo alimentar, a elevação da laringe e o
fechamento da glote.
Quadro 2.1 - Principais testes endoscópicos faringolaríngeos

2.3 TESTES AUDITIVOS


2.3.1 Audiometria tonal
A audiometria determina, para cada frequência sonora audível, a
menor intensidade detectável em via aérea e via óssea (Figura 2.14).
São testadas frequências que vão de 250 a 8.000 Hz, e as
intensidades são medidas em decibéis (dB). Graficamente, obtém-se
uma curva em que, nas abscissas, estão as frequências sonoras
testadas e, nas ordenadas, está a intensidade sonora. Vale ressaltar
que, no gráfico audiométrico, há a “inversão” dos valores nas
ordenadas, ou seja, as maiores intensidades testadas encontram-se
na região mais baixa do gráfico e as menores, na região mais alta.
Figura 2.14 - Audiometria

Legenda: (A) audiometria por via aérea – o fone é colocado no canal auditivo e o som
percorre, na sequência, conduto auditivo, membrana timpânica, cadeia ossicular, cóclea,
nervo auditivo; (B) audiometria por via óssea – o diapasão é colocado junto ao osso
temporal, e a vibração sonora é transmitida diretamente à cóclea e, em seguida, ao nervo
auditivo.

Fonte: adaptada de Missorla.

2.3.1.1 Audiometria por via aérea

Determina a menor intensidade sonora detectada por sons


transmitidos por via aérea, ou seja, por meio de fone colocado no
ouvido (CAE) do paciente (Figura 2.15).
Figura 2.15 - Audiograma normal de via aérea

Nota: observe que os menores sons detectáveis pela orelha direita são marcados por
círculos vermelhos, enquanto a orelha esquerda é marcada por um “X” azul. Há, também,
uma reta marcando o limiar da normalidade. Considera-se que há alguma perda auditiva
quando a detecção do som ocorre em intensidades maiores que 25 dB.
Fonte: adaptado de Improving detection of adolescent hearing loss, 2011.

2.3.1.2 Audiometria por via óssea

Determina a menor intensidade sonora detectada por sons


transmitidos por via óssea, ou seja, por meio de vibrador ósseo
(diapasão) colocado na região retroauricular – mastoide. Assim,
transmite-se o som diretamente à cóclea sem o uso do complexo
tímpano-ossicular.
Os limiares de detecção dos sons por via aérea e óssea devem ser
coincidentes e menores do que 20 dB. Pacientes com detecção de
sons em intensidades acima dessa apresentam algum grau de perda
auditiva ou disacusia.
As curvas das vias aérea e óssea são representadas no mesmo
gráfico, e, quando presente, a diferença entre os limiares ósseos e
aéreos é denominada gap, ou diferença aéreo-óssea. Os principais
tipos de perda auditiva são:
1. Neurossensorial:
a) Os limiares auditivos por via aérea e via óssea coincidem e
ambos são maiores do que 25 dB;
b) Geralmente esse tipo de perda decorre de lesões cocleares
como presbiacusia, ototoxicidade e perda por ruído ou nervosas –
neurinomas do NC VIII.
2. Condutiva:
a) O limiar auditivo por via aérea é maior do que 25 dB, e o limiar
auditivo por via óssea está dentro do normal (menor do que 25
dB);
b) É sugestivo de patologias exclusivas da orelha média, como as
otites médias – aguda, crônica e secretora – e a desarticulação da
cadeia ossicular pós-traumática.
3. Mista:
a) Tanto o limiar ósseo quanto o aéreo são maiores do que 25 dB,
mas não são coincidentes, sendo o limiar por via aérea maior do
que o limiar por via óssea;
b) É um tipo de perda menos comum, em geral decorrente de
patologias que acometem as orelhas média e interna (como
otosclerose ou alguns traumas de osso temporal).
2.3.1.3 Audiometria vocal

O examinador pronuncia palavras padronizadas em intensidades


predeterminadas de acordo com a audiometria tonal e solicita ao
paciente que as repita. Esse exame tem a função de confirmar o
limiar audiométrico por via aérea e determinar a capacidade de
discriminação da fala.
2.3.2 Timpanometria ou impedanciometria
Mede a complacência ou mobilidade da membrana timpânica em
função de pressões aplicadas no CAE. São possibilidades de curvas de
complacência:
1. Curva A: é normal;
2. Curva Ar ou As: é sugestiva de rigidez da cadeia ossicular, como
na otosclerose. Essa rigidez restringe a movimentação da membrana
timpânica;
3. Curva Ad: é sugestiva de desarticulação da cadeia ossicular ou de
formação de cicatrizes no tímpano – neotímpano. Nessa situação, a
ausência de pressão bariátrica na membrana permite sua intensa
mobilidade;
4. Curva B: é sugestiva de secreção na orelha média. A secreção
impede a mobilização tanto interna quanto externa da membrana, de
forma a não haver formação de “pico” de mobilidade;
5. Curva C: é sugestiva de disfunção da tuba auditiva. A membrana
encontra-se retraída em direção à caixa timpânica. Por isso, quando
aplicada pressão negativa no CAE, a membrana é “aspirada” de volta
à sua posição original e, assim, tem seu pico de mobilidade em
pressões negativas.

2.3.3 BERA (audiometria de tronco encefálico)


A condução do estímulo sonoro até o sistema nervoso central segue
este caminho (Figura 2.16):
1. Vibração da membrana timpânica;
2. Vibração da cadeia ossicular e estimulação da cóclea;
3. Ativação do órgão de Corti e ativação do nervo auditivo NC VIII;
4. Ativação do núcleo coclear ventral;
5. Ativação do núcleo coclear dorsal;
6. Ativação do complexo olivar superior;
7. Ativação do núcleo do lemnisco lateral.

O BERA é realizado por meio da estimulação sonora no CAE com


fones de ouvido e a captação do impulso nervoso gerado por tal
estímulo. Para isso, são usados eletrodos fixos à pele do paciente.
Obtém-se uma curva que demonstra os instantes em que o estímulo
atingiu as principais estruturas anatômicas da via auditiva (Figura
2.16). O BERA é um teste objetivo que não depende da resposta
voluntária do indivíduo. Aplica-se esse teste em:
Pacientes com suspeita de simulação de perda auditiva;
Crianças de até 4 anos, pois mede grosseiramente o limiar auditivo e
independe da resposta do indivíduo;
Triagem de lesões neurodegenerativas ou neurinomas de NC VIII:
quando presentes, elas retardam o tempo de aparecimento da onda I.
Figura 2.16 - Estruturas centrais avaliadas e suas ondas geradas (I a VII) na audiometria
de tronco encefálico

Fonte: adaptado do site Neurologic Labs.


2.3.4 Emissões otoacústicas
O órgão de Corti, na orelha interna, transforma o estímulo mecânico
vibratório, endolinfa, em estímulo elétrico que percorre o NC VIII.
Nesse processo, o órgão gera ruídos que podem ser captados por
fones ultrassensíveis colocados no CAE.
Esse teste é utilizado como triagem auditiva neonatal, chamado
“teste da orelhinha”. A ausência de respostas pode indicar perda
auditiva de graus variados, mas precisa ser mais bem avaliada por
apresentar muitos falsos negativos e deve ser testada novamente em
30 dias.
2.3.5 Eletrococleografia
Usada na suspeita de doença de Ménière, testa os potenciais
nervosos gerados no NC VIII após a estimulação sonora.
Seus principais parâmetros são a intensidade do Potencial de Ação
(PA) e do Potencial de Somação (PS), gerados no nervo. A relação PS-
PA maior do que 30% sugere hidropisia endolinfática.
2.4 TESTE OTONEUROLÓGICO
A avaliação do comportamento do nistagmo é fundamental para o
correto diagnóstico das vestibulopatias periféricas.
Nesse exame, testa-se a motilidade ocular voluntária e involuntária,
além da ocorrência e da intensidade no nistagmo em diversas
situações; o principal passo desse teste é realizado quando se
procede ao aquecimento ou resfriamento unilateral do CAE com água
corrente em temperatura entre 30 e 44 °C.
Nessas situações, geram-se correntes de convecção nos canais
semicirculares laterais, com consequentes estimulação labiríntica e
geração de estímulo neurológico de sensação de rotação. Em
seguida, há o aparecimento de nistagmo reflexo, cuja intensidade,
associada às demais informações obtidas no exame, permite sugerir
o tipo e a topografia da lesão labiríntica em curso.
2.5 POLISSONOGRAFIA
Consiste na avaliação quantitativa de parâmetros variáveis durante o
sono e da inter-relação entre esses parâmetros. É utilizada em
pacientes com suspeita de síndrome de apneia e hipopneia
obstrutiva do sono ou com outros distúrbios do sono. Nesse teste, o
paciente dorme no laboratório, e monitorizam-se,
simultaneamente:
1. Frequência cardíaca;
2. Frequência respiratória;
3. Oximetria;
4. Pressão arterial;
5. Tônus muscular;
6. Eletroencefalograma;
7. Fluxos aéreos nasal e oral;
8. Expansões torácica e abdominal;
9. Decúbito;
10. Ocorrência e intensidade de roncos;
11. Movimentos oculares.

Diante desses parâmetros, podem-se determinar a ocorrência e a


frequência média de apneias noturnas e seu grau de interferência na
qualidade do sono – Índice de Apneia e Hipopneia (IAH). Nos
adultos, os índices são definidos como:
1. Normal: 0 a 5;
2. Apneia leve: 5 a 10;
3. Apneia moderada: > 15 e < 30;
4. Apneia grave: ≥ 30.
Em crianças, os parâmetros de normalidades
são discutidos, porém aceita-se como normal
IAH de 0 a 1, sendo IAH maior que 1 sugestivo de
Síndrome da Apneia Obstrutiva do Sono (SAOS).

As apneias podem ser classificadas quanto à origem como:


1. Obstrutivas: decorrem de colabamento das vias aéreas durante o
sono, secundariamente a hipotonia muscular e alterações anatômicas.
Não há diminuição do estímulo neurológico para a respiração, e as
contrações musculares torácicas-diafragmáticas estão mantidas;
2. Centrais: decorrem da redução da ventilação por diminuição do
estímulo nervoso para a respiração durante o sono. É comum em
neuropatias;
3. Mistas: trata-se da combinação das apneias obstrutivas e centrais,
situação mais rara.
Em uma audiometria, por
que o limiar sonoro por via
aérea deve ser maior ou
igual ao limiar medido por
via óssea?
Conforme discutido no capítulo, para medição do limiar
por via aérea, o fone é colocado na abertura do conduto
auditivo externo. Dessa forma, o som deve atravessar: o
conduto auditivo, membrana timpânica e cadeia ossicular
até alcançar a cóclea, onde o estímulo vibratório sonoro é
convertido em estímulo nervoso. Já para medição do limiar
por via óssea, coloca-se um vibrador ósseo (diapasão) em
contato com alguma saliência do osso temporal,
geralmente a ponta da mastoide; dessa forma, o estímulo
vibratório ósseo estimula diretamente a cóclea sem passar
pelas 3 primeiras estruturas. Sendo assim, o limiar por via
aérea deve ser maior ou igual àquele medido por via óssea.
Paciente do sexo masculino,
18 meses. Mãe refere que há
2 dias apresenta febre de 38
°C, diminuição da aceitação
alimentar e prostração leve.
Ao exame físico, otoscopia
normal à direita e
abaulamento com
hiperemia à esquerda.
Devemos dar antibiótico a
esse paciente?

3.1 DOENÇAS DA ORELHA EXTERNA


3.1.1 Oto-hematoma
1. Definição: coleção sero-hemática entre o pericôndrio e a pele,
geralmente após trauma local;
2. Quadro clínico: abaulamento local, em geral na região da hélice, de
coloração vinho-acastanhada, com flutuação e dor à palpação;
3. Diagnóstico: eminentemente clínico;
4. Tratamento: deve ser feito por drenagem local, seguida de curativo
compressivo por 48 horas. Deve-se estabelecer a antibioticoterapia
oral (cefalexina).

3.1.2 Pericondrite
1. Definição: infecção bacteriana da cartilagem e do pericôndrio do
pavilhão auditivo;
2. Etiologia: em geral, ocorre após trauma com solução de
continuidade com o meio externo ou pelo uso de brincos ou piercings.
Também pode ocorrer como evolução de otites externas. Os principais
agentes são os germes da pele (Streptococcus, Staphylococcus) e
Pseudomonas aeruginosa;
3. Quadro clínico: hiperemia, dor e edema do pavilhão auricular de
rápida evolução, após trauma local;
4. Diagnóstico: eminentemente clínico;
5. Tratamento: deve ser feito de forma agressiva, com
antibioticoterapia oral (ciprofloxacino), associada a corticoterapia oral
(prednisona, dexametasona). Em casos de formação de abscesso,
deve ser drenado, e devem ser mantidos dreno e curativo
compressivo. Deve-se, também, considerar o uso de pomadas com
antibióticos na região da laceração traumática.

As principais complicações são a necrose tecidual e a ocorrência de


deformidades no pavilhão após a recrudescência da infecção.
Quadro 3.1 - Patologias que acometem o pavilhão auricular
3.1.3 Otite externa difusa aguda
1. Definição: infecção aguda da derme e da epiderme do Conduto
Auditivo Externo (CAE);
2. Agentes: P. aeruginosa, S. aureus, Streptococcus e Proteus
mirabilis. Seus fatores predisponentes mais comuns são:
a) Exposição à umidade: justifica maior incidência após piscina,
banhos de imersão e em nadadores;
b) Traumas: os externos, incluindo os por uso de hastes de
algodão e corpos estranhos, podem gerar pequenas lacerações
da pele do CAE, gerando solução de continuidade com o meio
externo e facilitando a infecção.
3. Composição do manto lipídico: a secreção das glândulas
ceruminosas, quando ausentes, ou alterações na composição da
secreção das glândulas locais podem aumentar o pH e favorecer a
penetração bacteriana;
4. Quadro clínico: otalgia moderada a intensa, geralmente unilateral,
que piora à compressão do trágus ou manipulação do pavilhão
auricular. Não se observam alterações sistêmicas como febre e queda
do estado geral. A otoscopia é dolorosa, e observam-se edema difuso
do CAE, chegando a ocluí-lo, hiperemia, descamação e secreção
(Figura 3.1);
5. Diagnóstico: eminentemente clínico; quando possível, deve-se
realizar cultura de secreção;
6. Tratamento: de acordo com a gravidade da doença:
a) Casos leves: gotas tópicas contendo antibióticos e corticoides;
b) Casos intensos: associar antibióticos orais (cefalexina,
cefadroxila ou ciprofloxacino)..

O paciente também deve ser orientado a proteger o ouvido contra a


entrada de água no CAE durante o tratamento.
Figura 3.1 - Otite externa difusa aguda

Fonte: Otite externa difusa aguda: um estudo prospectivo no verão do Rio de Janeiro,
2004.

3.1.4 Otite externa circunscrita


1. Definição: infecção em complexo pilossebáceo do CAE, de
localização bem definida e circunscrita (furúnculo);
2. Etiologia: causada principalmente por Staphylococcus aureus;
3. Quadro clínico: dor intensa local, evolução rápida, algumas vezes
com febre. À otoscopia, observa-se edema localizado com sinais
flogísticos, muito doloroso ao toque (Figura 3.2);
4. Tratamento: antibióticos com cobertura para Gram positivos, como
cefalexina; analgésicos e anti-inflamatórios também devem ser
utilizados. Em casos de abscessos organizados, deve-se proceder à
drenagem cirúrgica.

Figura 3.2 - Otite externa circunscrita

Nota: observe o abaulamento em região superior do conduto auditivo externo, com


aparente sinal de flutuação.

Fonte: site London ENT Clinic.

3.1.5 Rolha de cerume - epidérmica


1. Definição: acúmulo de cerume no CAE, formando uma rolha que
contém também debris celulares. Fisiologicamente, o cerume é
produzido por glândulas ceruminosas localizadas no terço externo do
CAE;
2. Quadro clínico: caracteriza-se por hipoacusia com plenitude
auricular, em geral desencadeado por uso de hastes de algodão ou
após imersão do ouvido em água. O diagnóstico é eminentemente
clínico, por meio da observação de rolha ocluindo total ou parcialmente
o CAE;
3. Tratamento: feito por meio da remoção do cerume com lavagem
com água morna ou com curetas especiais.

São situações que podem favorecer a formação da rolha de cerume:


1. Uso de hastes de algodão ou protetores auriculares: são objetos
que podem descolar o cerume do terço externo do CAE em direção
aos 2 terços internos, e, nessa região, o cerume não consegue ser
eliminado pelo CAE pelos mecanismos fisiológicos;
2. Variações anatômicas: CAEs estreitos ou tortuosos possuem maior
facilidade ao acúmulo de cerume;
3. Banho de imersão: a presença de água no CAE umedece uma
rolha previamente existente, tornando-a maior e mais oclusiva, dando
início aos sintomas.

3.1.6 Otite externa necrotizante ou maligna


1. Definição: infecção originária do CAE que se estende para as
estruturas adjacentes – articulação temporomandibular, pavilhão
auricular, orelha média, mastoide, base do crânio, orelha interna e
osso temporal. Tem caráter bastante invasivo, gerando destruição
tissular e intenso processo inflamatório;
2. Etiologia: Pseudomonas aeruginosa;
3. Fatores de risco: diabetes não controlado (90%), idosos e
imunossupressão como AIDS, doenças linfoproliferativas e
quimioterapias;
4. Quadro clínico: inicia-se como otite externa aguda refratária ao
tratamento clínico. A dor é de moderada a intensa, prurido, otorreia.
Sintomas sistêmicos como queda do estado geral, descompensação
diabética, febre e cefaleia. Hipoacusia com plenitude auricular e
otorreia são típicos. Com a evolução, pode acometer regiões
adjacentes, gerando dor à movimentação da articulação
temporomandibular, restrição à abertura oral – acometimento de
masseter –, adenopatia cervical e necrose dos tecidos adjacentes. A
invasão do osso temporal gera sinais de complicações secundárias,
como paralisia facial periférica – acometimento do nervo facial – e
formação de abscessos intracranianos ou intratemporais. Os nervos
cranianos mais comumente envolvidos são o VII, X, XI;
5. Exame físico: intenso edema e hiperemia no pavilhão auricular e no
CAE, secreção purulenta no CAE, ulcerações na derme, granulações
locais e dor intensa ao toque; paralisia facial pode ocorrer (Figura 3.3);
6. Diagnóstico laboratorial e exames complementares:
a) Cintilografia com gálio-67: marca os polimorfonucleares,
sendo um marcador de infecção ativa (osteomielite). Utilizada
para monitorizar a resposta terapêutica;
b) Cintilografia com tecnécio-99: mostra regiões osteoblásticas,
presentes em osteomielites. Usada para diagnóstico inicial da
doença;
c) Tomografia Computadorizada (TC) de mastoide:
inicialmente observa-se espessamento dos tecidos moles
relacionados ao osso temporal e base de crânio. Com a evolução,
os tecidos ósseos também se espessam;
d) Ressonância Nuclear Magnética (RNM) de mastoide:
fornece informações complementares à TC. Também avalia
possíveis alterações no Sistema Nervoso Central (SNC)
secundárias ao processo infeccioso;
e) Outros exames que devem ser solicitados são: cultura da
secreção do ouvido, hemocultura e exames gerais: hemograma,
proteína C reativa, velocidade de hemossedimentação, glicemia.
7. Tratamento:
a) Compensação das patologias associadas: citam-se diabetes
mellitus e compensação clínica geral – paciente internado;
b) Antibioticoterapia parenteral: ciprofloxacino como o
antibiótico mais comumente utilizado, porém outros agentes anti-
Pseudomonas devem ser considerados;
c) Abordagem cirúrgica: nos casos refratários ao tratamento
clínico, com debridamento das áreas necróticas, a abordagem
cirúrgica deve ser considerada.

Figura 3.3 - Otite externa maligna


Nota: observe a formação de tecido necrótico e de granulação em conduto auditivo e
pavilhão auricular.

Fonte: site Medical Foxx.

A maioria dos estudos atuais relata uma taxa de mortalidade de


menos de 10%, mas a mortalidade é mais alta para aqueles com
neuropatias cranianas, complicações intracranianas ou
imunossupressão sistêmica irreversível.
Figura 3.4 - Otite externa maligna
Nota: a seta aponta edema de tecidos e erosão óssea em região adjacente ao conduto
auditivo externo esquerdo.

Fonte: adaptado de Malignant External Otitis with Multiple Cranial Nerve Palsies: The Use
of Hyperbaric Oxygen, 2004.

3.1.7 Otomicose
1. Definição: infecção do CAE causada por fungos;
2. Agentes: dos gêneros Candida spp. e Aspergillus spp.;
3. Quadro clínico: desde pouco sintomático, como, por exemplo,
prurido isolado, até com sintomas mais intensos. À otoscopia,
observam-se os filamentos fúngicos, de coloração variável;
4. Tratamento: gotas tópicas contendo antifúngicos e limpeza local por
aspiração. Deve-se evitar umidade local até a resolução do quadro.
3.1.8 Miringite bolhosa
1. Definição: infecção na camada externa, epitelial, da Membrana
Timpânica (MT), formando bolha pela delaminação dessas camadas,
contendo secreção serosa em seu interior;
2. Etiologia: ainda não está totalmente elucidada, mas acredita-se que
seja decorrente de infecção viral pós-Infecção das Vias Aéreas
Superiores (IVAS) ou bacteriana, sendo possivelmente os mesmos
agentes da Otite Média Aguda (OMA);
3. Quadro clínico: intensa otalgia, geralmente unilateral, após IVAS.
Febre e outros sintomas gerais geralmente não são observados.
Observam-se hipoacusia e plenitude e, ao exame físico, bolha com
conteúdo líquido na MT, com hiperemia local e CAE preservado;
4. Diagnóstico: eminentemente clínico. Audiometria pode evidenciar
perda condutiva ou mista – transitória;
5. Tratamento: proteção contra umidade, antibioticoterapia com
claritromicina ou eritromicina, e corticoterapia oral e tópica.

A punção da bolha tem validade questionável.


Muitos autores sugerem não realizá-la.

Figura 3.5 - Histologia da membrana timpânica


Legenda: (1) conduto auditivo externo; (2) epitélio do conduto auditivo externo; (3) fibras
colágenas orientadas radialmente; (4) fibras colágenas orientadas circularmente; (5)
mucosa de orelha média; (6) orelha média; (7) plano de clivagem (entre epitélio e camada
externa de colágeno) na miringite bolhosa.

Fonte: adaptado de HistoWeb.

Quadro 3.2 - Patologias que acometem o conduto auditivo


3.2 DOENÇAS NÃO INFECCIOSAS DA
ORELHA MÉDIA
3.2.1 Disfunção tubária
1. Definição: incapacidade da tuba auditiva em se abrir
adequadamente e equalizar a pressão no interior da orelha média com
a pressão da cavidade nasal, pressão ambiente;
2. Etiologia: secundária ao edema da mucosa da tuba em vigência de
IVAS ou processos alérgicos; pode acontecer de forma crônica,
secundário a malformações craniofaciais, OMAs de repetição ou sem
fator evidente;
3. Quadro clínico: plenitude auricular associada a hipoacusia e
eventual zumbido. À otoscopia, observa-se a MT retraída para a orelha
média;
4. Diagnóstico: clínico; curva C à timpanometria e presença de gap
aéreo-ósseo;
5. Tratamento: direcionado à doença de base – antialérgicos e
descongestionantes são bastante eficazes. Movimentos repetitivos de
deglutição, como chupar bala ou mascar chiclete, ativam os músculos
tubários e auxiliam na abertura da tuba auditiva. Em casos refratários a
tratamento clínico, indica-se timpanotomia e tubo de ventilação.
3.2.2 Otosclerose (otospongiose)
1. Definição: patologia que acomete multifocalmente o labirinto ósseo,
notadamente na região da janela oval. Ocorre um processo de
reabsorção e síntese óssea que pode levar, no final, à fixação da
platina do estribo na janela oval, com consequente rigidez na vibração
da cadeia ossicular, que passa a não transmitir adequadamente as
vibrações sonoras;
2. Fatores de risco: mulheres jovens com pico de incidência entre a
terceira e quinta décadas de vida, antecedente de gestação;
3. Quadro clínico: hipoacusia progressiva com zumbidos; podem
também ocorrer sintomas vestibulares. O exame físico costuma ser
normal;
4. Testes complementares:
a) Audiometria: perda auditiva condutiva ou mista, acometendo
inicialmente as menores frequências – graves. Pode ocorrer
perda auditiva mais pronunciada na frequência de 2 kHz,
fenômeno conhecido como entalhe de Carhart (Figura 3.6);
b) Imitanciometria: caracterizada por curva tipo “Ar”;
c) Evolução: as demais frequências podem ser acometidas e a
perda auditiva tornar-se do tipo mista pela invasão dos focos
otoscleróticos no ouvido interno (Figura 3.7).
5. Tratamento:
a) Clínico: readaptação auditiva por meio de Aparelhos de
Amplificação Sonora Individual (AASIs), terapia medicamentosa
com fluoreto de sódio ou alendronato de sódio, que tem efeito
inibidor na progressão da doença;
b) Cirúrgico: substituição do estribo por uma prótese (Figura
3.8). Em casos severos pode-se considerar próteses implantáveis
ou implante coclear.

Figura 3.6 - Entalhe de Carhart na orelha direita


Nota: observar os limiares preservados na orelha esquerda.

Figura 3.7 - Disacusia mista bilateral, secundária a otosclerose de longa evolução


Fonte: adaptado de Malformações congênitas da orelha média - causa rara de hipoacusia
pediátrica, 2014.

Figura 3.8 - Tratamento cirúrgico da otosclerose


Legenda: (A) foco de ossificação da janela oval com fixação do estribo; (B) remoção das
cruras do estribo e alocação de prótese entre a bigorna e a platina do estribo.

#IMPORTANTE
As principais características da otosclerose são
perda auditiva condutiva de caráter progressivo,
sexo feminino, piora após gestação e fixação do
estribo na janela oval.
Quadro 3.3 - Patologias não infecciosas do ouvido

3.2.3 Traumas
Os traumas do osso temporal são divididos, didaticamente,
conforme segue.
3.2.3.1 Trauma do conduto auditivo externo
1. Mecanismo de trauma: geralmente secundário a uso de hastes de
algodão ou corpos estranhos;
2. Quadro clínico: dor intensa e sangramento, sem alteração da
acuidade auditiva. O exame físico evidencia laceração do conduto
auditivo e MT íntegra;
3. Tratamento: proteção contra umidade, gotas tópicas contendo
antibióticos e analgesia (Figura 3.9).

Figura 3.9 - Laceração do conduto auditivo externo, sem acometimento da membrana


timpânica

Fonte: site Otoscopy.

3.2.3.2 Perfuração timpânica traumática


1. Mecanismo de trauma: penetração de corpo estranho ou por
aumento súbito de pressão no CAE – “tapa na orelha”;
2. Quadro clínico: otalgia associada a otorragia e hipoacusia. À
otoscopia, observam-se MT lacerada e sangramento local;
3. Tratamento: proteção auricular e observação clínica, uma vez que
85 a 95% das perfurações traumáticas cicatrizam espontaneamente
(Figura 3.10), em geral até 2 meses. Caso não haja fechamento da
perfuração, deve-se considerar a correção cirúrgica.

Figura 3.10 - Perfuração traumática da membrana timpânica à esquerda

Observar a presença de sangue no conduto auditivo externo e a perfuração na membrana


timpânica, além da cadeia ossicular.

Fonte: site Ear, Nose and Throat - USA.

3.2.3.3 Desarticulação de cadeia ossicular


1. Mecanismo de trauma: traumas penetrantes ou fechados, de maior
energia;
2. Quadro clínico: hipoacusia súbita após o trauma, podendo estar
associado ou não a perfuração da MT;
3. Diagnóstico: com base na audiometria – disacusia, condutiva ou
mista –, com timpanometria evidenciando curva tipo Ad. A tomografia
de mastoide também pode mostrar a região acometida;
4. Tratamento: protetização auditiva e/ou reconstrução cirúrgica da
cadeia ossicular com próteses.

3.2.3.4 Fraturas do osso temporal

a) Longitudinal

1. Mecanismo de trauma: geralmente secundário a trauma de leve a


moderada energia; cerca de 80% das fraturas de ossos temporais são
longitudinais, resultantes de traumas em direção lateral ao crânio na
região parietal da cabeça;
2. Quadro clínico: varia de acordo com a estrutura acometida, porém
comumente não cursa com lesão de estruturas nobres – nervo facial e
cápsula ótica –, mas pode causar desarticulação da cadeia ossicular;
3. Diagnóstico: tomográfico; as estruturas mais envolvidas são a MT,
o teto da caixa timpânica e a porção anterior do ápice petroso;
4. Tratamento: possivelmente conservador nos casos menos graves.

b) Transversa

1. Mecanismo de trauma: geralmente secundário a trauma lateral ou


frontal de alta energia. A linha de fratura comumente acomete
estruturas nobres, como o canal do nervo facial, a cápsula ótica e a
base do crânio;
2. Quadro clínico: varia de acordo com as estruturas acometidas; em
geral envolvem paralisia facial periférica, hipo ou anacusia, vertigem
severa e até mesmo otorreia hialina em casos de fratura de base do
crânio e formação de fístula labiríntica;
3. Diagnóstico: com base em tomografia;
4. Tratamento: deve ser considerado de acordo com a estrutura
acometida. Nos casos de paralisia facial, devem-se considerar a
mastoidectomia e a descompressão do canal do NC VII.
Quadro 3.4 - Traumas que acometem estruturas nobres

3.3 DOENÇAS INFECCIOSAS-


INFLAMATÓRIAS DA ORELHA MÉDIA
3.3.1 Otite média aguda
1. Definição: a Otite Média Aguda (OMA) é um processo infeccioso da
orelha média que pode ser viral ou bacteriano;
2. Etiologia: os principais agentes etiológicos são:
a) Bacterianos:
Em recém-nascidos e até a sexta semana de vida: S. aureus
ou enterobactérias (E. coli, Klebsiella spp. e Enterobacter
spp.)
Após a sexta semana de vida: em ordem decrescente de
frequência, S. pneumoniae, H. influenzae e M. catarrhalis.
b) Virais: vírus sincicial respiratório, adenovírus, influenza A ou B.
3. Fisiopatologia: a OMA é, na maioria das vezes, secundária a IVAS
e processos inflamatórios-alérgicos das vias aéreas superiores.
Nessas situações, há edema mucoso e disfunção na tuba auditiva,
favorecendo a estase de secreção na orelha média;
4. Epidemiologia: as crianças estão mais sujeitas à ocorrência de
OMA, observando-se 2 picos de incidência – entre 6 e 24 meses e
entre 4 e 7 anos;
5. Fatores de risco: a maior incidência em crianças deve-se,
principalmente, a:
a) OMAs ou IVAS de repetição previamente;
b) Anatomia: a tuba auditiva mais curta e horizontalizada de
crianças predispõe à aspiração de secreções da nasofaringe até o
ouvido médio;
c) Hábito de mamar deitado;
d) Aleitamento materno por menos de 3 meses;
e) Frequência a creches ou berçários (exposição a IVAS);
f) Tabagismo passivo;
g) Doença do refluxo gastroesofágico;
h) Atopias;
i) Meses de inverno.
6. Quadro clínico: otalgia, otorreia, plenitude auricular, toxemia e
febre. Em crianças menores, são possíveis sinais e sintomas menos
específicos, como diarreia e prostração, além de dificuldade para
mamar;
7. Diagnóstico: eminentemente clínico. A otoscopia é fundamental e
muitas vezes permite diferenciar a etiologia da otite. A avaliação da MT
deve incluir: posição – normal, retraída, abaulada; cor – rosada, âmbar,
branca, vermelha, azul; translucidez – translúcida, semiopaca, opaca;
mobilidade, quando possível – normal, aumentada, diminuída, ausente;
a) OMAs virais: a MT está hiperemiada – sinais de aumento da
vascularização timpânica – e com transparência diminuída;
b) OMAs bacterianas: além dos sinais anteriores, observa-se MT
abaulada devido à secreção contida no interior do ouvido médio.
8. Tratamento: visa aliviar os sinais e sintomas e prevenir
complicações. No entanto, a opção por introduzir a antibioticoterapia
deve considerar a certeza do diagnóstico (vide adiante).

Figura 3.11 - Membrana timpânica

Nota: observar a membrana timpânica abaulada, opacificada e hiperemiada.

Legenda: (A) MT normal; (B), (C) e (D) – diferentes estágios de otite média aguda.

Fonte: The diagnosis and management of acute otitis media, 2013.

Figura 3.12 - Orelha média em vigência de otite média aguda


Nota: observar o preenchimento da cavidade por secreção purulenta.

Fonte: adaptado de Svetlana Verbinskaya.

Quadro 3.5 - Diagnóstico de otite média aguda bacteriana

Apesar de os protocolos não serem consensuais, a American


Academy of Pediatrics orienta seguir alguns critérios para iniciar
antibioticoterapia (Quadro 3.6).
Quadro 3.6 - Indicação de antibioticoterapia na otite média aguda
A escolha da antibioticoterapia deve ser feita da seguinte forma:
1. Primeira escolha: amoxicilina. Em caso de falha terapêutica em 48
horas, deve ser aumentada a dose de amoxicilina. Alternativa para
alérgicos: claritromicina ou clindamicina;
2. Segunda escolha: amoxicilina + ácido clavulânico ou
axetilcefuroxima – na presença de falha terapêutica, deve ser usada
para eliminar Haemophilus influenzae e Moraxella catarrhalis
resistentes;
3. Terceira escolha: ceftriaxona por 3 dias consecutivos intramuscular
em casos de vômitos persistentes ou em outras situações que não seja
possível receber medicação oral. Pacientes sem resposta a amoxicilina
com ácido clavulânico devem receber 3 dias de ceftriaxona
intramuscular.

O paciente deve ser reavaliado em 48 a 72 horas após o início do


tratamento e fatores como dor, febre e estado geral devem ser
considerados. Nos casos em que não há alívio dos sintomas, pode-se
optar pela ampliação do espectro antimicrobiano.
3.4 TRATAMENTOS ASSOCIADOS
A miringotomia, microincisão de alívio na MT, visa à drenagem da
coleção purulenta da orelha média e deve ser considerada em
pacientes refratários ao tratamento clínico, dor intensa e vigência de
complicações. Vale ressaltar que a perfuração na MT cicatriza após a
remissão do quadro infeccioso.
Anti-inflamatórios não hormonais e antitérmicos complementam o
tratamento. Gotas tópicas otológicas não são usadas de rotina.
A vacina conjugada do pneumococo – PCV7 antes de 24 meses –
mostrou-se efetiva na proteção contra otite, com redução dos casos
e das visitas ao médico. Sabe-se que a maioria das otites é de causa
viral e que 2 terços das crianças com influenza podem tê-la. A
introdução da vacinação contra o vírus influenza mostrou eficácia,
diminuindo em 30% a 55% as otites durante o inverno. A vacina é
recomendada a partir de 6 meses.
3.4.1 Otite média aguda recorrente
1. Definição: episódios frequentes de OMA intercalados por períodos
de normalidade;
2. Fatores de risco: os fatores predisponentes são semelhantes aos
da OMA, associados a hipertrofia adenoidiana, imunodeficiências,
deficiência de IgA, malformações craniofaciais;
3. Agentes etiológicos: em geral, são os mesmos da OMA. Deve-se
atentar também para tratamentos prévios feitos incorretamente ou para
possível resistência a antibióticos;
4. Tratamentos: tratamento dos fatores de base; a miringotomia com
alocação de tubo de ventilação apresenta bons resultados. A
quimioprofilaxia da OMA recorrente por meio do uso prolongado de
antimicrobianos (geralmente amoxicilina) em baixa dose tem efeitos
comprovados e pode ser considerada antes da cirurgia.

3.4.2 Otite média secretora


1. Definição: também denominada otite média serosa, otite média com
efusão, a Otite Média Secretora (OMS) caracteriza-se pela persistência
de líquido no interior do ouvido médio; frequentemente está associada
a OMA de repetição;
2. Epidemiologia: há 2 picos de incidência, entre 6 e 13 meses e aos
5 anos. Deve-se lembrar que, após um episódio de OMA, é possível
persistir certa quantidade de líquido no ouvido médio, podendo levar
até 8 semanas para ser completamente reabsorvido. Esse líquido pode
ou não ser o ponto de partida para a OMS;
3. Fatores de risco: semelhantes a OMA recorrente;
4. Quadro clínico: caracteriza-se pelo histórico de otites recorrentes e
queixa de hipoacusia. Ao exame físico, encontram-se diminuição da
transparência da MT, aumento de sua vascularização radial, coloração
variável e áreas de retração;
5. Diagnóstico: os testes audiométricos indicam a disacusia
condutiva, e a impedanciometria mostra alteração de curva tipo B;
6. Tratamento: no início e nos casos mais leves, pode ser clínico, com
a associação de antibióticos e corticoides. Nos casos refratários ao
tratamento clínico, está indicada miringotomia com colocação de tubos
de ventilação na MT, associada a adenoidectomia quando necessário.

São fatores de risco para a OMS idade menor do


que 6 anos, hiperplasia de adenoide,
malformação craniofacial, patologias da tuba
auditiva e doença do refluxo gastroesofágico.

3.4.3 Otite média crônica


1. Definição: a Otite Média Crônica (OMC) pode ser definida como um
processo inflamatório crônico da orelha média associado à perfuração
da MT (Figura 3.13);
2. Quadro clínico: otorreia recorrente, principalmente ao molhar o
ouvido, hipoacusia e zumbido; sintomas menos comuns são plenitude
auricular, otalgia e otorragia;
3. Fatores de risco: disfunções da tuba auditiva, mau estado
nutricional, distúrbios imunológicos, hiperplasia adenoidiana e
malformações craniofaciais. Nesses pacientes o médico deve
investigar cuidadosamente fatores predisponentes de otite e tratá-los;
4. Tipos: divide-se em 3 subtipos – simples, supurativa e
colesteatomatosa.

Figura 3.13 - Otoscopia em otite média crônica: notar a perfuração da membrana


timpânica

Fonte: Welleschik, 2006.


3.4.3.1 Otite média crônica simples

1. Definição: caracteriza-se por perfuração da MT, associada a pouca


ou nenhuma alteração na mucosa da orelha média. Em geral, a
perfuração acontece após OMA supurada, ou perfuração traumática;
2. Quadro clínico: hipoacusia e otorreias em crises, principalmente ao
molhar o ouvido e em vigência de IVAS. A otoscopia mostra perfuração
na MT sem alterações significativas na caixa timpânica;
3. Diagnóstico: basicamente clínico; na audiometria, o paciente
apresenta gap aéreo-ósseo pelo comprometimento da condução
sonora. Os casos de OMC simples dispensam exames de imagem.
Quando realizada, a TC de mastoide em geral não apresenta
alterações;
4. Tratamento: a proteção auricular contra água deve ser realizada
diariamente;
a) Crises de otorreias – reagudizações: gotas tópicas com
ciprofloxacino. Antibioticoterapia por via oral (amoxicilina e ácido
clavulânico) pode ser utilizada nos casos mais intensos;
b) Tratamento em longo prazo: pode ser feito com
acompanhamento clínico-audiométrico periódico sem abordagem
cirúrgica ou por meio de timpanoplastia para a reconstrução da
perfuração da MT.

3.4.3.2 Otite média crônica supurativa

1. Definição: processo inflamatório persistente da mucosa da orelha


média, associado a perfuração da MT, sem lamelas de colesteatoma;
2. Quadro clínico: hipoacusia e otorreia fétida persistente, refratária
ao tratamento clínico com gotas e antibioticoterapia oral. Ao exame
físico, há perfuração parcial ou total da MT e mucosa com aspecto
inflamatório, além de eventual presença de pólipos na orelha média;
3. Diagnóstico: a audiometria evidencia gap aéreo-ósseo. Nesse
caso, a investigação radiológica por tomografia é fundamental,
principalmente para fazer o diagnóstico diferencial com colesteatomas.
A TC evidencia sinais de mastoidite crônica (mastoide ebúrnea), sem
sinais de erosão ou destruição óssea. Em geral, a cadeia ossicular
está íntegra ou parcialmente erodida;
4. Tratamento: em geral, é cirúrgico, com mastoidectomia simples e
reconstrução da MT.
3.4.3.3 Otite média crônica colesteatomatosa

1. Definição: processo inflamatório persistente da mucosa da orelha


média, associado à perfuração da MT e à presença de colesteatoma
na orelha média. O colesteatoma (Figura 3.14) é uma estrutura lamelar
que histologicamente apresenta estrutura epitelial do tipo escamoso
estratificado queratinizado que cresce descontroladamente (formando
lamelas) e apresenta comportamento lítico em relação às estruturas
adjacentes, destruindo o ouvido progressivamente. Esse
comportamento é denominado pseudotumoral. As lamelas podem
infectar-se por diversas bactérias, como S. aureus e Pseudomonas,
produzindo otorreia de odor muito fétido e característico, com grande
prejuízo psicossocial. Os colesteatomas podem ser classificados em:
a) Congênitos: podem ocorrer em diversos locais do osso
temporal. Têm crescimento lento e, quando sintomáticos,
geralmente já atingiram grandes proporções. À otoscopia,
caracterizam-se por manterem a MT intacta, crescendo na
cavidade timpânica. Com a progressão, podem, também, destruir
a MT e as estruturas da orelha média;
b) Adquiridos: podem ser classificados em primários e
secundários. Os primários decorrem de retração crônica da MT na
sua porção flácida. Os secundários originam-se por OMA de
repetição, com perfuração da MT, que permite a migração de
tecido epitelial do CAE para o interior do ouvido médio (Figura
3.14).

Figura 3.14 - Colesteatoma após coloração com hematoxilina e eosina


Legenda: (A) perimatriz, contendo tecido conectivo (colágeno, fibras elásticas e células
inflamatórias); (B) matriz: contendo células semelhantes à pele normal; (C) conteúdo cístico
formado por queratina.

Fonte: adaptado de Descrição histológica de colesteatomas adquiridos: comparação entre


amostras de crianças e de adultos, 2006.

2. Diagnóstico: à otoscopia, pode-se observar secreção muito fétida,


de coloração variável, com perfuração da MT geralmente em sua
porção flácida, formando bolsas de retração com secreção e
descamação epitelial no seu interior. A TC de osso temporal – cortes
axiais e coronais – mostra uma massa de tecido mole posterossuperior
na caixa timpânica, homogênea, não calcificada, de contornos
lobulados, com efeito de massa com deslocamento medial da cadeia
ossicular, osteólise de estruturas finas, como os ossículos, e retração
da MT;
3. Tratamento: é cirúrgico, por meio de mastoidectomia subtotal ou
radical. Muitas vezes é necessário mais de 1 procedimento além do
acompanhamento periódico ambulatorial.

Quadro 3.7 - Patologias da orelha média


3.4.4 Complicações das otites médias
3.4.4.1 Otite média aguda supurada

1. Definição: nos casos de OMA bacteriana, em que há formação


rápida de grande quantidade de secreção purulenta na orelha média,
pode haver perfuração da MT e extravasamento de secreção purulenta
ou piossanguinolenta para o CAE;
2. Quadro clínico: semelhante a OMA, com otorreia;
3. Diagnóstico: é clínico, com base na evolução do quadro. Ao exame
físico, observa-se grande quantidade de secreção pio-hemática no
conduto auditivo. A MT, quando visível, apresenta-se hiperemiada,
espessada e abaulada. Em geral não se consegue observar o ponto de
perfuração da MT;
4. Tratamento: deve-se iniciar antibioticoterapia com amoxicilina +
clavulanato ou axetilcefuroxima. O uso de gotas com antibióticos é
controverso;
5. Evolução: cerca de 95% dos casos apresentam cicatrização
espontânea da perfuração. Quando a perfuração persiste, deve-se
considerar a timpanoplastia após a adolescência.

3.4.4.2 Mastoidite aguda

A mastoidite aguda é a complicação supurativa da OMA que afeta a


porção mastoide do osso temporal. A secreção no ouvido médio pode
penetrar no antro mastoide e disseminar-se para as demais células
mastoides. Após o preenchimento dessas células com secreção
purulenta, pode haver disseminação para o espaço extraósseo, com
edema subcutâneo.
As principais características da mastoidite são quadro de OMA
precedente, erosão do trabeculado ósseo da mastoide, maior
frequência em crianças de até 5 anos e abaulamento retroauricular.
Figura 3.15 - Mastoidite aguda (tomografia computadorizada)
A hiperemia e o edema retroauricular não têm
evidências de flutuação na mastoidite aguda. A
tomografia mostra preenchimento de células
mastoides, sem evidências de coleções
organizadas.

Com a evolução da doença, há destruição das trabéculas ósseas da


mastoide, gerando a coalescência das células – mastoidite
coalescente. Evolui-se com destruição da cortical externa da
mastoide e extravasamento da secreção purulenta, com formação de
abscesso organizado subperiosteal retroauricular.
Figura 3.16 - Mastoidite coalescente à esquerda – tomografia computadorizada axial em
janela óssea com erosão da cortical óssea externa na mastoide
Figura 3.17 - Mastoide à esquerda – tomografia computadorizada axial em janela de partes
moles com contraste com coleção com realce periférico subperiosteal
Se a progressão da mastoidite acometer outros
locais da mastoide, poderão ocorrer
complicações intratemporais e até
intracranianas.

Figura 3.18 - Imagem radiológica de mastoide indicando mastoidite


Há queda importante do estado geral e febre. O principal agente
etiológico é o S. pneumoniae. A avaliação radiológica por meio de TC
de mastoide (cortes axiais e coronais) é fundamental para avaliar a
extensão da lesão e, principalmente, verificar se há a coalescência
dela. O tratamento inclui, além de antibioticoterapia intravenosa,
miringotomia para drenagem de secreções do ouvido médio (enviar
material para cultura e antibiograma). Incisão retroauricular para
drenagem de grandes abscessos mastoides também pode ser feita.
Em casos extremos, pode-se optar pelo tratamento cirúrgico
(mastoidectomia), que é realizado com drenagem seguida de
antibioticoterapia intravenosa.
Figura 3.19 - Possíveis formas de evolução das otites
Fonte: elaborado pelos autores.

3.4.4.3 Paralisia facial periférica

1. Definição: paralisia do NC VII por extensão do processo


inflamatório ou destruição do canal ósseo em que o nervo facial se
aloja, principalmente em indivíduos que apresentam deiscências
naturais desse canal;
2. Diagnóstico: baseia-se no quadro clínico. Deve-se realizar TC de
mastoides que, em geral, evidencia mastoidite e sinais de
coalescência;
3. Tratamento: antibioticoterapia, corticoterapia e miringotomia para
remoção de secreção. Se houver progressão da paralisia, indica-se
descompressão do nervo facial, com a abertura do canal do nervo
facial.

3.4.4.4 Labirintites

1. Definição: são processos infecciosos que acometem o ouvido


interno, em geral secundárias a OMA ou OMC. Há 2 tipos principais:
a) Serosa: há a entrada de toxinas bacterianas nos líquidos do
ouvido interno através da janela oval ou da redonda, mais
comum;
b) Supurativa: há a invasão dos próprios micro-organismos;
2. Quadro clínico: hipoacusia ou anacusia de rápida evolução,
vertigens severas e zumbido em paciente com OMA ou agudização de
OMC;
3. Diagnóstico: clínico, e podem-se solicitar exame de imagem – TC,
RNM – e audiometria;
4. Tratamento: antibioticoterapia intravenosa com ceftriaxona,
corticoterapia, depressores labirínticos e monitorização da audição e
da função vestibular. Nos casos mais severos, deve-se considerar a
abordagem cirúrgica, com labirintectomia e debridamento do osso
infectado.

3.4.4.5 Complicações intracranianas

Ocorrem por disseminação do processo infeccioso da orelha média


em direção ao SNC.
3.4.4.6 Meningite

1. Definição: é uma das principais complicações intracranianas, por


disseminação bacteriana hematogênica ou contiguidade por meio da
fina lâmina óssea – tegmen tympani –, que separa o ouvido médio do
SNC (Figura 3.20);
2. Agentes: os principais são S. pneumoniae, H. influenzae, S. aureus
e Streptococcus beta-hemolítico;
3. Quadro clínico: é típico de meningite – febre, queda do estado
geral, cefaleia com foto e fonofobia e vômitos “em jatos”, associado a
um processo infeccioso na orelha média. Em casos suspeitos, deve-se
investigar a possibilidade de meningite por punção lombar;
4. Tratamento: miringotomia associada a antibioticoterapia parenteral
cobrindo principalmente Gram positivos e produtores de
betalactamase. No caso de meningite secundária à OMC, devemos
cobrir também Gram negativos e anaeróbios.

Figura 3.20 - Tegmen tympani

Legenda: (A) estreita tábua óssea, tegmen tympani, que separa a meninge da orelha
média – área destacada; (B) tomografia computadorizada coronal da orelha direita, em
janela óssea: observar o – teg – delgado tegmen tympani, o – ttt – tendão do músculo
tensor do tímpano e a – coch – cóclea.

Fonte: adaptado do site Radiology Key.

3.4.4.7 Abscessos

Abscessos intracranianos secundários a otites são comumente


extradurais, decorrentes de erosão da cortical do osso temporal ou
do tegmen tympani. Os sintomas são semelhantes aos da meningite,
podendo também apresentar sinais focais dependendo da
localização. Nesses casos, é obrigatória a realização de exames de
imagem. O tratamento neurocirúrgico deve preceder os demais e
baseia-se na drenagem da infecção. A esse procedimento se associa
antibioticoterapia parenteral – ceftriaxona – por tempo prolongado.
Abscessos no parênquima encefálico ou no cerebelo ocorrem mais
comumente como complicação de OMC colesteatomatosa em estágio
avançado de evolução, quando há lise nas estruturas ósseas
adjacentes, formação de abscesso extradural e disseminação para o
parênquima cerebral ou cerebelar.
O quadro é típico de infecção no SNC associado a piora dos sintomas
otológicos crônicos, podendo haver sinais neurológicos focais. O
diagnóstico é tomográfico e o tratamento deve ser cirúrgico, com
drenagem do abscesso e remoção da doença originária do quadro
(mastoidectomia e remoção do colesteatoma).
3.4.4.8 Trombose de seio sigmoide

Pode ocorrer por contiguidade dessa estrutura ao osso temporal. A


infecção pode se disseminar por via hematogênica local ou por
erosão da cortical da mastoide. O quadro caracteriza-se por sinais de
infecção no SNC com hipertensão intracraniana – cefaleia intensa,
papiledema – e toxemia. O diagnóstico é tomográfico e o tratamento
consiste em abordagem cirúrgica, com exposição do seio e
antibioticoterapia.
3.5 DISTÚRBIOS DA ORELHA INTERNA
3.5.1 Distúrbios auditivos
3.5.1.1 Presbiacusia

1. Definição: processo degenerativo da orelha interna decorrente do


envelhecimento;
2. Epidemiologia: acomete indivíduos de ambos os gêneros, em geral
a partir da sexta década de vida;
3. Quadro clínico: perda auditiva progressiva, bilateral. O paciente
frequentemente relata que ouve, mas que não entende o que é dito;
4. Diagnóstico: audiometria mostra disacusia neurossensorial,
descendente e bilateral e simétrica (Figura 3.21);
5. Tratamento: correção de fatores associados, como exposição a
ruídos, doenças metabólicas como diabetes, hipertensão arterial, entre
outros. O ganho funcional pode ser feito com uso de Aparelhos de
Amplificação Sonora Individual (AASI).

Figura 3.21 - Audiometria em presbiacusia: padrão neurossensorial, simétrico e


descendente

3.5.1.2 Perda de audição induzida pelo ruído (PAIR)

1. Definição: destruição lenta e progressiva das células ciliadas do


órgão de Corti, gerada por exposição crônica a ruídos;
2. Epidemiologia: é a segunda maior causa de perda auditiva do tipo
neurossensorial na população e acomete, principalmente, o sexo
masculino. Na maior parte, é causada por exposição a ruídos no
ambiente de trabalho;
3. Quadro clínico: hipoacusia bilateral acompanhada de zumbidos;
4. Diagnóstico: a audiometria tonal (Figura 3.22) mostra perda
auditiva do tipo neurossensorial bilateral e simétrica, inicialmente
acometendo as frequências agudas – 3.000 a 6.000 Hz. Com a
evolução da doença, a perda pode estender-se até frequências mais
graves – 500 a 2.000 Hz;
5. Tratamento: predominantemente preventivo. Uma vez instalada, a
lesão é irreversível. Nos casos mais severos, o uso de AASI pode ser
necessário.

Figura 3.22 - Audiometria em perda de audição induzida pelo ruído

Nota: padrão de “gota acústica”, com perda mais intensa em 3 a 4 kHz.

3.5.1.3 Trauma acústico

1. Definição: o trauma acústico deve inicialmente ser distinguido da


PAIR, que é de instalação lenta e insidiosa. O trauma acústico é uma
perda auditiva de instalação súbita decorrente de uma única exposição
ao ruído muito intenso e de curta duração, como, por exemplo,
explosivos. Essa exposição pode resultar em imediata, severa e
permanente perda auditiva;
2. Fisiopatologia: a energia acústica explosiva pode lesar a MT,
causando perfuração. A transmissão da energia sonora explosiva para
a orelha interna pode lacerar a membrana basal ou a membrana de
Reissner – cóclea –, resultando em perda auditiva imediata;
3. Quadro clínico: hipoacusia, zumbido e otalgia após exposição a
som explosivo; vertigem pode também ocorrer. A otoscopia pode ser
normal ou pode haver ruptura da MT e hemorragia da orelha média;
4. Diagnóstico: com base no quadro clínico. A audiometria mostra
disacusia neurossensorial ou mista de graus variáveis, uni ou bilateral;
5. Tratamento: o tratamento inicial pode ser feito com uso de
corticosteroides, porém sua eficácia é controversa. A reabilitação pode
ser feita por meio da adaptação de AASI.

3.5.1.4 Hipoacusia por ototoxicidade

1. Definição: lesão de células da orelha interna por ação de


medicamentos ou outros agentes químicos, produzindo perdas
auditivas do tipo neurossensorial. Podem ser citados os antibióticos
aminoglicosídeos (gentamicina, amicacina, estreptomicina,
canamicina), diuréticos (furosemida), anti-inflamatórios, geralmente
reversível, além de drogas antineoplásicas, como cisplatina e
carboplatina;
2. Quadro clínico: inicia-se com zumbido uni ou bilateral, e a
audiometria se caracteriza por perda auditiva do tipo neurossensorial
uni ou bilateral simétrico. Poderá haver queixas de vertigem;
3. Diagnóstico: o critério é a piora nos limiares auditivos da
audiometria tonal superior a 20 dB em 1 ou mais frequências em
indivíduos na vigência do tratamento com alguma droga; no entanto,
raramente temos uma audiometria prévia e recente do indivíduo;
4. Tratamento: muitos apresentam regressão do quadro após a
interrupção do uso da medicação. Por isso, a prevenção ainda é a
melhor estratégia e se baseia no uso racional e monitorizado dessas
drogas, com audiometria de controle e exames adicionais, como as
otoemissões acústicas.

#IMPORTANTE
Paciente em uso de medicação com potencial
ototoxicidade deve ser avaliado continuamente.
Caso apresente zumbido, suspende-se a
medicação imediatamente.

3.5.1.5 Surdez súbita


1. Definição: perda auditiva de início repentino, ou piora súbita de
hipoacusia preexistente. A fisiopatologia ainda é muito controversa, e a
maioria dos casos permanece como de causa idiopática. São 3 os
mecanismos sugeridos: infecção viral do ouvido interno, patologias
autoimunes afetando a orelha interna e alterações da
microvascularização dessa região;
2. Quadro clínico: normalmente é unilateral e o paciente queixa-se de
hipoacusia, plenitude auricular e zumbidos de início súbito ou com
rápida evolução, em horas. Queixas de vertigem algumas vezes
podem estar presentes;
3. Diagnóstico: otoscopia geralmente é normal. A confirmação é feita
por meio de audiometria tonal e vocal, que mostra perda auditiva do
tipo neurossensorial unilateral de intensidade variável (Figura 3.23). A
investigação deve ser complementada por exames de imagem, como a
RNM, principalmente para descartar tumores do VIII par;
4. Tratamento: os corticosteroides devem ser administrados por via
oral, injetados na orelha média via transtimpânica ou combinação das
2 terapias anteriores. Outras terapias, como aciclovir, hemorreológicos
e antiagregantes plaquetários não têm forte evidência na literatura e
seu uso ainda é controverso. Todos esses tratamentos, no entanto,
parecem ter eficácia limitada. A maioria apresenta algum grau de
reversão da perda auditiva, entretanto, em alguns pode ser irreversível.

Figura 3.23 - Audiometria em surdez súbita com curva normal acima – orelha direita – e
sinais de disacusia neurossensorial severa abaixo – orelha esquerda
Quadro 3.8 - Patologias da orelha interna
3.6 SURDEZ NA INFÂNCIA
1. Definição: a surdez infantil bilateral permanente é definida como
perda auditiva bilateral caracterizada por limiares auditivos superiores
a 40 dB no melhor ouvido, considerando as frequências de 500, 1000,
2000 e 4000 Hz;
2. Quadro clínico: varia de acordo com a idade da criança e momento
de início da doença. Em linhas gerais, crianças com déficit auditivo
apresentam retardo no desenvolvimento da fala, dificuldade de
aprendizado e concentração, ausência de resposta a comandos
verbais e desenvolvimento de hábitos e sinais que facilitem a
comunicação com os pais e familiares. Nos casos de surdez leve, a
palavra é percebida pela criança, apesar de alguns fonemas lhe
escaparem. Na surdez moderada, a criança ouve sons intensos, porém
não os discrimina; na surdez severa ou profunda, a palavra não é
entendida, o que praticamente impede a comunicação verbal;
3. Etiologias:
a) Hipoacusias neurossensoriais de causa genética:
Congênitas;
Displasias:
Michel;
Mondini;
Scheibe.
Surdez neurossensorial genética congênita não associada a
malformações;
Surdez neurossensorial genética congênita associada a
malformações:
Dismorfias cranianas – Crouzon;
Dismorfias das extremidades – Wildewanck;
Anomalias de pigmentação – Waardenburg;
Doenças oftalmológicas – Usher-Hallgren;
Doenças cardíacas – Jervell e Lange-Nielsen.
Patologia tireóidea – Pendred;
Aberrações cromossômicas – trissomias e monossomias;
Síndrome do aqueduto vestibular largo;
Fístula perilinfática congênita;
Pós-natais;
Surdez neurossensorial genética pós-natal não associada a
malformações;
Surdez neurossensorial genética pós-natal associada a
malformações:
Doenças metabólicas – mucopolissacaridoses;
Malformações oftalmológicas – Cockayne;
Doenças renais (Alport, Fanconi);
Malformações esqueléticas (Klippel-Feil);
Doenças neurológicas (neurofibromatose).
b) Hipoacusias neurossensoriais de causa não genética:
Pré-natais:
Ototóxicos durante a gravidez:
Aminoglicosídeos;
Diuréticos;
Talidomida;
Álcool.
Infecções congênitas: sífilis, toxoplasmose, rubéola,
citomegalovírus e herpes (STORCH);
Outras causas:
Hemorragias do primeiro trimestre;
Deficiências vitamínicas;
Hormonoterapia;
Irradiação pélvica.
Perinatais:
Icterícia neonatal grave;
Baixo peso, menos de 1.500 g, e prematuridade;
Asfixia perinatal;
Traumatismo do parto;
Pós-natais:
Infecções:
Labirintite;
Meningite;
Parotidite;
Sarampo;
Traumatismos cranianos;
Traumatismos sonoros;
Fármacos ototóxicos;
Neoplasias – neurinoma de acústico, leucemias;
Doenças metabólicas como hipotireoidismo e diabetes;
Doenças autoimunes;
Surdez súbita idiopática.
4. Diagnóstico: além da Triagem Auditiva Neonatal Universal (TANU),
discutida adiante, os métodos diagnósticos se baseiam na realização
de BERA com pesquisa de limiar auditivo, uma vez que esse teste
independe da resposta voluntária do indivíduo. Em crianças a partir
dos 5 anos de idade, pode-se tentar executar a audiometria tonal
condicionada;
5. Tratamento: além de tratar a causa base, deve-se testar AASI e
fonoterapia, além de educação especial.

3.6.1 Triagem auditiva neonatal universal


Hoje é consensual que a simples observação médica e a suspeita
parental não são suficientes para a detecção de surdez no primeiro
ano de vida. Dessa forma, deve-se aplicar a triagem auditiva a todos
os recém-nascidos de até 3 meses, e atenção especial deve ser dada a
indivíduos portadores de fatores de risco para surdez, conforme
explicitado a seguir.
São fatores de risco de surdez infantil:
a) Preocupação dos pais com relação ao desenvolvimento de
linguagem;
b) História familiar de surdez com início na infância;
c) Permanência na UTI por mais de 5 dias;
d) Qualquer um dos fatores independentemente do tempo em
UTI:
Ventilação assistida;
Drogas ototóxicas;
Apgar Neonatal de 0 a 4 no primeiro minuto, ou 0 a 6 no
quinto minuto;
Infecções congênitas – STORCH;
Malformações anatômicas de cabeça e pescoço;
Baixo peso – menos que 1.500 g;
Hiperbilirrubinemia;.
e) Síndromes genéticas que usualmente expressam deficiência
auditiva:
Waardenburg;
Alport;
Pendred;
f) Uso materno de drogas ou ototóxicos;
g) Distúrbios neurodegenerativos
Ataxia de Friedreich;
Síndrome de Charcot-Marie-Tooth;
h) Infecções bacterianas ou virais pós-natais:
Citomegalovírus;
Herpes;
Sarampo;
Varicela;
Meningite;
i) Trauma cranioencefálico no parto;
j) Quimioterapia.

3.6.2 Métodos de triagem auditiva neonatal


universal
3.6.2.1 Emissões otoacústicas evocadas

São os testes universais de triagem. Consistem em registros de


energia sonora gerada pelas células da cóclea em resposta a sons
emitidos no CAE do recém-nascido. Não há resposta quando existe
alguma anormalidade funcional no ouvido interno.
3.6.2.2 Potencial evocado auditivo do tronco encefálico
Avalia a integridade neural das vias auditivas até o tronco cerebral e
apresenta sensibilidade de 98% e especificidade de 96%. Em geral é
realizado a partir de 3 a 6 meses.
3.6.2.3 Protocolo de TANU

Em linhas gerais, o protocolo do Ministério da Saúde é seguido da


seguinte maneira:
1. Indivíduos sem fatores de risco: aplicam-se otoemissões
evocadas auditivas;
a) Passou: sem necessidade de reavaliação;
b) Não passou: retestar em 30 a 60 dias;
Passou: sem necessidade de reavaliação;
Não passou: realizar BERA.
2. Indivíduos com fatores de risco ou com 2 otoemissões
evocadas auditivas negativas: realizar BERA;
a) Passou: observação ou monitorização;
b) Não passou: repetir BERA em 30 dias;
Passou: monitorização;
Não passou: avaliação otorrinolaringológica – provável
surdez.

Os detalhes desse protocolo estão na Figura 3.24.


Figura 3.24 - Triagem auditiva neonatal

Legenda: Potenciais Evocados Auditivos de Tronco Encefálico (PEATE); Indicador de


Risco para Deficiência Auditiva (IRDA); Aparelho de Amplificação Sonora Individual (AASI);
emissões otoacústicas evocadas (EOAE); otorrinolaringológico (ORL).

Fonte: Ministério da Saúde.

3.7 VESTIBULOPATIAS PERIFÉRICAS


Inicialmente, deve-se diferenciar as vestibulopatias periféricas das
de origem central. As primeiras originam-se por acometimento do
órgão vestibular periférico, o labirinto, e as últimas têm sua origem
em doenças que acometem as vias neurológicas centrais
responsáveis pelo equilíbrio. As diferenças clínicas entre as 2
patologias, com relação às características do nistagmo, estão
apresentadas no Quadro 3.9.
Quadro 3.9 - Características do nistagmo nas vestibulopatias periférica e central

3.7.1 Hidropisia endolinfática – doença de Ménière


1. Definição: acúmulo de endolinfa no labirinto membranoso;
2. Etiologia: pode ser idiopática ou associada a distúrbios metabólicos
como diabetes mellitus, distúrbios de tireoide e dislipidemia,
autoimunes ou infecciosos, como sífilis;
3. Quadro clínico: crises de vertigem, de curta a média duração –
cerca de 10 a 60 minutos – com ou sem fator desencadeante evidente,
associadas a hipoacusia, plenitude e zumbido uni ou bilateral. Outros
sintomas autonômicos, como cefaleia, mal-estar, náuseas e vômitos,
podem acompanhar o quadro.

3.7.1.1 Diagnóstico

Não existe nenhum teste que estabeleça o diagnóstico de doença de


Ménière, no entanto a eletrococleografia é o exame mais comumente
utilizado.
1. Eletronistagmografia-otoneurológico: não existe quadro
patognomônico. A prova calórica mostra hipo, hiper ou arreflexia em
cerca de 85% dos casos;
2. Eletrococleografia: auxilia no diagnóstico de hidropisia
endolinfática e apresenta sensibilidade de 70%. Se normal, não exclui
o diagnóstico. A relação Potencial de Somação (PS)-Potencial de Ação
(PA) está aumentada em 62% nos pacientes com Ménière.

O tratamento deve visar ao controle das patologias associadas


quando existentes. Drogas sedativas labirínticas – betaistina,
dimenidrinato – e diuréticas, hidroclorotiazida, são as mais
consensualmente usadas. Em casos refratários ao tratamento
clínico, pode-se considerar abordagem cirúrgica, como
descompressão de saco endolinfático, neurectomia do NC VIII.
Figura 3.25 - Síndrome de Ménière – acúmulo de endolinfa e consequente dilatação na
região do labirinto membranoso
Fonte: Ilustração Claudio Van Erven Ripinskas.

3.7.2 Vertigem posicional paroxística benigna


1. Definição: distúrbio labiríntico relacionado a migração das otocônias
do vestíbulo em direção aos canais semicirculares, principalmente ao
posterior;
2. Quadro clínico: crises vertiginosas intensas que duram segundos
relacionadas a movimentos rotacionais da cabeça, como olhar para
cima ou virar-se rapidamente. Durante essa movimentação, as
otocônias alteram o fluxo da endolinfa, causando desequilíbrio;
3. Diagnóstico: é clínico, sendo fundamental a manobra de Dix-
Hallpike (Figura 3.26). A audiometria tem pouco valor diagnóstico, e o
exame otoneurológico pode evidenciar hiper-reatividade unilateral;
4. Tratamento: é feito com sintomáticos nas crises – dimenidrinato,
meclizina –, porém a resolução acontece com as manobras de
reposicionamento Epley e Semont (Figura 3.27).

Figura 3.26 - Manobra de Dix-Hallpike


Figura 3.27 - Manobras de reposicionamento

Legenda: (A) Epley; (B) Semont.

#IMPORTANTE
Na vertigem posicional paroxística benigna, o
principal canal acometido é o posterior. Como
manobra diagnóstica, cita-se a de Dix-Hallpike,
e de reposicionamento, as de Epley e Semont.
3.7.3 Neuronite vestibular
1. Definição: processo inflamatório agudo do ramo vestibular do NC
VIII, de etiologia indefinida;
2. Quadro clínico: vertigem dramática, súbita, com sintomas
neurovegetativos, com duração de dias e normalmente sem sintomas
auditivos. IVAS pode preceder ou acompanhar a doença;
3. Diagnóstico: baseia-se no quadro clínico. A RNM pode evidenciar
hipersinal do VIII par craniano, secundário a processo inflamatório
agudo. A audiometria pode estar normal ou mostrar disacusia
neurossensorial de graus variáveis;
4. Tratamento: feito com sintomáticos – depressores labirínticos, como
dimenidrinato – e corticoterapia. A melhora é gradual e geralmente
definitiva.

Quadro 3.10 - Vestibulopatias periféricas


3.8 TUMORES DE OSSO TEMPORAL E
ÂNGULO PONTOCEREBELAR
3.8.1 Schwannoma vestibular
O schwannoma vestibular é o tumor benigno do VIII par craniano, o
nervo vestibulococlear, sendo o mais comum com origem na fossa
craniana posterior. É mais comum em mulheres.
3.8.1.1 Fisiopatologia

Tem origem na divisão vestibular do VIII par craniano e deriva


histologicamente das células de Schwann. O crescimento do tumor é
lento, geralmente de 0,25 a 0,4 mm/ano.
3.8.1.2 Quadro clínico

Os pacientes podem ser assintomáticos ou apresentar queixa de


hipoacusia unilateral, diminuição na compreensão da fala, zumbido
ou mesmo surdez súbita.
O acometimento do nervo facial ocorre em 10 a 30% dos casos.
3.8.1.3 Exame físico geral

O exame físico pode estar normal ou demonstrar sinais de


comprometimento dos nervos cranianos.
3.8.1.4 Testes audiométricos

Qualquer padrão audiométrico pode ser encontrado, incluindo o


normal, ou perda tipo neurossensorial em altas frequências. O
reflexo estapediano está ausente em 88% dos pacientes. Há baixo
índice de discriminação (< 60%). O BERA mostra aumento de
latência de onda I, com intervalo interpicos geralmente preservado.
3.8.1.5 Exames de imagem

A RNM é o exame de escolha na suspeita de patologia do ângulo


pontocerebelar. Quando realizado com contraste – gadolínio –,
apresenta especificidade de praticamente 100%, mesmo para
pequenos tumores – 2 mm. Pela TC, podem-se observar massas iso
ou hipodensas dentro do Conduto Auditivo Interno (CAI).
Figura 3.28 - Ressonância nuclear magnética em corte axial e sagital em T1 evidenciando
tumoração em ângulo pontocerebelar à esquerda, com captação homogênea de contraste,
sugerindo neurinoma
3.8.1.6 Conduta
Tumores pequenos com poucos sintomas: pode-se adotar conduta
expectante, com repetição periódica da RNM.
A decisão da abordagem cirúrgica deve considerar a manutenção da
vida, evitando grandes sequelas neurológicas.
O consenso para escolha de tratamento clínico ou cirúrgico é:
1. A ressecção cirúrgica é a melhor opção àqueles com menos de 65
anos e saudáveis;
2. Em pacientes entre 65 e 75 anos, é prudente avaliar estado geral,
exame neurológico, tamanho do tumor, localização, taxa de
crescimento, status auditivo e história familiar. Nesses casos, costuma-
se considerar a radioterapia bom método para controlar o crescimento
do tumor;
3. Em pacientes com mais do que 75 anos, dá-se preferência à
radioterapia;
4. Pacientes idosos, sem condições clínicas, com tumores pequenos e
sem sintomas neurológicos podem ser seguidos clinicamente, com
RNM a cada 6 meses. Caso seja observado crescimento evidente,
indica-se a radioterapia; caso contrário, mantém-se o seguimento com
RNM.

3.8.2 Meningiomas
1. Definição: são o segundo tipo mais comum de tumor com origem
no ângulo pontocerebelar. Originam-se de células da dura-máter e da
subaracnoide;
2. Quadro clínico: quando surgem do CAI, podem ter sintomas
idênticos aos de um tumor do VIII par;
3. Diagnóstico: os achados audiométricos são semelhantes aos dos
schwannomas. A diferenciação entre schwannoma e meningioma é
feito baseado na RNM e na TC. Em paciente com disacusia
neurossensorial unilateral ou assimétrica, considera-se a possibilidade
de tumores retrococleares. O teste de triagem inicial é o BERA;
4. Tratamento: excisão cirúrgica é o tratamento de escolha.

Figura 3.29 - Ressonância nuclear magnética de encéfalo evidenciando meningioma de


ângulo pontocerebelar
Nota: observar como a captação de contraste é mais intensa.

3.9 TUMORES GLÔMICOS DE OSSO


TEMPORAL
3.9.1 Anatomia
Podem ser encontrados ao longo do trajeto da veia jugular interna.
Corpos glômicos – paragânglios – são encontrados no canalículo
timpânico.
3.9.2 Patologia
Apresentam considerável variação de comportamento. Os mais
frequentes exibem crescimento lento, mas podem, no entanto,
causar erosão óssea.
3.9.3 Quadro clínico
O zumbido é pulsátil. A audição pode estar normal, ou pode haver
surdez de condução quando o tumor invade a orelha média. Se
houver invasão de cóclea, NC VIII ou SNC, ocorrerá disacusia
neurossensorial. Paresia ou paralisia facial são possíveis em estágios
mais avançados.
3.9.4 Exame físico
A otoscopia é largamente variável e depende da localização e do
estadiamento do tumor. Observam-se hipervascularização da MT;
coloração vermelho-escura ou azul, sugerindo massa
retrotimpânica intacta; ou visualização da massa quando o tumor já
perfurou a membrana ou já se exterioriza pelo CAE.
3.9.5 Exames de imagem
1. TC: nela se observa massa no ouvido médio ou na região do bulbo
da veia jugular com captação de contraste. Também mostra pequenas
áreas de invasão intradural;
2. Arteriografia e venografia: são os principais exames no
diagnóstico, pois visualizam massa vascularizada no espaço
acometido;
3. RNM: tem-se boa avaliação do tumor com esse método, já que as
densidades ósseas não são demonstráveis. Esse exame mostra
imagem “em sal e pimenta” tanto em T1 quanto em T2;
4. Angiorressonância: método não invasivo bastante adequado para
lesões hipervascularizadas. O realce pós-contraste no glômus é
rápido, intenso e homogêneo.

3.9.6 Tratamento
É realizado com radioterapia, embolização arterial e cirurgia.
Quadro 3.11 - Tumores de osso temporal

3.10 PARALISIA FACIAL PERIFÉRICA


A Paralisia Facial Periférica (PFP) compreende a alteração da função
do VII par craniano, o nervo facial, a partir de seu núcleo motor na
ponte. Há comprometimento das funções da mímica facial, com
sérios prejuízos funcionais e psicossociais ao paciente.
Primeiramente, devemos diferenciar se a origem da paralisia é
central – córtex motor – ou periférica – a partir do núcleo do NC VII.
Na paralisia de origem central, o terço superior da face não está
acometido, uma vez que algumas fibras neurais de origem
contralateral ao lado acometido, que inervam a hemiface ipsilateral,
não decussam. Assim, ambos os lados do terço superior da face
recebem estímulos provenientes dos 2 hemisférios cerebrais. Na
paralisia de origem periférica, após o núcleo do VII par, a paresia de
hemiface é completa, ou seja, o paciente tem movimentos
deficientes também no terço superior da face.
Figura 3.30 - Paciente com paralisia facial central
Fonte: Ilustração Claudio Van Erven Ripinskas.

3.10.1 Causas
A principal causa de PFP é a paralisia facial de Bell, cuja etiologia não
é totalmente elucidada, porém acredita-se que alguns vírus, como o
herpes I e o Epstein-Barr, estabeleçam uma infecção persistente –
ou latente – sem sintomas. Com isso, ocorre uma neurite
inflamatória que leva ao edema do nervo facial, levando à inibição,
lesão ou morte do nervo.
1. Quadro clínico: o paciente queixa-se da PFP de início súbito. Ao
exame, observa-se desvio da musculatura facial para o lado não
acometido; esse desvio se acentua ao sorrir e assoviar. O fechamento
palpebral pode estar incompleto ou ausente no lado afetado, o que
permite a observação de desvio do globo ocular do lado paralisado ao
piscar os olhos. É o chamado sinal de Bell;
2. Diagnóstico: a paralisia de Bell é unilateral e idiopática, ou seja, um
diagnóstico de exclusão. Todas as possíveis causas devem ser
descartadas, como trauma do osso temporal e neoplasias. Exames de
imagem como RNM devem ser feitos nos casos de PFP quando se
suspeita de patologias não Bell – tumores. A audiometria pode mostrar
limiares auditivos preservados. No entanto, o reflexo estapediano pode
estar abolido;
3. Tratamento: o uso de corticosteroides, como a prednisolona, em
altas dosagens, é notadamente o principal medicamento efetivo. A
associação a aciclovir oral também pode ser feita, porém os estudos
não são consensuais quanto ao seu emprego. A maioria tem bom
prognóstico, com retorno à normalidade em algumas semanas ou
meses após o surto.

Outras causas de paralisia facial são as traumáticas, como TCEs ou


cirurgias de ouvido, e tumorais, quando há envolvimento do nervo.
Em traumas, utilizam-se corticosteroides em altas doses e pode-se
considerar a descompressão cirúrgica. Já em etiologias tumorais,
deve-se tratar a causa-base.
Quadro 3.12 - Características da paralisia facial
Paciente do sexo masculino,
18 meses. Mãe refere que
há 2 dias apresenta febre
de 38 C, diminuição da
aceitação alimentar e
prostração leve. Ao exame
físico, otoscopia normal à
direita e abaulamento com
hiperemia à esquerda.
Devemos dar antibiótico a
esse paciente?
Como vimos no capítulo, quadros de OMA unilateral, sem
aparente gravidade (febre baixa, pouca prostração),
segundo o protocolo de tratamento, não preenchem
critério para prescrição de antibiótico no momento do
atendimento. No entanto, deve-se observar sinais de alerta
e retornar em 48 horas para reavaliação. Caso o paciente
apresente piora, será considerada a antibioticoterapia.
O que é mononucleose e a
síndrome mono-like? Quais
são seus agentes
patológicos?

4.1 LARINGITES
As laringites podem ser classificadas em agudas, com duração de até
15 dias, ou crônicas, com duração de mais de 4 semanas. Os adultos
geralmente têm, como queixa, a disfonia; nas crianças, a
sintomatologia normalmente é estridor e dispneia, pois as
dimensões do órgão são menores.
4.1.1 Infecciosas
As infecções agudas ocorrem com febre e comprometimento das vias
aéreas, sendo mais prevalentes na infância. As crônicas geralmente
têm a dor e a rouquidão como sintomas predominantes e são mais
frequentes em adultos.
4.1.1.1 Agudas

a) Epiglotite ou supraglotite pediátrica

1. Definição: inflamação aguda grave das estruturas supraglóticas que


pode levar a obstrução respiratória e, portanto, ser fatal. Embora a
epiglote represente a principal estrutura acometida, também pode
haver edema nas pregas ariepiglóticas, falsas pregas vocais e
aritenoide;
2. Etiologia: Haemophilus influenzae B – epiglotite –, com mais
frequência na faixa etária de 2 a 4 anos, nos meses de inverno e
primavera. As crianças são protegidas contra a infecção até os 3
meses por anticorpos maternos. Nos últimos anos, essa afecção se
tornou rara, dada a ampla cobertura vacinal;
3. Quadro clínico: a evolução é rápida, com instalação do quadro
geralmente em 2 a 6 horas. O paciente apresenta odinofagia, disfonia
intensa, taquidispneia, estridores, febre alta. Os sinais típicos são a
hiperextensão cervical e a projeção torácica anterior – posição adotada
para manter a perviedade da via aérea. Pode ocorrer obstrução aérea
pelo edema da epiglote e da prega ariepiglótica e pela produção
excessiva de secreção espessa, evoluindo para parada respiratória
súbita;
4. Diagnóstico: é clínico, com base na história e no exame físico. A
radiografia lateral cervical revela espessamento de tecidos moles –
sinal “do polegar” significa epiglote edemaciada –, porém os exames
subsidiários não devem retardar o início da terapêutica;
5. Tratamento: a suspeita de epiglotite deve ser tratada como
emergência. O paciente deve ser monitorado, iniciadas oxigenoterapia,
inalação com adrenalina e corticoterapia intravenosa. Nos casos em
que o paciente apresente sinais de insuficiência respiratória, o médico
mais experiente deve realizar a laringoscopia e a intubação orotraqueal
para garantir a via aérea. Após a estabilização, deve-se realizar a
antibioticoterapia com ceftriaxona. Vale lembrar que a vacina contra
Haemophilus influenzae tipo B – Hib – pode prevenir epiglotites
causadas por essa bactéria.

Figura 4.1 - Supraglotite:


Legenda: (A) laringe de aspecto preservado; (B) paciente com quadro de supraglotite,
além de edema mucoso intenso e comprometimento do espaço aéreo.

Fonte: site Voice Doctor.

Na epiglotite, quando há suspeita diagnóstica,


deve-se iniciar inalação com adrenalina e
intubação precoce pelo médico mais
experiente.

b) Supraglotite adulta

O quadro em adultos é menos intenso, não levando ao


comprometimento das vias aéreas. O diagnóstico é feito por
radiografia cervical e visualização por meio da
nasofibrolaringoscopia.
A faixa etária atingida está entre os 18 e os 40 anos, sem prevalência
sazonal. O agente causador nos adultos não é bem definido. Somente
cerca de 20% a 30% apresentam hemocultura positiva, a maioria
para Haemophilus influenzae B, podendo também ser viral, fúngico,
C. albicans ou por trauma.
O tratamento recomendado consiste em observação, hemocultura,
antibioticoterapia com cobertura para H. influenzae – amoxicilina e
clavulanato –, corticoterapia e manutenção da via aérea nos casos
mais intensos.
c) Laringotraqueíte aguda

1. Definição: a laringotraqueíte aguda, ou crupe, pode ser definida


como uma infecção aguda das vias aéreas altas. O fator crucial da
doença é o edema da área subglótica. Em crianças menores de 3
anos, esse é o ponto de maior estreitamento das vias aéreas altas;
2. Quadro clínico: ocorre mais em crianças de 1 a 3 anos e a maioria
dos casos se resolve em até 48 horas. A manifestação clínica inicial
costuma ser a infecção das vias aéreas superiores, que evolui
subitamente para disfonia e tosse não produtiva tipo “latido de
cachorro”, com piora à noite, febre e estridor;
3. Etiologia: vírus parainfluenza 1 e 2 e influenza tipo A, Mycoplasma
pneumoniae;
4. Diagnóstico: habitualmente é clínico, porém a radiografia cervical
pode mostrar o sinal clássico “da torre de igreja”, causado pelo
estreitamento subglótico;
5. Diagnósticos diferenciais: epiglotite, laringite bacteriana,
aspiração de corpo estranho;
6. Tratamento: consiste em umidificação das vias aéreas, hidratação
para facilitar a expectoração de secreção e repouso vocal. Se a
dispneia for severa, pode-se utilizar adrenalina inalatória. O
corticosteroide – dexametasona – parenteral deve ser administrado
para a regressão do edema e, principalmente, para evitar recidivas.
Também devem-se utilizar corticoides inalatórios, como a budesonida.
Antibióticos são indicados apenas no caso de infecções bacterianas
secundárias.

Figura 4.2 - Estreitamento subglótico da laringe com sinal “da torre de igreja”
Fonte: Learning Radiology

d) Traqueíte bacteriana

1. Definição: trata-se de uma infecção pediátrica que, em seus


estágios iniciais, pode ser difícil diferenciar do crupe, e cujo principal
sintoma inicial é o estridor. Tem taxas de mortalidade de 20%;
2. Quadro clínico: febre alta, tosse, estridor e disfonia, com
antecedente de crupe; sinais de obstrução das vias aéreas, síndrome
do desconforto respiratório agudo e choque séptico. Aproximadamente
80% dos pacientes com traqueíte bacteriana requerem intubação. Os
achados clínicos são ulceração e formação de pseudomembrana na
traqueia, exsudato mucopurulento e descamação da mucosa, que
obstruem a traqueia;
3. Etiologia: os principais agentes são Staphylococcus aureus e
Streptococcus alfa-hemolítico, Haemophilus influenzae, Streptococcus
pyogenes e Moraxella catarrhalis;
4. Diagnóstico: baseia-se na presença de secreção espessa,
principalmente em crianças que não apresentam melhora após
tratamento medicamentoso para crupe, com febre alta e leucocitose.
Nesses casos, sugere-se broncoscopia para observar a presença de
secreção, com coleta de material para cultura;
5. Diagnósticos diferenciais: crupe, laringotraqueobronquite viral,
epiglotite, bronquiolite, angioedema e difteria em pacientes não
vacinados. A suspeita de traqueíte bacteriana deve aumentar quando
um paciente com desconforto não responde à epinefrina inalatória e/ou
aos corticosteroides sistêmicos, que tipicamente melhoram a
laringotraqueobronquite do crupe e do vírus;
6. Tratamento: baseia-se na aspiração das secreções,
antibioticoterapia e eventual intubação ou traqueostomia.

e) Coqueluche

1. Definição: infecção causada pela Bordetella pertussis, é mais


frequente em crianças com menos de 6 meses, pois não ocorre
passagem intraútero de anticorpos maternos, e a criança se torna
imune somente após vacinação, e em adultos, pois a imunidade por
meio da vacinação dura cerca de 10 a 15 anos;
2. Quadro clínico: geralmente se divide em 3 estágios:
a) Primeiro estágio – catarral: duração de 7 a 10 dias e
sintomas inespecíficos de infecção das vias aéreas, com coriza,
febre baixa e prostração;
b) Segundo estágio – paroxismo: duração de 1 a 6 semanas,
caracterizado por quadros paroxísticos de tosse prolongada
seguida por uma inspiração forçada e prolongada com as pregas
vocais aduzidas, gerando um ruído característico, o “guincho”;
c) Terceiro estágio – convalescência: duração de 7 a 10 dias,
caracterizado por convalescência gradual.
3. Diagnóstico: basicamente clínico, podendo ser avaliado
laboratorialmente por meio de cultura de secreção de via aérea, PCR
ou sorologias, que geralmente são positivas somente após a terceira
semana do quadro;
4. Tratamento: antibioticoterapia com eritromicina, claritromicina ou
azitromicina é recomendada nas primeiras semanas do quadro.

Quadro 4.1 - Laringites infecciosas agudas


4.1.1.2 Crônicas

a) Tuberculose
1. Definição: trata-se de uma das principais causas de doença
granulomatosa laríngea. Pode ocorrer por via hematogênica ou
linfática, originando-se de sítios primários distantes;
2. Quadro clínico: o principal sintoma é a rouquidão persistente,
seguida por tosse e dor de garganta. Com a progressão, afeta
caracteristicamente bandas ventriculares e supraglote, causando
disfagia e odinofagia;
3. Diagnóstico topográfico: seu sinal mais sugestivo é a cordite
unilateral (Figura 4.3);

Figura 4.3 - Cordite unilateral em prega vocal esquerda causada por tuberculose laríngea,
indicada por seta

4. Diagnóstico: o diagnóstico de tuberculose extrapulmonar, segundo


o Ministério da Saúde, deve se basear em evidências clínicas, achados
laboratoriais, inclusive histopatológicos compatíveis, ou paciente com
pelo menos 1 cultura positiva de material proveniente de localização
extrapulmonar;
5. Tratamento: feito com o esquema RIPE – Rifampicina, Isoniazida,
Pirazinamida e Etambutol –, por 2 meses, seguido de mais 4 meses
com rifampicina e isoniazida. Após o tratamento, pode haver
progressão para fibrose e estenose laríngea.
b) Sífilis

1. Definição: acometimento laríngeo por sífilis mucosa – secundária;


2. Quadro clínico: disfonia persistente, tosse e sensação de incômodo
faríngeo;
3. Diagnóstico: a laringoscopia pode evidenciar lesões nodulares ou
ulceradas difusas, que podem estar associadas a pericondrite, fibrose
e deformidades cicatriciais, levando à obstrução das vias aéreas. A
identificação do treponema confirma o diagnóstico realizado pela
microscopia de fundo escuro;
4. Tratamento: a escolha é a penicilina benzatina.

c) Leishmaniose

1. Definição: lesão mucosa crônica na laringe causada pela


Leishmania braziliensis. A forma mucocutânea é a mais comum,
atingindo a mucosa das vias aéreas superiores;
2. Quadro clínico: as lesões são ulcerogranulomatosas, mais
comumente supraglóticas, podendo acometer também glote e
subglote, levando à obstrução das vias aéreas. Os principais sintomas
incluem disfonia, tosse, disfagia e dispneia e geralmente apresenta
porta de entrada cutânea em atividade ou cicatricial – úlcera de Bauru;
3. Diagnóstico: baseia-se nos achados clínicos – história de lesões
cutânea e nasal – associados à reação de Montenegro, no achado do
agente via exame histopatológico e sorologia;
4. Tratamento: antimoniais pentavalentes, Glucantime®. A anfotericina
B surge como segunda opção de tratamento.

Quadro 4.2 - Laringites crônicas


4.1.2 Não infecciosas
4.1.2.1 Refluxo laringofaríngeo

Classicamente, a laringite por refluxo laringofaríngeo (RLF) foi


observada como tendo a doença do refluxo gastroesofágico (DRGE)
como sua principal etiopatogenia. Estudos recentes, no entanto,
questionam essa relação causal, uma vez que a pHmetria de
indivíduos portadores de RLF é muito semelhante à dos controles e
nem sempre o paciente com refluxo laringofaríngeo apresenta
sintomas ou sinais de doença do refluxo gastroesofágico.
O quadro clínico caracteriza-se por queixas de disfonia de grau
variável, tosse seca, pigarro, sensação de globus faríngeo ou de
secreção presa na hipofaringe. Abuso vocal e tabagismo podem
intensificar essas queixas.
À endoscopia laríngea, pode-se observar edema das estruturas
supraglóticas e glóticas, com hiperemia delas. Esses achados
normalmente se localizam na porção posterior da laringe
(aritenoides e região interaritenóidea).
O tratamento utiliza inibidores da bomba de prótons (omeprazol,
pantoprazol, esomeprazol) por 60 a 90 dias. Medidas
comportamentais e dietéticas são fundamentais.
4.2 PATOLOGIAS NÃO
INFLAMATÓRIAS DA LARINGE
4.2.1 Laringomalácia
1. Definição: é a malformação mais comum da laringe e a principal
causa de estridor respiratório na infância. As cartilagens laríngeas
encontram-se amolecidas devido à imaturidade no desenvolvimento
histológico dessas estruturas e seus tecidos de sustentação;
2. Quadro clínico: há um colapso das estruturas supraglóticas –
epiglote, pregas ariepiglóticas e a mucosa aritenóidea – durante a
inspiração, causando estridor inspiratório. Em geral, não está presente
logo após o nascimento, e os sintomas iniciam-se após 4 a 6 semanas
de vida e tendem a se agravar nos primeiros meses;
3. Diagnóstico: pode ser confirmado pelo exame endoscópico, no
qual se observam a epiglote “em ômega” ou tubular (Figura 4.4),
pregas ariepiglóticas curtas e mucosa supraglótica redundante, que,
durante a inspiração, colabam no introito laríngeo;
4. Tratamento: essa hipotonia laríngea é uma disfunção fisiológica que
se resolve com o crescimento, em geral em torno do segundo ano de
vida. Dessa forma, a doença normalmente é autolimitada, com
resolução gradual ao redor dos 18 aos 24 meses de vida. Geralmente,
associa-se a DRGE, e o seu tratamento é fundamental para o controle
do quadro respiratório dessas crianças.

Figura 4.4 - Epiglote “em ômega”, típica da laringomalácia


Fonte: DoctorMichael, 2008.

Figura 4.5 - Representação esquemática do colabamento laríngeo durante a inspiração


Nota: observar o colabamento laríngeo devido à projeção das aritenoides sobre a via
aérea.

Fonte: Ilustração Claudio Van Erven Ripinskas.

4.2.2 Estenoses laríngeas


As estenoses laríngeas podem ocorrer na região da supraglote, da
glote ou da subglote. Estenose subglótica é a situação que mais
apresenta necessidade de algum tipo de intervenção, seguida pela
estenose glótica. Estenoses supraglóticas são raras em crianças e
decorrem, na maioria dos casos, de lesões térmicas ou químicas.
4.2.2.1 Estenose laríngea congênita

a) Estenose subglótica

1. Definição: em neonatos, uma luz traqueal de diâmetro menor do


que 4 mm na região da cartilagem cricoide. As estenoses subglóticas
podem se apresentar como webs laríngeos, estenoses ou mesmo
atresias;
2. Quadro clínico: pode se apresentar como infecções das vias
aéreas superiores de repetição, diagnosticadas como crupe, em que
um leve edema mucoso resulta em diminuição da luz laríngea e
obstrução abaixo das pregas vocais. Casos mais graves podem
necessitar de traqueostomia ou intubação.

Embora a classificação das estenoses subglóticas varie, o diagnóstico


é basicamente endoscópico.
A obstrução laríngea é classificada em:
1. Grau I: < 50%;
2. Grau II: 51 a 70%;
3. Grau III: 71 a 99%;
4. Grau IV: obstrução completa.

Figura 4.6 - Graus de estenose laríngea


Fonte: adaptado do site do Children’s Hospital of Philadelphia.

b) Atresia

A atresia supraglótica é a malformação laríngea mais grave,


geralmente associada a outras malformações, como atresia
esofágica e fístula traqueoesofágica.
c) Webs – membranas laríngeas

Webs laríngeos – formações diafragmáticas – são malformações


causadas pela falha na recanalização completa da laringe.
Geralmente, localizam-se na porção anterior da luz, permitindo a
passagem de ar posteriormente. Os sintomas mais comuns incluem
dispneia e disfonia.
Webs mais espessos requerem traqueostomia em cerca de 40% dos
casos e reconstrução da comissura anterior por via aberta.
Figura 4.7 - Web laríngeo

Fonte: Combined endoscopic and open approach in treating congenital laryngeal web,
2015.

4.2.2.2 Estenose laríngea adquirida

a) Pós-intubação

A intubação por tempo prolongado pode gerar resposta inflamatória


e consequente retração cicatricial e estenose.
b) Pós-operatória

Frequente após procedimentos cirúrgicos laríngeos. Os fatores


associados são infecções bacterianas, desenvolvimento de tecido de
granulação e presença de DRGE.
Pacientes em UTI sob intubação prolongada, maior do que 5 dias,
devem ser submetidos a traqueostomia, visando evitar estenose
subglótica ou granulomas laríngeos.
4.2.3 Laringocele
1. Definição: os ventrículos da laringe estão limitados superiormente
pelas bandas vestibulares e, inferiormente, pelas cordas vocais.
Possuem um apêndice, também chamado sáculo da laringe. Podemos
definir a laringocele como a dilatação anormal ou herniação do sáculo
laríngeo preenchido por ar. As laringoceles são classificadas em:
a) Interna: está limitada, lateralmente, pela cartilagem tireóidea e,
medialmente, pela parede mucosa da laringe;
b) Externa: o saco herniário se estende através da membrana
tireóidea, formando uma protrusão no pescoço;
c) Mista ou combinada: ambos os componentes estão
presentes;
2. Etiopatogenia: considerada multifatorial e está relacionada com o
aumento da pressão transglótica, como em sopradores de vidro ou em
músicos que tocam instrumentos de sopro;
3. Epidemiologia: são encontradas mais comumente em homens na
quinta ou na sexta década de vida;
4. Quadro clínico: nas internas, podem interferir na fonação e levar a
rouquidão, sensação de corpo estranho, dor de garganta e tosse. Nas
laringoceles externas, há uma massa cervical com ou sem sintomas
laríngeos associados;
5. Diagnóstico: baseia-se em exames de imagem, principalmente a
Tomografia Computadorizada (TC), que pode distinguir entre cistos
preenchidos com ar ou líquidos;
6. Tratamento: depende do tamanho e da repercussão da doença.
Laringoceles pequenas assintomáticas são acompanhadas e só
removidas em caso de se tornarem sintomáticas ou provocarem
alteração estética. No caso de laringoceles sintomáticas, o tratamento
é cirúrgico, com exérese da lesão.
Figura 4.8 - Visão esquemática posteroanterior da laringe

Legenda: (E) Epiglote; (H) osso hioide; (T) banda ventricular; (S) prega vocal; (K)
cartilagem cricoide; (IL) laringocele interna; (AL) laringocele externa.

Fonte: Ilustração Claudio Van Erven Ripinskas.

Quadro 4.3 - Patologias não inflamatórias


4.3 LESÕES BENIGNAS DAS PREGAS
VOCAIS
São lesões não neoplásicas da mucosa das pregas vocais. A queixa
principal é a disfonia, que pode ser contínua ou intermitente, estável
ou progressiva. Deve-se sempre investigar fatores causais,
principalmente o abuso vocal.
O diagnóstico dessas lesões baseia-se na laringoscopia flexível ou
rígida e na estroboscopia laríngea.
4.3.1 Nódulo vocal
São lesões bilaterais e simétricas, geralmente na porção média das
pregas vocais. Acometem preferencialmente mulheres e crianças e
têm relação com o fonotrauma crônico e repetitivo. É muito comum
entre profissionais da voz, como professores.
O tratamento geralmente se inicia com fonoterapia. Quando a lesão
for resistente a tratamentos, a cirurgia poderá ser necessária para a
remoção das lesões.
4.3.2 Pólipo vocal
Trata-se de uma lesão de aspecto polipoide unilateral. Geralmente se
localiza na porção anterior das pregas vocais e está relacionada a
abuso vocal intenso e laringites agudas. O tratamento é cirúrgico,
para remoção e biópsia das lesões, complementado pela fonoterapia.
4.3.3 Edema de Reinke
O edema do espaço de Reinke é uma patologia relacionada
diretamente ao tabagismo. Ocorre o acúmulo de substância
fundamental amorfa no espaço submucoso, o espaço de Reinke. É
mais comum em mulheres, obesas e após a quinta década de vida. A
queixa principal é a disfonia. Outras queixas são tosse, pigarro e
sensação de globus faríngeo.
O diagnóstico é feito com laringoscopia, na qual se observa edema
polipoide difuso e bilateral que pode acometer toda a extensão das
pregas vocais, chegando a provocar dispneia nos casos mais
intensos.
Para o tratamento, o abandono do tabagismo é fundamental. Nos
casos menos sintomáticos, podem-se realizar acompanhamento
periódico e fonoterapia. Já nos casso mais graves, realiza-se cirurgia
para remoção do edema.
4.4 ALTERAÇÕES ESTRUTURAIS
MÍNIMAS DAS PREGAS VOCAIS
As alterações estruturais mínimas são um subgrupo das lesões
benignas de pregas vocais.
4.4.1 Sulco vocal
Trata-se de uma depressão longitudinal ao longo da borda da prega
vocal. Pode ser assintomático ou provocar disfonia. O tratamento
não está bem definido, e os resultados cirúrgicos são limitados.
Figura 4.9 - Sulco vocal à esquerda
4.4.2 Cisto epidermoide – cisto vocal
Trata-se de lesão submucosa de aspecto amarelado ou
esbranquiçado, unilateral, no terço médio das pregas vocais. O
tratamento depende do tamanho da lesão e de suas repercussões na
voz. Considera-se a cirurgia seguida de fonoterapia pós-operatória.
Figura 4.10 - Cisto vocal submucoso à esquerda
Os cistos diferenciam-se dos nódulos por serem
unilaterais e submucosos e acometerem ambos
os sexos.

4.4.3 Vasculodisgenesias – ectasias vasculares


Trata-se de dilatações vasculares encontradas sobre a superfície das
pregas vocais. São indicativas de que outras alterações estruturais
mínimas, já citadas, podem estar presentes.
Figura 4.11 - Vasculodisgenesia bilateral: observar a dilatação dos vasos sanguíneos
mucosos de pregas vocais
Fonte: Brazilian Journal of Otorhinolaryngology.

Quadro 4.4 - Lesões benignas das pregas vocais


4.5 PAPILOMATOSE LARÍNGEA
1. Definição: os papilomas laríngeos são os tumores benignos mais
comuns de laringe, que se apresentam como lesões vegetantes
extremamente recidivantes provocadas pelo papilomavírus humano,
principalmente os tipos 6 e 11;
2. Quadro clínico: nos adultos, o quadro clínico principal é a disfonia
e, nas crianças, a dispneia, por obstrução da passagem aérea;
3. Diagnóstico: baseia-se na laringoscopia. Observam-se múltiplas
lesões de aspecto exofítico, irregulares. As pregas vocais são as
estruturas laríngeas mais afetadas, podendo se estender para
traqueia, brônquios e parênquima pulmonar;
4. Tratamento: é cirúrgico, com a remoção das lesões utilizando o
laser de CO2.

Figura 4.12 - Papiloma faríngeo difuso não obstrutivo


Nota: observar as diversas lesões, indicadas por setas.

Fonte: Center for The Care of The Professional Voice.

4.6 PARALISIA DE PREGAS VOCAIS


A musculatura intrínseca da laringe é inervada principalmente pelo
Nervo Laríngeo Recorrente (NLR), que é ramo do nervo vago NC X. À
direita, o NLR emerge do NC X abaixo do nível do tronco
braquiocefálico, contornando-o, de modo que o NLR ascende à
laringe a partir dessa região; já à esquerda, o NLR emerge do NC X
abaixo da crossa da aorta, contornando-a e depois ascendendo à
laringe (Figura 4.13).
Figura 4.13 - Inervação motora da laringe
Nota: à direita, o nervo laríngeo recorrente contorna o tronco braquiocefálico, enquanto à
esquerda contorna a crossa da aorta.

Fonte: adaptado de Unilateral Vocal Cord Paralysis: A Review of CT Findings, 2015.

São tipos de paralisia:


1. Unilateral:
a) Em adução – pregas fechadas: o paciente pode apresentar-
se assintomático;
b) Em abdução – pregas abertas: os pacientes cursam com
disfonia, voz soprosa e cansaço ao falar.
2. Bilateral:
a) Em adução: pode não alterar significativamente a voz, porém
é comum dispneia devido à diminuição da luz laríngea durante a
inspiração;
b) Em abdução: cursa com disfonia intensa e cansaço ao falar.
As etiologias são:
1. Lesão cirúrgica: é a etiologia mais comum. Pode ocorrer lesão do
NLR durante cirurgia de tireoide ou outras cirurgias cervicais;
2. Compressões: considerando o trajeto do nervo vago e do NLR,
devemos investigar possíveis lesões compressivas desses nervos.
Para tanto, é importante a solicitação de exames de imagem, TC ou
RNM do trajeto do NC X e do NLR desde a base do crânio até o
mediastino. As lesões compressivas mais comumente encontradas
são:
a) Tumores no trajeto do nervo: metástases cervicais,
infiltrações tumorais diversas, linfoma mediastinal;
b) Aneurisma de aorta torácica ou de tronco braquiocefálico:
o aumento de diâmetro do vaso distende o nervo, causando perda
de função e paralisia.

O tratamento é direcionado à etiologia de base. A qualidade vocal


pode ser melhorada com fonoterapia ou abordagem cirúrgica,
lateralização ou medianização da prega vocal. A traqueostomia é
recomendada em casos de paralisia em adução e é mandatória a
pacientes com dispneia.
No caso de paralisia unilateral de pregas vocais sem etiologia
evidente, como traumas, deve-se fazer avaliação radiológica –
tomografia ou ressonância – que compreenda o trajeto do nervo
vago desde a emergência do crânio até a crossa da aorta.
4.7 MASSAS CERVICAIS CONGÊNITAS
Frequentemente têm aparência característica e geralmente refletem
anomalias cervicais no desenvolvimento de músculos, pele, vasos
sanguíneos, linfáticos e aparato branquial.
4.7.1 Embriologia
São afecções cervicais de características tumorais que podem se
apresentar na região cervical da criança ou do adulto jovem. A partir
da terceira semana de vida intrauterina, começam a surgir as
estruturas que formarão o pescoço do embrião.
4.7.1.1 Arcos faríngeos

São estruturas laminares que contêm, em seu interior, uma lâmina


de cartilagem, uma artéria, um músculo e um nervo craniano.
O primeiro arco a se formar é o mandibular. É o que tem
desenvolvimento mais rápido e intenso, seguido pelo arco hioideo e
pelo terceiro, quarto, quinto e sexto arcos, todos menos
desenvolvidos. Seu desenvolvimento faz com que as fendas e bolsas
sejam ocluídas ou ocupadas por estruturas sólidas.
Figura 4.14 - Arcos faríngeos e suas estruturas

Em vermelho: primeiro arco, formando martelo, bigorna e mandíbula; em azul: segundo


arco, formando o estribo; em verde: terceiro arco, formando o hioide.

Fonte: adaptado do site Embryology.


4.7.1.2 Bolsas faríngeas

São espaços localizados na face interna – endoderma – entre os arcos


faríngeos, que, conforme seu desenvolvimento, formarão diversas
estruturas cervicofaciais. Os derivados das bolsas faríngeas são:

1. Primeira bolsa: formação da orelha média, tuba auditiva e


mastoide;
2. Segunda bolsa: tonsilas palatinas;
3. Terceira bolsa: glândulas paratireoides inferiores e timo;
4. Quarta bolsa: glândulas paratireoides superiores, tireoide e
musculatura e cartilagem laríngeas;
5. Quinta bolsa: estrutura rudimentar que forma, mais tarde, parte da
glândula tireoide;
6. Sexta bolsa: junto com a quarta bolsa, forma musculatura e
cartilagens laríngeas.

Tudo isso acontece da terceira à décima semana de gestação, um


período de intensa atividade embriológica, mas bastante curto.
4.7.1.3 Fendas branquiais

1. Primeira fenda branquial: formará o conduto auditivo externo;


2. Segunda, terceira e quarta fendas: são fundidas ao segundo arco
branquial e formarão o seio cervical, que futuramente será obliterado.

Figura 4.15 - Formação dos cistos e das fístulas branquiais

Fonte: Claudio Van Erven Ripinskas.


4.7.2 Cistos e fístulas da primeira fenda branquial
Os cistos da primeira fenda branquial são de 2 tipos:
1. Tipo I: localizados perto do canal auditivo externo, inferiores e
posteriores ao trágus, podem também estar na glândula parótida ou no
ângulo da mandíbula;
2. Tipo II: podem ser difíceis de distinguir de uma massa sólida na
parótida ao exame clínico. Cistos do tipo II são associados à glândula
submandibular ou encontrados no triângulo anterior do pescoço.
Realce capsular fino é observado em exames pós-contraste.

Figura 4.16 - Cistos de primeira fenda branquial, tipo I


Fonte: JAMA Otolaryngology - Head and Neck Surgery.

Figura 4.17 - Cisto de primeira fenda branquial tipo II

Nota: observar a proximidade com a mandíbula.

Fonte: Radiopaedia.

O tratamento das anomalias de primeira fenda branquial pode variar


de expectante nos casos em que há apenas alteração cosmética da
região malar, cervical ou pré-auricular até a exérese cirúrgica da
lesão.
Figura 4.18 - Cistos e fístulas da primeira fenda branquial

Fonte: Ilustração Claudio Van Erven Ripinskas.

4.7.3 Cistos e fístulas da segunda fenda branquial


A fenda que fornece o maior número de casos clínicos é a segunda. O
exame clínico geralmente encontra uma lesão lisa, fibroelástica, com
mobilidade lateral, porém com redução de sua movimentação
vertical. Sua localização preferencial é a parte mais alta da região
jugulocarotídea, abaixo do ângulo da mandíbula e na frente dos
grandes vasos. Tem relação com o músculo esternocleidomastóideo
e aparece entre os 10 e 20 anos, em geral, após quadro infeccioso da
cavidade oral.
O diagnóstico é feito pela história característica de massa de
crescimento relativamente rápido, de 1 a 4 semanas, na região
cervical lateral, com dor local e sinais flogísticos leves a moderados,
associado a um processo infeccioso de boca e orofaringe.
O principal exame radiográfico é a TC, que mostra uma massa
cística, de cápsula fina, com conteúdo líquido que se cora levemente
com contraste e que pode formar septos.
O tratamento é eminentemente cirúrgico.
Figura 4.19 - Cisto da segunda fenda branquial
Nota: observar o realce periférico e a íntima relação com o músculo
esternocleidomastóideo; (m) lesão e (s) esternocleidomastóideo.

Fonte: Medscape.

Figura 4.20 - Indivíduo com cisto da segunda fenda branquial


Fonte: Atlas of Pediatric Surgery.

Figura 4.21 - Cistos e fístulas da segunda fenda branquial

Fonte: Ilustração Claudio Van Erven Ripinskas.

Quadro 4.5 - Cistos e fístulas da primeira e segunda fendas branquiais


4.7.4 Cistos e fístulas da terceira e quarta fendas
branquiais
São acometimentos muito raros. Os cistos da terceira fenda
branquial localizam-se posteriormente à carótida; os da quarta
fenda acompanham o trajeto do NLR.
4.7.5 Linfangiomas cervicais – higroma cístico
1. Definição: tumor benigno de formações císticas desenvolvidas a
partir do endotélio linfático e das ilhas de linfa e sangue; tais tumores
estão rodeados por tecido fibroadiposo;
2. Quadro clínico: a apresentação clínica mais frequente constitui
uma tumoração cervical indolor heterogênea, amolecida e
assintomática. Em 50% dos casos, o diagnóstico é feito ao nascimento
e os demais casos tendem a se manifestar durante os primeiros anos
de vida;
3. Diagnóstico: a transiluminação é patognomônica, mas sua
ausência não permite descartar o diagnóstico. A pele sobre o tumor
costuma ser normal ou ter aparência ligeiramente azulada. Na TC, as
lesões císticas apresentam densidade líquida. A RNM é considerada
técnica de escolha, e o higroma cístico apresenta hipersinal
característico em T2;
4. Tratamento: a literatura favorece a cirurgia como tratamento de
escolha para linfangiomas cervicais. A regressão espontânea dos
higromas tem sido ponto de discussão em muitos estudos. Existem
evidências suficientes para considerá-la opção terapêutica em casos
em que uma massa assintomática é o único problema.

Figura 4.22 - Neonato com higroma cístico na face lateral do pescoço

Fonte: Medical Pictures Info.

Figura 4.23 - Ressonância magnética mostrando higroma cístico indicado por setas
Nota: observar as septações na face lateral do pescoço.

Fonte: adaptado de Imaging Features of Common Non-nodal Neck Masses in Children,


2006.

4.7.6 Cistos do ducto tireoglosso


4.7.6.1 Embriologia

A glândula tireoide começa a se desenvolver na orofaringe no feto e


migra para a sua posição final tomando caminho através da língua,
foramen cecum, do osso hioide e dos músculos do pescoço. A ligação
entre a sua posição original e a sua posição final é o ducto
tireoglosso. Esse ducto normalmente atrofia antes do nascimento,
mas pode permanecer em algumas pessoas (Figura 4.24).
Figura 4.24 - Etapas de desenvolvimento da tireoide e do ducto tireoglosso
A persistência de uma porção do ducto com seu epitélio resultará em
lesão cística preenchida por material coloide. O osso hioide está
intimamente envolvido com o ducto tireoglosso. Como resultado, o
ducto pode estar localizado anteriormente, na sua substância ou
atrás do osso hioide.
4.7.6.2 Apresentação clínica

O cisto do ducto tireoglosso é a massa cervical benigna mais comum.


A maioria dos cistos torna-se sintomática antes de o paciente
completar 5 anos, mas pode ocorrer em qualquer idade. Apresenta-
se como uma massa sólida em linha média cuja principal
característica é a elevação durante a protrusão da língua e à
deglutição. Durante os episódios de infecção das vias aéreas
superiores, o cisto pode edemaciar e apresentar outros sinais
flogísticos, porém retorna ao seu estado original após a resolução do
quadro infeccioso.
4.7.6.3 Diagnóstico

Baseia-se nos achados já descritos e em exame radiológico,


preferencialmente TC. Tais exames nos permitem avaliar a extensão
da lesão e sua relação com estruturas adjacentes.
4.7.6.4 Histologia

Histologicamente, encontramos um canal unido ao cisto,


intimamente relacionado ao corpo do osso hioide e revestido por
epitélio escamoso glandular. Em alguns casos, encontramos ilhas de
tecido tireoidiano associadas à malformação.
4.7.6.5 Tratamento

Em 1920, Sistrunk recomendou a retirada de um bloco de tecido


desde a base da língua envolvendo o ducto, o cisto e parte do osso
hioide (Figura 4.25). Com essa técnica, a taxa de recidiva é de cerca
de 3%.
Figura 4.25 - Cirurgia de Sistrunk

Legenda: (A) acesso por cervicotomia anterior; (B) dissecção do cisto; (C) exérese da
porção medial do hioide; (D) ligadura do ducto tireoglosso residual.

4.7.7 Hemangiomas
Os hemangiomas são as neoplasias de cabeça e pescoço mais comuns
na infância. Embora sejam predominantemente localizados em
superfícies cutâneas, podem ser vistos também em mucosas e
vísceras.
Cerca de 1 terço dos hemangiomas já está presente ao nascimento,
mas tipicamente é notado durante o primeiro mês de vida e aumenta
progressivamente durante o primeiro ano.
Os métodos diagnósticos de escolha são a RNM e a ultrassonografia
(USG).
Em relação ao tratamento, a terapia conservadora é regra para sua
maioria, mas a observação está indicada para o possível
desenvolvimento de complicações. Cerca de 10% a 20% dos
hemangiomas podem demandar outras opções de tratamento.
Figura 4.26 - Hemangioma de face lateral do pescoço

Fonte: Nicklaus Children’s Hospital.

4.7.8 Cistos dermoides


Os cistos dermoides consistem em uma cavidade formada por
epitélio e preenchida por apêndices de pele, como cabelo, glândulas
sebáceas e pelos. São lesões de 1 a 4 cm (Figura 4.27) que ocorrem
mais comumente na cabeça – testa, principalmente em torno dos
olhos, e podem apresentar um apêndice com extensão intracraniana,
especialmente as lesões glabelares.
Na região cervical em geral movem-se com o deslocamento da pele e
são indolores, exceto quando infectados. Ao contrário dos cistos do
ducto tireoglosso, não se movem com a protrusão da língua. A TC e a
RNM são úteis para fazer o diagnóstico diferencial correto dos cistos
dermoides. O tratamento, por sua vez, é feito com excisão completa.
Figura 4.27 - Cisto dermoide no dorso nasal e na sobrancelha
Fonte: Pathology Outlines.

Quadro 4.6 - Massas cervicais congênitas de cistos dermoides


4.8 NEOPLASIAS DE LARINGE
A maioria das neoplasias tem origem nas pregas vocais verdadeiras
(55% a 75%), sendo a rouquidão que persiste por mais de 2 semanas
um forte sinal de que uma investigação diagnóstica apropriada deve
ser iniciada. Os tumores também podem se originar de região supra
– 25% – ou infraglótica – 5% – (Figura 4.28).
Toda disfonia com duração maior que 15 dias
deve ser investigada com avaliação
endoscópica de laringe.

Figura 4.28 - Frequência de distribuição do carcinoma espinocelular laríngeo


Fonte: adaptado do site Oncolink.

O carcinoma espinocelular (CEC) representa aproximadamente 95%


dos tumores dessa região. Raramente há tumores das glândulas
salivares menores.
Os principais fatores de risco são o tabagismo, seguido pelo etilismo
crônico, e a associação desses 2 fatores potencializa os riscos.
O diagnóstico baseia-se principalmente na história clínica de
disfonia. Pode também haver globus faríngeo, pigarro, odinofagia,
disfagia, tosse e voz soprosa. À nasofibroscopia, observa-se lesão
vegetante e invasiva. A biópsia é necessária para confirmar a lesão
tumoral.
Uma vez confirmado o CEC, realiza- se o estadiamento,
principalmente com TC de pescoço e tórax. O risco relativo de
metástase cervical é diretamente proporcional ao tamanho do
tumor.
Tumores supraglóticos apresentam altos índices de metástases
linfáticas regionais, enquanto os tumores glóticos apresentam
incidência menor. Tumores subglóticos têm incidência variável de
metástases.
Quadro 4.7 - Estadiamento
4.8.1 Tratamento e sobrevida
Carcinoma in situ e displasia severa requerem decorticação – exérese
da mucosa superficial – da prega vocal acometida.
Os tumores iniciais podem ser tratados com bons resultados, tanto
com cirurgia quanto com radioterapia.
Tumores supraglóticos avançados têm sobrevida em 5 anos de, no
máximo, 50%, similarmente a tumores glóticos T4 (40 a 49%).
Tumores T3 glóticos podem ter sobrevida de até 80% em 5 anos
(Quadro 4.8).
Quadro 4.8 - Sobrevida em 5 anos para tumores glóticos e supraglóticos, de acordo com a
terapêutica utilizada
Fonte: Instituto Nacional de Câncer.

4.9 FARINGOTONSILITES
As faringotonsilites, ou anginas, são doenças inflamatórias e
infecciosas envolvendo faringe, tonsilas palatinas –amígdalas – e
tonsilas faríngeas – adenoides. Constituem um dos distúrbios mais
frequentes nos consultórios médicos e pronto atendimentos.
4.9.1 Imunologia
As tonsilas são órgãos linfoides secundários compostos
primariamente por linfócitos, localizados na entrada da via aérea.
Apresentam função de captação, processamento de antígeno e
resposta primária, além de produção de anticorpos, mais
especificamente a IgA, principalmente entre os 4 e 10 anos. A
involução do tecido linfoide tende a ocorrer após a puberdade. Em
relação aos anticorpos, a adenotonsilectomia cursa com redução
inicial dos níveis de IgG e IgA séricos principalmente no primeiro
mês pós-operatório. Após 3 meses, tais níveis já se restabeleceram.
Essa variação não é suficiente para causar imunossupressão no
indivíduo.
As faringotonsilites são clinicamente classificadas em:
1. Amigdalite aguda: o quadro é de início rápido e sintomas
evidentes;
2. Amigdalite recorrente: segundo a American Academy of
Otolaryngology – Head and Neck Surgery, amigdalite recorrente é
definida como 7 episódios de amigdalite em 1 ano, 5 episódios por ano
em 2 anos consecutivos ou 3 episódios por ano em 3 anos
consecutivos;
3. Amigdalite crônica: o quadro é de dor faríngea persistente,
frequentemente associada a cálculos, ou cáseos, em tonsilas;
4. Hiperplasia amigdaliana: ocorre aumento volumétrico de
amígdalas por hiperplasia do órgão, com obstrução nasal, roncos e
apneia.

4.9.2 Tonsilites eritematosas


São as anginas mais comuns, com cerca de 90% dos casos. Podem
ser de origem viral ou bacteriana e cursam com mucosa
hiperemiada, edema de amígdalas. Apresentam eventual exsudato
esbranquiçado que se desprende facilmente da mucosa.
4.9.2.1 Origem viral
As amigdalites de origem viral correspondem a 75% das
faringoamigdalites agudas. Os principais agentes são: rinovírus
(20%); coronavírus (5%); adenovírus (5%); herpes simples (4%);
influenza (2%); parainfluenza (2%).
Vírus menos comuns são Coxsackievirus, citomegalovírus, vírus
Epstein-Barr (EBV) e HIV. A faringoamigdalite causada pelo EBV
apresenta peculiaridades que serão detalhadas adiante.
Quanto ao quadro clínico, os sintomas são leves. Os principais são
dor de garganta e disfagia, podendo ocorrer mialgia, febre baixa,
espirros e coriza hialina. Há eritema da mucosa faríngea e edema de
tonsilas. Não se observa exsudato. E o tratamento consiste em
terapias de suporte: analgésicos, anti-inflamatórios, hidratação e
repouso.
4.9.2.2 Origem bacteriana

As amigdalites de etiologia bacteriana acometem de 20 a 40% dos


indivíduos.
a) Agentes etiológicos

Streptococcus pyogenes – estreptococo beta-hemolítico do grupo A –


ocorre em 30% das faringotonsilites agudas em crianças em idade
escolar e adolescentes. Mycoplasma pneumoniae ocorre entre os 9 e
20 anos, embora alguns autores discutam sua importância. Outras
bactérias, como Staphylococcus aureus, Haemophilus sp. E Moraxella
catarrhalis são menos comuns e em geral produzem betalactamase.
b) Faringoamigdalite estreptocócica

1. Epidemiologia: a faringite aguda causada pelo estreptococo do


grupo A tem importância em Saúde Pública devido à sua alta
frequência e suas potenciais complicações, como febre reumática e
glomerulonefrite difusa aguda;
2. Quadro clínico: ocorre após os 3 anos, com pico entre 5 e 10 anos.
A sintomatologia é composta por dor faríngea, odinofagia, febre acima
de 38,5 °C, cefaleia, calafrios, dor abdominal. Pode haver toxemia e
prostração, principalmente em adultos. O exame físico mostra
hiperemia, aumento de tonsilas e exsudato purulento, além de
adenomegalia em cadeia jugulodigástrica;
3. Diagnóstico: é basicamente clínico. Entre os testes rápidos para a
detecção do Streptococcus, o strepto test, feito por meio da coleta de
secreção de orofaringe, permite identificar a presença de antígenos do
Streptococcus; quando feito idealmente, tem uma sensibilidade de
95% e especificidade de 98% e é amplamente utilizado na prática
clínica. Nos pacientes com grande suspeita clínica e resultado do teste
rápido negativo, a cultura pode ser utilizada para elucidação
diagnóstica, porém não se deve retardar o início da antibioticoterapia.
O hemograma geralmente se apresenta normal ou com leucocitose
com desvio à esquerda. Os exames sorológicos são de pequena
utilidade, uma vez que a elevação dos títulos de anticorpos –
antiestreptolisina O (ASLO), anti-hialuronidase, anti-DNAse B,
antiestreptoquinase – ocorre 2 ou 3 semanas após a fase aguda;
4. Tratamento: seguindo as orientações da Anvisa, o tratamento deve
ser feito como segue:
a) Primeira escolha é a penicilina G benzatina em dose única para
adultos e crianças;
b) Medicações orais:
Amoxicilina por 10 dias. Os macrolídeos – eritromicina,
azitromicina – ficam limitados a casos de alergia a penicilina;
Em caso de suspeita de germes produtores de
betalactamase, a amoxicilina pode ser associada ao ácido
clavulânico. Também se considera axetilcefuroxima ou
cefaclor como opções;
Em casos de resistência ou impossibilidade de administração
oral, tratamento parenteral pode ser feito com ceftriaxona ou
clindamicina.

Quando o tratamento é iniciado prontamente, encurtam-se o


período de transmissão, o tempo dos sintomas e a incidência de
complicações supurativas. Adiar o tratamento até 9 dias após o início
da faringite parece não aumentar o risco de febre reumática.
c) Complicações de anginas estreptocócicas

As complicações se dividem em:


1. Complicações não supurativas:
a) Escarlatina: decorre da produção de endotoxinas pelo
estreptococo. O quadro inclui rash cutâneo finamente papular e
eritematoso, conferindo à pele um aspecto áspero. Observam-se
febre, eritema de amígdalas e orofaringe. O sinal de Filatov
consiste em palidez perioral, enquanto o sinal de Pastia se
caracteriza pelo surgimento de hiperpigmentação em linhas de
flexão. A língua “em framboesa” também é característica. O
diagnóstico é clínico e o tratamento é feito com penicilina G
intravenosa;
b) Febre reumática: doença endêmica nos países em
desenvolvimento. O pico de incidência ocorre entre 5 e 15 anos e
os sinais e sintomas se manifestam de 2 a 3 semanas após um
episódio de faringite estreptocócica. Os critérios de Jones
modificados podem ser usados: quando há 2 critérios maiores ou
1 critério maior e 2 menores associados à evidência de infecção
estreptocócica recente. Cardite e valvulite podem ser
autolimitadas ou provocar degeneração valvar progressiva. Os
critérios de Jones para o diagnóstico de febre reumática são:
Critérios maiores:
Cardite – valvulite;
Poliartrite;
Eritema marginado;
Coreia;
Nódulos subcutâneos.
Critérios menores:
Febre;
Artralgia;
Antecedente de febre reumática;
Aumento da velocidade de hemossedimentação;
Aumento da proteína C reativa;
Aumento do intervalo PR.
Evidências de infecção:
Escarlatina recente;
Presença de anticorpos (ASLO, antiestreptoquinase,
anti-DNAse B);
Cultura positiva.
c) Glomerulonefrite: ocorre após infecção faríngea ou de pele, e
a fisiopatologia não é totalmente elucidada. O paciente apresenta
síndrome nefrítica 1 a 2 semanas após infecção de orofaringe. No
tratamento, não há evidência de que a administração de penicilina
diminua a taxa de ataque ou altere a história natural da
glomerulonefrite; em casos mais intensos, indicam-se diuréticos.
Restrição de sódio e líquidos deve ser indicada;
d) Síndrome do choque tóxico estreptocócico: ocorre após
infecção ou colonização estreptocócica de qualquer sítio.
Compreende hipotensão associada a ao menos 2 dos seguintes
fatores: insuficiência renal, coagulopatia, anormalidades de
função hepática, síndrome da angústia respiratória do adulto,
necrose tecidual extensa e rash eritematomacular.
3. Complicações supurativas: compreendem os abscessos, que
geralmente se iniciam no espaço periamigdaliano – cápsula fibrosa
entre a musculatura do palato mole e as amígdalas – e podem se
estender para outros espaços cervicais profundos, principalmente o
espaço parafaríngeo;
a) Etiologia: em geral, as culturas são multimicrobianas. Os
principais agentes encontrados são: anaeróbios, estreptococos
beta-hemolíticos do grupo A, estafilococos, pneumococos e
Haemophilus;
b) Quadro clínico: os pacientes apresentam-se tipicamente com
história de amigdalite aguda com agravamento do desconforto
faríngeo unilateral; ocorrem halitose, odinofagia, disfagia e uma
voz “abafada”. A presença de trismo, limitação na capacidade de
abrir a cavidade oral, é fortemente sugestiva de abscesso;
c) Exame físico: observa-se, principalmente, abaulamento
anteromedial de pilar amigdaliano associado a rechaçamento
lateral da úvula e trismo;
d) Diagnóstico: baseia-se no quadro descrito; a avaliação com
TC com contraste permite quantificar o volume e a extensão do
quadro (Figura 4.29). A TC também permite avaliar a extensão do
abscesso para os demais espaços cervicais;
e) Tratamento: inicialmente, deve-se realizar a drenagem do
abscesso, mantendo um pertuito aberto para manter a drenagem
nos dias seguintes. Segundo as orientações da Academia
Brasileira de Otorrinolaringologia, a amigdalectomia não é
recomendada. Uma vez drenado, deve-se iniciar
antibioticoterapia, podendo-se utilizar amoxicilina + ácido
clavulânico, ceftriaxona ou clindamicina. A corticoterapia também
deve ser realizada. Na suspeita de abscesso periamigdaliano,
deve-se realizar tomografia de pescoço para confirmar o quadro.
E, em pacientes com flegmão, sem abscesso formado, deve-se
considerar a ampliação do espectro do antibiótico, introduzindo-se
amoxicilina + ácido clavulânico, associados a corticoterapia.

Figura 4.29 - Tomografia de pescoço com contraste, janela de partes moles e corte axial

Nota: observar o abscesso periamigdaliano à esquerda, com realce periférico – seta


amarela –, e o rechaçamento medial da amígdala esquerda em relação à direita – pontas
de setas.

4.9.2.3 Mononucleose infecciosa e as síndromes mono-like


A mononucleose infecciosa (MNI) apresenta caráter sistêmico e
acomete, com maior frequência, adolescentes e adultos jovens. Seu
agente é o vírus Epstein-Barr (EBV), da família Herpesviridae.
Diversos agentes cursam com sintomas de MNI, as chamadas
síndromes mononucleose-like, sendo os principais agentes e
doenças citomegalovírus, toxoplasmose aguda, HIV agudo,
Trypanosoma cruzi, herpes-vírus, adenovírus e Treponema pallidum –
sífilis primária.
1. Quadro clínico: tríade clássica de febre, dor faríngea e
adenomegalia cervical. A tonsila pode estar hiperemiada, bem como
recoberta por secreção purulenta ou pseudomembranosa. Os sinais
sistêmicos são petéquias em palato (40%), icterícia (15%),
hepatomegalia (10%) e esplenomegalia (50%). Esse diagnóstico deve
ser considerado em pacientes cujos sintomas persistem mesmo
durante antibioticoterapia. Geralmente os sintomas persistem por até 4
semanas;
2. Diagnóstico: baseia-se na urgência do quadro clínico e no
hemograma. As sorologias levam dias para ficar prontas e não devem
retardar o início do tratamento;
3. Diagnóstico laboratorial: hemograma que evidencia linfocitose –
linfócitos > 50% da população de leucócitos – com atipia linfocitária –
10% ou mais dos leucócitos totais – e elevação de TGO e TGP;
4. Sorologias:
a) IgM pode ser detectado desde o início do quadro clínico,
atingindo seu pico em 4 a 6 semanas. IgG pode apresentar-se
permanentemente elevado;
b) Teste de Paul-Bunnell-Davidsohn: positivo após 20 dias de
doença, com alta sensibilidade e especificidade.
5. Tratamento: baseia-se em terapias de suporte como hidratação e
analgésicos. O uso de corticoides permanece controverso. Rash
cutâneo pode ocorrer com o uso de penicilina-ampicilina, muitas vezes
prescrito ao se considerar o quadro como de etiologia bacteriana;
6. Complicações: na grande maioria dos casos, a mononucleose é
autolimitada, no entanto algumas possíveis complicações são:
a) Esplenomegalia: ocorre em cerca de 75% dos casos e regride
espontaneamente;
b) Rotura esplênica é um distúrbio raro que ocorre em casos mais
intensos e deve ser abordado em caráter de urgência, com
exérese do órgão;
c) Anemia leve que, em geral, regride em 2 a 3 semanas;
d) Distúrbio de ritmo cardíaco;
e) Encefalite, meningite;
f) Surdez súbita;
g) Síndrome de Guillain-Barré.
7. Patologias associadas:
a) Esclerose Múltipla (EM): recentes estudos mostram
associação de EBV com EM. A infecção tardia pelo vírus parece
ser um fator de risco para EM;
b) Linfoma de Hodgkin: forte associação com infecção pelo
EBV, principalmente em crianças;
c) Carcinoma de rinofaringe: há também associação
fisiopatológica entre EBV e esse carcinoma.

4.9.3 Forma clínica particular: difteria


1. Definição: a difteria é uma doença infecciosa agravada pela toxina
diftérica, produzida por seu agente infeccioso;
2. Etiologia: infecção pelo Corynebacterium diphtheriae, Gram positivo
anaeróbio facultativo, produtor de endotoxina, responsável pelos
fenômenos locais e sistêmicos da doença. Acomete, principalmente,
crianças entre o primeiro e o sétimo anos de vida e hoje em dia é rara
devido à vacinação antidiftérica;
3. Quadro clínico: dor faríngea de início insidioso, febre baixa e
prostração. Ao exame, observam-se pseudomembranas branco-
acinzentadas sobre as amígdalas e invadindo os pilares anteriores até
a úvula. As falsas membranas aderem à mucosa, resistem ao
descolamento – deixam leito sangrante ao se destacarem – e não se
dissociam na água. Há linfonodomegalia cervical, e nos casos mais
graves, há intenso aumento do pescoço – pescoço taurino –, por
comprometimento dos gânglios linfáticos dessa área;
4. Complicações: miocardite, neurites periféricas, nefropatia tóxica,
insuficiência renal aguda;
5. Diagnóstico: é confirmado pelo exame bacterioscópico direto e pela
cultura de exsudatos faríngeos ou até de fragmento de
pseudomembrana em meio de Klebs-Loeffler;
6. Tratamento: internação e soroterapia específica: soro antidiftérico,
intramuscular ou subcutâneo. Penicilina ou eritromicina devem ser
utilizadas para assegurar a erradicação do foco infeccioso.
Os comunicantes de um caso índice de difteria não vacinados,
inadequadamente vacinados, vacinados há mais de 5 anos ou com
situação vacinal desconhecida devem receber a vacina antidiftérica e
ser submetidos à coleta de material de orofaringe para cultura. No
caso de serem portadores do bacilo, está indicada quimioprofilaxia
com eritromicina.
A difteria é uma doença de notificação compulsória e de investigação
imediata e obrigatória.
Figura 4.30 - Pseudomembranas faríngeas em paciente com difteria

4.9.4 Anginas ulceronecróticas


4.9.4.1 Angina de Plaut-Vincent – gengivite ulcerativa necrosante
aguda

1. Agente etiológico: o bacilo fusiforme Fusobacterium plaut-vincenti


e o espirilo Spirochaeta dentium são saprófitos normais da cavidade
bucal e adquirem poder patogênico quando em simbiose. Os fatores
de risco são má higiene bucal e mau estado dos dentes e das
gengivas. Acomete, principalmente, adultos jovens e adolescentes em
estado nutricional inadequado;
2. Quadro clínico: odinofagia importante unilateral e disfagia intensa,
sem febre, com queda do estado geral. À oroscopia, observam-se
ulceração da amígdala e pseudomembrana, desprendida e friável
(Figura 4.31), sem tendência a estender-se, mas acarretando
fenômenos de necrose e eliminação de exsudato de odor fétido.
Frequentemente acomete as gengivas e região periodontal
concomitantemente (Figura 4.32);

Figura 4.31 - Angina de Plaut-Vincent – ulceromembranosa

Nota: observar as membranas acinzentadas nas tonsilas, com ulceração à direita.

Figura 4.32 - Gengivoestomatite necrosante


3. Diagnóstico: quadro clínico com acometimento unilateral das
lesões. O exame bacteriológico auxilia a evidenciar a natureza
fusoespiralar do agente;
4. Tratamento: penicilina por via parenteral ou metronidazol,
gargarejos com soluções antissépticas e sintomáticos.

A angina de Plaut-Vincent tem maior ocorrência


em imunossuprimidos e pacientes com dentes
em mau estado.

4.9.5 Vírus herpes simples


1. Agente etiológico: HSV subtipos 1 e 2;
2. Quadro clínico: a infecção primária é geralmente
gengivoestomatite, mas pode se manifestar como faringite aguda ou
mono-like. O quadro é mais comum nos primeiros anos de vida. Inicia-
se um quadro sistêmico e há evolução com lesões vesiculosas na
orofaringe e nas tonsilas. Pode ser encontrada linfonodomegalia
cervical e submentoniana. O quadro regride após 7 a 10 dias do pico
das lesões. O vírus pode permanecer latente em gânglios nervosos
sensitivos, voltando em situações de estresse;
3. Complicações: as possíveis complicações acontecem em casos de
infecção disseminada, comprometendo o sistema nervoso central ou
os dermátomos;
4. Diagnóstico: na maioria das vezes é clínico. Avaliação laboratorial
pode ser feita por meio da cultura viral, com observação de células
“balão” e por imunofluorescência contra as células infectadas. Outro
método é o esfregaço de Tzanck, porém este não distingue entre HSV
1 ou 2. A PCR é utilizada em casos específicos, principalmente para
diferenciação dos tipos virais. Por fim, a conversão sorológica também
tem seu valor diagnóstico;
5. Tratamento: baseia-se em sintomáticos, podendo utilizar aciclovir
por 7 a 10 dias.

4.9.5.1 Herpangina – síndrome mão-pé-boca

1. Agente etiológico: Coxsackievirus A, Coxsackievirus B,


enterovírus;
2. Quadro clínico: o paciente apresenta angina eritematosa com
vesículas pequenas no palato mole, na úvula e nos pilares
amigdalianos (enantema – Figura 4.33). As vesículas formam
ulcerações esbranquiçadas ao se romperem e são circundadas por
halo eritematoso espalhadas por toda a orofaringe. É comum em
crianças, podendo ocorrer em adultos jovens. Incide principalmente no
verão e acompanha febre, cefaleia e micropoliadenopatia cervical, com
resolução em 5 a 10 dias; em crianças é comum a ocorrência
concomitante de lesões vesiculosas em palma da mão e planta do pé,
daí o nome da síndrome;
3. Tratamento: é sintomático, com analgésicos e hidratação.

Figura 4.33 - Herpangina


Nota: observe as lesões vesiculares em palato e úvula.

Fonte: James Heilman, MD, 2016.

Quadro 4.9 - Anginas


4.9.6 Hiperplasia adenotonsilar
A hiperplasia das tonsilas faríngeas, adenoide, e ou palatinas,
amígdalas, é um distúrbio de etiologia ainda não esclarecida que
acomete cerca de 5% das crianças de 2 a 10 anos. A hiperplasia
adenoidiana tem, como principal sintoma, roncos noturnos com
períodos de apneia.
Clinicamente, os pacientes apresentam obstrução nasal persistente,
respiração bucal, roncos com períodos de apneia noturna, bruxismo,
sonolência diurna, diminuição da velocidade de crescimento e do
ganho de peso. A cronicidade da hiperplasia, e principalmente da
respiração oral, gera consequências descritas a seguir:
1. Alterações na morfologia craniofacial: palato ogival, alterações
no desenvolvimento dentário, hipoplasia maxilar;
2. Otite média secretora: aparentemente ligada à obstrução do tórus
tubário na rinofaringe;
3. Distúrbios comportamentais e neurológicos: aparentemente
relacionados à má qualidade do sono;
4. Distúrbios de aprendizado: também relacionados à má qualidade
do sono;
5. Distúrbios mastigatórios: relacionados a obstrução nasal, mau
posicionamento dentário e alteração do crescimento orofacial;
6. Distúrbios da fala: decorrentes da alteração do crescimento
orofacial, podendo ser agravados nos casos de otite secretora
concomitante.

Ao exame físico, observa-se aumento das tonsilas, sinais de


hipotonia da musculatura do terço inferior da face, respiração oral
persistente, projeção anterior da cabeça e voz anasalada. Os testes
complementares mais utilizados para diagnóstico são:
1. Radiografia de cavum em perfil: observa-se a redução do espaço
da rinofaringe por aumento da adenoide (Figura 4.34);
2. Nasofibrolaringoscopia: permite a visualização direta da adenoide
e avaliação do percentil aproximado de obstrução do espaço faríngeo
(Figura 4.35).
A avaliação da apneia noturna é basicamente clínica, por meio do
questionamento aos pais sobre períodos de parada respiratória,
salivação noturna, sono em posições específicas e principalmente de
roncos. A polissonografia é pouco utilizada em crianças.
Figura 4.34 - Radiografia de cavum em perfil mostrando aumento de tecido adenoidiano

Figura 4.35 - Avaliação nasofibroscópica de fossa nasal esquerda: observar o tecido


adenoidiano obstruindo cerca de 70 a 80% do espaço faríngeo
O tratamento, em casos menos severos, com roncos e obstrução
nasais intermitentes ou leves, pode ser feito com terapia clínica, com
corticoterapia tópica nasal (budesonida, fluticasona, mometasona)
associada a inibidores dos receptores de leucotrieno (montelucaste).
O tratamento cirúrgico, adenoidectomia ou adenoamigdalectomia,
deve ser indicado quando há resistência ao tratamento clínico,
obstrução do cavum maior do que 70%, presença de deformidade
craniofacial, apneia do sono moderada a grave e otite média
secretora associada.
4.9.7 Tonsilites de repetição
As amigdalites de repetição também são bastante frequentes na faixa
etária pediátrica. A etiologia das crises repetitivas não está
totalmente elucidada, porém acredita-se que haja combinação de
fatores imunológicos e de resistência bacteriana.
O diagnóstico é clínico com base na história do paciente e no exame
físico. Vale ressaltar que, apesar de geralmente associada, a
hiperplasia amigdaliana nem sempre está presente, pois as infecções
podem acontecer em tonsilas de tamanho normal.
O tratamento durante os quadros infecciosos deve ser feito com
antibioticoterapia. Segundo o consenso da Associação Brasileira de
Otorrinolaringologia, o tratamento cirúrgico deverá ser considerado
quando ocorrerem:
1. 7 episódios no período de 1 ano;
2. 5 episódios anuais no período de 2 anos consecutivos;
3. 3 episódios anuais no período de 3 anos;
4. 2 ou mais abscessos periamigdalianos.

Cada episódio deve apresentar pelo menos uma das seguintes


características:
1. Temperatura maior ou igual a 38,3 °C;
2. Linfadenomegalia cervical;
3. Exsudato amigdaliano;
4. Cultura de secreção faríngea positiva para estreptococo beta-
hemolítico do grupo A.

4.10 RONCOS E SÍNDROME DA APNEIA


E HIPOAPNEIA OBSTRUTIVA DO SONO
1. Definição: a Síndrome da Apneia e Hipopneia Obstrutiva do Sono
(SAHOS) caracteriza-se pela repetida diminuição ou mesmo parada do
fluxo respiratório durante o período de sono, secundária a fatores
obstrutivos em topografia de faringe; está associada a sintomatologia
diurna;
2. Quadro clínico: classicamente, há roncos noturnos, com períodos
de apneia – parada do fluxo aéreo por mais que 15 segundos –, ou
hipopneia – diminuição do fluxo ou parada por 10 a 15 segundos. Além
desses sinais, os pacientes queixam-se de cefaleia matinal, sonolência
diurna e dificuldade de concentração e atenção;
3. Fisiopatologia: durante o sono, o tônus muscular diminui
gradativamente do estágio I ao sono REM, sendo menor no último
período. Nesse momento, o paciente fica mais suscetível ao
colabamento total da faringe durante a inspiração e consequente
apneia. Na ausência de ventilação, há redução progressiva na
saturação sanguínea de O2. Quando atinge níveis críticos, o centro
respiratório – bulbo – desencadeia ativação do sistema nervoso
central, gerando microdespertar com aumento do tônus muscular e
melhora do fluxo aéreo, corrigindo a queda da saturação de O2 (Figura
4.36). Consequentemente, passa a haver “quebra” cíclica do sono, ou
seja, os períodos de sono REM são entrecortados para que se
mantenha o fluxo aéreo minimamente necessário. A consequência do
sono irregular e dos microdespertares é a má qualidade do sono, com
ausência de repouso adequado, gerando a sonolência diurna e a perda
de desempenho funcional dos apneicos;

Figura 4.36 - Esquema do ciclo de apneia e microdespertares durante o sono em


pacientes com síndrome da apneia e hipopneia obstrutiva do sono

Fonte: elaborado pelos autores.

4. Fatores de risco: a SAHOS apresenta como principais fatores de


risco a obesidade, o envelhecimento, as malformações craniofaciais, a
obstrução nasal e as doenças do neurônio motor.
a) Sobrepeso: o aumento da gordura no tronco gera o colapso da
via aérea no sono por meio da compressão faríngea na posição
horizontal;
b) Obstrução nasal: apesar de ter papel secundário, o fluxo
aéreo nasal inadequado favorece a respiração oral e,
consequentemente, os roncos;
c) Pescoço curto: anatomicamente, favorece o colapso faríngeo
durante o sono;
d) Retrognatismo e alterações craniofaciais: geralmente estão
associados à posteriorização do palato mole e da língua,
facilitando o colapso faríngeo durante o sono;
e) Desproporção do volume da língua em relação à cavidade
bucal: durante o sono, principalmente em decúbito dorsal, pode
haver a “queda” da língua, com obstrução faríngea.
5. Distúrbios secundários: a apneia noturna está relacionada à
ocorrência de outras patologias, como:
a) Cardíacas: hipertensão arterial sistêmica, por aumento da
pressão intratorácica no período noturno, distúrbios de condução,
arritmias e até mesmo cor pulmonale nos casos mais graves;
b) Metabólicas: a irregularidade no ciclo sono-vigília cursa com
aumento de catecolaminas e cortisol, sendo um fator de risco
para diabetes mellitus;
c) Neurológicas e psiquiátricas: a dificuldade de repouso está
associada à ocorrência de distúrbios de ansiedade e depressão.
6. Diagnóstico: a polissonografia é fundamental ao diagnóstico e à
avaliação da gravidade da doença (Figura 4.37). Os principais
parâmetros avaliados na polissonografia são a oximetria periférica e o
Índice de Apneia e Hipopneia (IAH) por hora;
7. Tratamento: feito com base nas alterações anatômicas e nos
fatores predisponentes. As opções terapêuticas são:
a) Medicamentosas e comportamentais: perda de peso; evitar
ingesta alimentar e alcoólica antes do sono, corticoterapia nasal
para rinite;
b) Próteses: uso de aparelhos ortodônticos nos casos de
alterações craniofaciais-dentárias; CPAP nasal ou oronasal
noturno nos casos resistentes ou em casos específicos. Essa
terapêutica é muito utilizada e visa manter a pressão aérea
positiva mínima contínua na faringe, evitando o seu colapso
(Figura 4.38);
c) Cirurgia: deve ser considerada em casos de hiperplasia
adenotonsilar em crianças ou adultos, rinite resistente ao
tratamento clínico e retroposição ou malformação faríngea/palatal.

Em casos de apneia muito severa, considera-se a traqueostomia,


considerada a melhor terapêutica.
Figura 4.37 - Polissonografia

Legenda: (A) realização da polissonografia mostrando a monitorização do


eletroencefalograma, fluxo aéreo oronasal, movimentação toracoabdominal e oximetria; (B)
traçado esquemático mostrando um período de apneia.

Fonte: Ilustração Claudio Van Erven Ripinskas.

Figura 4.38 - CPAP nasal


Legenda: (A) pressão progressivamente negativa na faringe gerando seu colapso; (B) e
(C) ação da CPAP, mantendo a pressão faríngea constantemente positiva.

Fonte: Ilustração Claudio Van Erven Ripinskas.


O que é mononucleose e a
síndrome mono-like? Quais
são seus agentes
patológicos?
A mononucleose é uma doença de caráter sistêmico cujo
quadro clínico é caracterizado pela tríade: dor faríngea,
febre e adenomegalia cervical, mas também se observa
hepatoesplenomegalia, prostração e queda do estado geral.
Laboratorialmente, vemos linfocitose, com 10% de atipia e
aumento de TGO e TGP; o agente etiológico é o EBV e os
sintomas podem durar até 4 semanas.
A síndrome mono-like se caracteriza por sintomas clínicos
semelhantes: dor faríngea, febre, adenomegalia, podendo
também haver comprometimento linforreticular e
alteração enzimática. No entanto, geralmente o quadro
tende a ser menos intenso e seus principais agentes são:
citomegalovírus, toxoplasmose, HIV, herpes, sífilis e
Trypanosoma cruzi.
Uma paciente de 4 anos e
um paciente de 14 anos,
ambos apresentam
obstrução nasal e rinorreia
espessa em fossa nasal
direita há 30 dias. Podemos
sugerir o mesmo
diagnóstico para ambos os
pacientes?

5.1 RINOSSINUSITES
As rinossinusites correspondem ao processo inflamatório da mucosa
nasossinusal e podem ser classificadas, de acordo com a sua
etiologia, em infecciosas – viral, bacteriana ou fúngica –, ou não
infecciosas – alérgica, vasomotora ou química; e, pelo tempo de
evolução, em agudas, até 4 semanas, subagudas, 4 a 12 semanas, ou
crônicas, mais de 12 semanas. Os conceitos aqui discutidos tomam
por referência o Consenso “Rinossinusites: evidências e
experiências” de 2015 da Associação Brasileira de
Otorrinolaringologia.
5.2 RINOSSINUSITE AGUDA
1. Definição: a rinossinusite aguda (RSA) corresponde ao processo
inflamatório agudo da mucosa nasal e dos seios paranasais com até
12 semanas;
2. Fisiopatologia: as secreções produzidas nos seios da face fluem
pela ação ciliar através dos óstios e drenam para a cavidade nasal.
Distúrbios que interferem nessa fisiologia favorecem a ocorrência de
sinusites agudas;
3. Fatores associados: os principais fatores associados à RSA são:
a) Ambiente: exposição a umidade, poluição, produtos químicos;
b) Anatômicos: principalmente obstrução do óstio de drenagem
por edema na mucosa ou presença de lesões oclusivas locais
(pólipos, tumores, desvios septais, corpo estranho); hiperplasia
adenoidiana;
c) Lesão ciliar: ocorre por perda de células epiteliais ciliadas;
toxinas virais, bacterianas ou ambientais; mediadores
inflamatórios; pH baixo, tabagismo e drogas;
d) Discinesia ciliar primária: distúrbio genético autossômico
recessivo caracterizado por discinesia ciliar e cursa com
rinossinusite crônica (RSC) e episódios de agudização;
e) Doenças crônicas: diabetes, asma e doenças
cardiovasculares.
4. Classificação:
a) RSA viral ou resfriado comum: condição autolimitada com
duração dos sintomas menor que 10 dias;
b) RSA pós-viral: definida quando há piora dos sintomas após 5
dias de doença ou quando os sintomas persistem por mais de 10
dias de doença;
c) RSA bacteriana (RSAB): uma pequena porcentagem dos
pacientes com RSA pós-viral (0,5 a 2%) pode evoluir com RSAB.
5. Agentes etiológicos:
a) Virais: as infecções virais do trato respiratório superior são o
fator de risco mais importante para o desenvolvimento de sinusite
bacteriana aguda;
b) Bacterianos: 2 terços dos pacientes com sinusite aguda
apresentam pelo menos 1 espécie bacteriana patogênica em
aspirados sinusais, enquanto 1 terço dos pacientes tem múltiplas
espécies bacterianas predominantes. Os patógenos mais comuns
são, em ordem decrescente, Streptococcus pneumoniae,
Haemophilus influenzae e Moraxella catarrhalis;
c) Outros: Streptococcus pyogenes, Staphylococcus aureus e
anaeróbios são encontrados em menos de 10% dos pacientes.
6. Quadro clínico e diagnóstico: o Consenso afirma que “a RSA
pode ser diagnosticada com base apenas nos sintomas, sem exame
otorrinolaringológico detalhado e/ou exames de imagem”. O
diagnóstico de sinusite aguda é feito com base na presença de 2 ou
mais dos seguintes sintomas:
a) Obstrução, congestão nasal;
b) Secreção nasal, rinorreia anterior ou posterior – mais
frequentemente, mas não obrigatoriamente purulenta;
c) Dor, pressão facial, cefaleia;
d) Distúrbio do olfato.
7. A possibilidade de se estar diante de uma RSAB é maior na
presença de 3 ou mais dos sinais e sintomas a seguir:
a) Secreção nasal, presença de pus na cavidade nasal com
predomínio unilateral;
b) Dor local com predomínio unilateral;
c) Febre > 38 °C;
d) Deterioração, piora dos sintomas após o período inicial de
doença;
e) Elevação da velocidade de hemossedimentação (VHS) e
proteína C reativa (PCR).
8. No exame físico, geralmente se observa:
a) Rinoscopia anterior: presença de secreção purulenta uni ou
bilateral;
b) À oroscopia observa-se secreção mucopurulenta pela
orofaringe (sinal “da vela”);
c) Febre.
9. Exames complementares:
a) PCR ou VHS: quando elevadas estão associadas a RSAB,
podendo ser usadas como critério diagnóstico;
b) Endoscopia nasal: não é um exame obrigatório, porém fornece
informações importantes sobre a condição atual da doença
(Figuras 5.1 e 5.2);
c) Tomografia Computadorizada (TC) de face: deve ser realizada
em situações específicas, principalmente em suspeita de
complicações;
d) Radiografia de face: não deve ser solicitado.
10. Tratamento: O tratamento das sinusites leves ou quadros iniciais
deve ser feito com sintomáticos corticoides nasais e lavagem nasal
com solução fisiológica, uma que vez a maioria dos quadros é de
etiologia viral e autolimitada.
a) Antibióticos: estão indicados nos casos de RSAB moderada ou
grave e nos casos de RSAB leve que não apresentam melhora
com o tratamento inicial;
b) A amoxicilina é o antibiótico de primeira escolha, sendo a
claritromicina uma opção. Se houver indícios de resistência ao
tratamento ou história de tratamento prévio, pode-se optar por
amoxicilina + clavulanato, cefalosporinas de segunda ou de
terceira geração ou quinolonas. Outros tratamentos adjuvantes,
além do corticoide nasal, serão explicitados na sequência;
c) Corticoides orais: apresentam eficácia no tratamento
sintomático;
d) Lavagem nasal com soluções salinas isotônicas ou
hipertônicas: apresentam benefício controverso no tratamento da
RSA, mas devem ser indicados;
e) Descongestionantes orais e tópicos: podem ser utilizados como
medicações sintomáticas, mas não estatisticamente mudam a
evolução da doença.

Nos casos de rinorreia purulenta unilateral em crianças, deve-se


considerar sempre a hipótese da presença de corpo estranho, que
deve ser investigada, se possível, com nasofibrolaringoscopia, já que
alguns materiais são de difícil visualização por exames radiológicos.
Figura 5.1 - Nasofibroscopia de secreção em cavum na rinossinusite aguda
Os esquemas de tratamento de RSA são definidos de acordo com a
situação do paciente:
1. Sem tratamento prévio ou alergias medicamentosas:
a) Amoxicilina;
b) Amoxicilina + inibidores de betalactamase em casos intensos;
c) Claritromicina;
d) Axetilcefuroxima;
e) Período: 10 a 14 dias;
f) Terapias adjuvantes: lavagem nasal com solução isotônica,
analgésicos, corticoterapia oral e tópica.
2. Com alergia a betalactâmicos:
a) Claritromicina;
b) Quinolonas – levofloxacino, gatifloxacino;
c) Sulfametoxazol-trimetoprima.
3. Sem resposta após 72 horas de antibioticoterapia
a) Amoxicilina + inibidores de betalactamase;
b) Axetilcefuroxima;
c) Quinolonas – levofloxacino, gatifloxacino.

Figura 5.2 - Endoscopia da fossa nasal esquerda evidenciando secreção purulenta no


meato médio, típico de quadro de sinusite
5.2.1 Complicações das rinossinusites agudas
1. Definição: são complicações decorrentes de evolução da sinusite
aguda, principalmente de seio etmoidal. Seus principais subtipos são
as orbitárias, as intracranianas e as osteomielites;
2. Fisiopatologia: as infecções podem se disseminar por 3 vias
principais:
a) Contiguidade e erosão óssea: através das paredes dos
seios, principalmente da lâmina papirácea;
b) Hematogênica: através do plexo venoso subepitelial;
c) Linfática: através do plano subperiosteal ou dos linfáticos
perivasculares.

5.2.1.1 Orbitárias
É a principal complicação das rinossinusites agudas, favorecida pela
fina lâmina óssea que separa o seio etmoidal da órbita.
Anatomicamente, há o periósteo interno da órbita que,
anteriormente, na região palpebral, é denominado septo palpebral.
Esse septo tem especial importância na classificação das
complicações, sendo aquelas em que a infecção se estende para a
região palpebral anterior ao septo denominadas complicações pré-
septais e são as menos severas, enquanto as complicações que
atingem a órbita ou seio cavernoso tendem a ser mais graves. Tal
avaliação é feita com TC de seios da face
Quadro 5.1 - Classificação de Chandler para complicações da rinossinusite

1. Etiologia: os micro-organismos mais comuns são os mesmos das


rinossinusites;
2. Quadro clínico:
a) Flegmão palpebral: edema palpebral, eritema, dor localizada,
obstrução nasal, rinorreia, dificuldade na abertura do olho e,
eventualmente, febre. A acuidade visual e a motilidade ocular
estão preservadas;
b) Celulite e abscesso orbitário: alterações do estado geral,
exoftalmia com exoforia, exoftalmia, quemose, hiperemia
conjuntival, diminuição da motilidade ocular, acuidade visual
preservada ou diminuída.
3. Diagnóstico: leucograma evidencia leucocitose com desvio para a
esquerda. A TC revela presença de edema ou coleção purulenta na
parede medial da órbita, entre a periórbita e o osso orbitário. A
realização de TC de seios da face é mandatória em suspeitas de
complicação de sinusites.

a) Tratamento

O tratamento é realizado de acordo com o tipo da complicação:


1. Chandler I: amoxicilina + ácido clavulânico; corticoterapia oral;
2. Chandler II: penicilina cristalina + cloranfenicol ou ceftriaxona +
clindamicina; corticoterapia intravenosa;
3. Chandler III e IV: abordagem cirúrgica dos seios da face; penicilina
cristalina + cloranfenicol ou ceftriaxona + clindamicina; corticoterapia
intravenosa;
4. Chandler V: abordagem craniana caso haja abscesso; fibrinolíticos;
penicilina cristalina + cloranfenicol ou ceftriaxona + clindamicina;
corticoterapia intravenosa.

Figura 5.3 - Celulite pré-septal à direita (Chandler I)


Fonte: Orbital complications of acute rhinosinusitis: A new classification, 2007.

Figura 5.4 - Sinusite complicada (Chandler III)


Nota: observar o velamento da região dos etmoides, com acometimento da órbita.
Legenda: (A) formação de abscesso; (B) flegmão formado.
Fonte: adaptado de Orbital complications of acute rhinosinusitis: A new classification, 2007
Figura 5.5 - Complicação intracoanal (Chandler IV)

Nota: observar a formação de gás na órbita esquerda.


Fonte: Digital Journal of Ophthalmology.

Figura 5.6 - Chandler V, com trombose de seio cavernoso


Nota: observar o velamento de células etmoidais e esfenoide (SpS) e a trombose no seio
cavernoso (CS).

5.2.1.2 Complicações intracranianas

Incluem os abscessos intraparenquimatoso, epidural e subdural,


além de meningites, que ocorrem por extensão do processo sinusal
para o sistema nervoso central por via hematogênica ou
contiguidade.
Entre os seios que podem gerar tal infecção, o frontal é o mais
associado às complicações intracranianas, seguido pelo etmoide,
esfenoide e maxilar.
O quadro clínico nem sempre é típico, mas em geral o paciente se
queixa de cefaleia intensa. Quando há empiema subdural, os sinais
focais normalmente ocorrem quando a coleção é grande e provoca
efeito de massa.
A antibioticoterapia deve ser empregada com um antibiótico que
atravesse a barreira hematoencefálica, sendo a ceftriaxona o mais
comumente prescrito.
A abordagem cirúrgica deve ser feita quando há coleção purulenta
evidente.
5.2.1.3 Osteomielites

1. Osteomielite frontal: o quadro pode ser de curta ou longa duração.


Um edema mole do osso frontal – tumor de Pott (Figura 5.7) –
evidencia um abscesso subperiostal;
2. Osteomielite maxilar: geralmente é secundária à infecção dentária;
3. Tratamento: inicialmente consiste na drenagem do abscesso e
debridamento cirúrgico. A antibioticoterapia baseia-se no Gram e na
cultura do exsudato. A antibioticoterapia intravenosa deve ser mantida
por pelo menos 21 dias no pós-operatório e por via oral,
posteriormente completando de 4 a 8 semanas.

Figura 5.7 - Tumor de Pott: osteomielite de osso frontal secundário a sinusite, com
fistulização e extravasamento de conteúdo purulento para espaço subperiosteal

Fonte: adaptado do site Otolaryngology Houston.


5.3 RINOSSINUSITE CRÔNICA
1. Definição: alguns autores definem a RSC como sinais e sintomas
de rinossinusite por mais de 4 semanas, enquanto outros só a
consideram quando os sintomas passam de 12 semanas. De qualquer
maneira, é muito importante que, nesses casos, sejam investigados
fatores associados que possam contribuir para a manutenção do
processo infeccioso;
2. Classificação: a RSC pode ser dividida em 2 principais entidades –
a RSC sem polipose nasossinusal e a RSC com polipose
nasossinusal. Atualmente, existem evidências que sugerem que essas
2 entidades apresentam mecanismos fisiopatogênicos distintos;
3. Quadro clínico: sintomas persistentes por mais de 12 semanas. As
principais queixas são: obstrução nasal, rinorreia anterior ou posterior,
hiposmia ou anosmia, dor ou pressão facial e tosse. Ao exame físico, a
rinoscopia tem valor limitado, exceto nos casos de polipose. A
presença de secreção mucocatarral em orofaringe pode ser
visualizada;
4. Exames complementares:
a) Endoscopia nasal: usada para detecção de alterações
anatômicas que possam levar à obstrução crônica dos óstios dos
seios paranasais, como hipertrofia de conchas nasais, desvio
septal, polipose nasossinusal etc.;
b) TC de seios da face: é o exame padrão-ouro, observa-se
opacificação das cavidades paranasais e do complexo
ostiomeatal;
5. Fatores predisponentes e doenças associadas: doenças
sistêmicas e possíveis alterações imunológicas devem ser
investigadas;
a) Exposição a tabaco, dióxido de enxofre, poluentes do ar e
tabagismo;
b) Alterações anatômicas, como desvio septal, concha média
bolhosa, desvios do processo uncinado, célula de Haller;
c) Infecções odontogênicas, como fístula oroantral, doenças
periodontais e abscesso periapical;
d) Discinesia ciliar primária, doença autossômica recessiva na
qual os cílios se encontram imóveis ou com padrão de
movimentação alterado, provocando falha no transporte do muco
da via aérea;
e) Fibrose cística, principalmente em crianças com Polipose Nasal
(PN);
f) A associação de intolerância ao ácido acetilsalicílico, asma e
PN é frequente, também conhecida como síndrome de Widal.
6. Tratamento: nos casos de sinusite de etiologia bacteriana, é feito
inicialmente com antibióticos orais. Lavagem nasal com solução
isotônica, analgésicos, corticosteroides nasais e antileucotrienos
podem ser associados. Nos casos refratários, deve-se considerar a
abordagem cirúrgica com a ampliação dos óstios dos seios. As opções
de antibióticos nas RSCs são:
a) Amoxicilina + inibidores de betalactamase;
b) Axetilcefuroxima;
c) Quinolonas – levofloxacino, gatifloxacino;
d) Período de 14 a 21 dias.

5.3.1 Subclassificações das rinossinusites crônicas


5.3.1.1 Sem pólipos

1. Definição: trata-se da forma mais comum de RSC. Caracteriza-se


por um processo infeccioso crônico dos seios paranasais e tem relação
com processos obstrutivos de meato médio, como desvios septais,
alergias e rinites, asma, alterações no transporte mucociliar – como na
discinesia ciliar primária ou na fibrose cística –, malformações, traumas
ou sem fatores evidentes;
2. Quadro clínico: estão presentes os sintomas clássicos da doença,
como obstrução nasal, cefaleia em peso, rinorreia amarelada e
descarga retronasal contínua por períodos maiores do que 12
semanas;
3. Diagnóstico: baseia-se na história clínica, na endoscopia nasal e
na tomografia;
4. Tratamento: é feito inicialmente com antibioticoterapia oral por 14 a
21 dias, associada a corticoterapia oral e tópica nasal. Em casos
resistentes ao tratamento, partimos para a abordagem cirúrgica, com a
abordagem dos óstios maxilares e sua ampliação (Figura 5.8).

Figura 5.8 - Representação esquemática de etmoidectomia direita e antrostomia maxilar


Fonte: adaptado de American Family Physician.

5.3.1.2 Com polipose nasal

1. Definição: a PN é definida pela presença de pólipos eosinofílicos na


cavidade nasal, principalmente na região do meato médio. A etiologia
ainda não está totalmente elucidada;
2. Fatores relacionados: asma, alergia, intolerância ao ácido
acetilsalicílico e doenças sistêmicas, principalmente fibrose cística;
3. Diagnóstico: baseia-se no quadro clínico e na endoscopia
nasossinusal, com visualização dos pólipos bilateral e difusamente. A
TC de seios paranasais permite avaliar as áreas acometidas por
pólipos;
4. Tratamento: deve ser feito inicialmente de forma medicamentosa,
com acompanhamento frequente e uso de corticoides nasais em doses
altas e limpeza com solução isotônica frequente. Nos casos refratários
ao tratamento clínico, pode-se optar pela polipectomia cirúrgica e
ampliação dos óstios de drenagem e limpeza dos seios da face,
seguida de corticoterapia nasal contínua. No entanto, a doença
apresenta grande potencial de recidiva, em geral após 3 a 5 anos. A
decisão sobre novas abordagens deve considerar os sintomas e o
potencial risco de complicações cirúrgicas.

5.3.1.3 Fibrose cística


1. Definição: trata-se de uma patologia de origem genética associada
a disfunção no transporte iônico celular; manifesta-se sistemicamente,
sobretudo no pâncreas, pulmão, intestino e nas vias aéreas. Nos seios
da face, manifesta-se por quadro de RSC de difícil controle, podendo
ou não estar associada a pólipos nasais (Figura 5.9);
2. Diagnóstico: em adultos baseia-se no teste do suor e em
rastreamentos genéticos. A manifestação nasossinusal é baseada nos
sintomas referidos associados a endoscopia nasal e tomografia. O
tratamento é feito com uso de antibióticos orais nas agudizações e
abordagem cirúrgica nos casos mais intensos.

Figura 5.9 - Quadro de sinusite crônica por fibrose cística

Fonte: site Stanford Medicine.

5.3.1.4 Discinesia ciliar primária


1. Definição: é uma doença autossômica recessiva na qual os cílios
se encontram imóveis ou com padrão de movimentação alterado,
provocando falha no transporte mucoso. Cursa com os sintomas
típicos de sinusite crônica e bronquiectasia;
2. Diagnóstico: deve ser suspeitado em crianças com asma atípica,
bronquiectasia, tosse crônica produtiva, secreção nasal espessa
perene e otite média crônica grave. Os testes diagnósticos incluem
dosagem do óxido nítrico nasal, cujo resultado é menor que os valores
de controle, e teste da sacarina. Testes mais específicos incluem o
exame dos cílios por microscopia eletrônica. O diagnóstico da sinusite
crônica também é fundamentado na TC, que mostra hipoplasia maxilar,
velamento pansinusal e espessamento de paredes laterais de seios
maxilares;
3. Tratamento: o protocolo é semelhante ao da fibrose cística, com
consultas periódicas, lavagem nasal com solução isotônica e
abordagem cirúrgica em casos refratários.

5.3.1.5 Pólipos antrocoanais ou de Killian

1. Definição: o pólipo antrocoanal, ou de Killian, é uma lesão polipoide


solitária benigna que acomete principalmente crianças e adultos
jovens. Origina-se na mucosa do antro do seio maxilar e desenvolve-
se através do óstio do seio maxilar para a cavidade nasal e em direção
à cóana e à parte posterior da nasofaringe, podendo estender-se até a
orofaringe. Nota-se que o óstio do seio maxilar está aumentado em
diâmetro. A etiologia não foi elucidada, não há relação com alergia ou
mesmo com a PN;
2. Quadro clínico: manifesta-se clinicamente por obstrução nasal,
geralmente unilateral, secreção mucosa ou mucopurulenta. Ao exame
físico, observa-se pólipo único saindo pelo meato médio e projetando-
se na cóana, podendo preencher parcial ou totalmente a fossa nasal e
a rinofaringe (Figura 5.10 - A);
3. Exames complementares: TC com opacificação e preenchimento
homogêneo da cavidade do seio maxilar, da cavidade nasal ipsilateral
e, em alguns casos, da rinofaringe (Figura 5.10 - B);
4. Tratamento: exclusivamente cirúrgico, com exérese da lesão e
principalmente do seu pedúnculo de inserção. Recidivas são
infrequentes e devem ser reabordadas quando sintomáticas;
5. Diagnóstico diferencial: quanto a diferenças entre polipose nasal e
pólipo de Killian, a primeira é bilateral, recorrente e associada à tríade
de Widal, e o último é unilateral, não recorrente e sem associação a
alergias ou asma.

Figura 5.10 - Pólipo de Killian


Legenda: (A) rinoscopia da fossa nasal direita, mostrando pólipo antrocoanal implantado
no meato médio; (B) tomografia computadorizada coronal do mesmo paciente, mostrando
alargamento do óstio maxilar e velamento unilateral dos seios maxilar e etmoidal à direita.

5.3.1.6 Mucoceles

1. Definição: lesão de aspecto cístico por retenção de secreção


mucosa, em geral decorrente de obstrução da drenagem do seio:
processo inflamatório, trauma ou efeito de massa. Apresenta
comportamento lítico e de remodelamento das paredes ósseas.
Quando a secreção é purulenta, denomina-se piocele ou mucopiocele.
As mucoceles são classificadas em 2 tipos:
a) Primária: composta por goblet cells de crescimento lento,
gerando secreção líquida. Há expansão e remodelamento da
parede do seio;
b) Secundária: decorre da obstrução do óstio de drenagem do
seio. Tal obstrução causa retenção de secreção e, muitas vezes,
infecção secundária.
2. Quadro clínico: é dependente de sua extensão e localização. Em
geral cefaleia frontal em pressão, acometimentos oculares, como
exoftalmia, diplopia e edema de pálpebra, obstrução nasal. O exame
físico evidencia edema periorbitário, proptose, aumento da
sensibilidade frontal ou frontoetmoidal (Figura 5.11);

Figura 5.11 - Mucocele frontoetmoidal direita, com erosão da tábua anterior do frontal e
herniação externa
Fonte: site World Articles in Ear, Nose and Throat.

3. Diagnóstico radiológico: a TC é considerada o método


complementar de escolha. A mucocele aparece, usualmente, como
uma opacidade do seio, isodensa, discretamente hiperdensa. O
arcabouço ósseo adjacente encontra-se remodelado com áreas de
espessamento e erosão. Pode-se notar, ainda, sua herniação para
estruturas próximas, nos pontos de maior fragilidade (Figura 5.12); a
RNM pode diferenciar o conteúdo cístico do purulento;
4. Tratamento: abordagem cirúrgica visando restabelecer drenagem
sinusal.

Figura 5.12 - Tomografia de mucocele frontoetmoidal esquerda, com proptose do globo


ocular esquerdo
Fonte: Frontoethmoidal mucoceles: a case report and literature review, 2004.

Figura 5.13 - Mucocele do seio maxilar esquerdo

Legenda: (1) mucocele; (2) paredes mediais do seio maxilar; (3) parede lateral do seio
maxilar; (4) arcada dentária.
Fonte: site World Articles in Ear, Nose and Throat.

5.3.1.7 Rinossinusites fúngicas

a) Rinossinusite fúngica alérgica

1. Definição: trata-se de uma RSC associada à presença de fungos,


como Aspergillus e Bipolaris. A maioria dos autores reconhece que a
rinossinusite fúngica alérgica (RSFA) é uma enfermidade mais de
caráter alérgico do que infeccioso. Representa uma resposta imune do
tipo I pelos altos níveis séricos de IgE total e específica para fungo;
2. Quadro clínico: em cerca de 75% dos casos há queixas de
rinorreia purulenta, ou crostas, de coloração marrom, chamada mucina
alérgica. O quadro clínico caracteriza-se por sinais e sintomas de RSC,
atopia, PN e deformidade facial progressiva sem invasão fúngica da
mucosa nasal;
3. Diagnóstico: a identificação da mucina alérgica é essencial para o
diagnóstico. Esse material tem consistência mucinosa, é espesso, de
coloração verde, amarela ou marrom, e pode ser aspirado das fossas
nasais durante endoscopia nasal ou cirurgias (Figura 5.14).
Histologicamente, são encontradas hifas fúngicas, sem esporos, que
não invadem a mucosa. Invasão fúngica, formação de granuloma ou
necrose tecidual não são características presentes no exame
histopatológico da RSFA. As hifas fúngicas na RSFA encontram-se
dispersas na mucina, muitas vezes difíceis de serem encontradas.
Esses indivíduos apresentam níveis elevados de IgE e teste cutâneo
fortemente positivo para fungos. Na TC sem contraste, os seguintes
sinais estão presentes nesses pacientes: áreas focais de
hiperatenuação, osteíte reativa, obliteração do complexo osteomeatal,
erosão óssea principalmente na parede lateral nasal e remodelação
das paredes ósseas com expansão da cavidade sinusal (Figura 5.15);
4. Tratamento: inclui cirurgia endoscópica funcional dos seios
paranasais com o uso de corticoides local e sistêmico; imunoterapia
também pode ser indicada.

Figura 5.14 - Aspiração de mucina de fossa nasal durante o procedimento cirúrgico


Nota: observe o aspecto escurecido e espesso da secreção.
Fonte: Medscape.

Figura 5.15 - Obliteração sinusal bilateral com massa heterogênea com áreas de
radiopacidade

Fonte: Radiology Assistant.


b) Bola fúngica

1. Definição: acúmulo não invasivo de uma densa conglomeração de


hifas fúngicas em um seio paranasal. É caracterizada por uma massa
de restos fúngicos endurecidos e muco, com crescimento progressivo
na cavidade sinusal, sem envolvimento da mucosa subjacente,
principalmente em seios maxilares e esfenoides, onde os fungos
encontram condições ótimas para crescimento. Os indivíduos são
tipicamente imunocompetentes. A fisiopatologia ainda não é totalmente
compreendida;
2. Quadro clínico: pode ser assintomática ou simular quadros de
RSC. Sintomas frequentes incluem obstrução nasal, secreção nasal
purulenta, cacosmia e dor facial. O diagnóstico deve ser considerado
nos casos de sinusites recorrentes ou refratárias aos tratamentos com
antibióticos, principalmente quando unilaterais. O diagnóstico
radiológico é feito por TC, que mostra opacificação completa ou parcial
do seio acometido. Microcalcificações ou manchas com densidade
metálica também são encontradas;
3. Tratamento: cirurgia, com abertura dos seios e remoção das hifas.
As drogas antifúngicas, tópicas ou sistêmicas não estão indicadas.

c) Mucormicose

1. Definição: sinusite fúngica de caráter invasivo da mucosa e


estruturas ósseas adjacentes que acomete, na grande maioria das
vezes, imunodeprimidos, tendo como principais agentes etiológicos os
fungos Mucor e Aspergillus. As imunodeficiências são fatores
predisponentes ao aparecimento da mucormicose;
2. Quadro clínico: os principais sintomas são febre, úlcera nasal ou
necrose, edema periorbital ou facial, diminuição da acuidade visual,
oftalmoplegia e cefaleia;
3. Diagnóstico: a TC de seios da face pode mostrar invasão tecidual e
necrose, erosão óssea e trombose de seio cavernoso. Endoscopia
nasal evidencia áreas de granulação e necrose;
4. Tratamento: é feito com debridamento cirúrgico, cujo principal
objetivo é remover o máximo de tecido desvitalizado possível, além de
estabelecer adequada drenagem sinusal, associada à administração
de anfotericina B.

Quadro 5.2 - Rinossinusites fúngicas


5.4 RINOSSINUSITES NÃO
INFECCIOSAS – RINITES
1. Definição: trata-se de processos inflamatórios da mucosa de
revestimento das fossas nasais desencadeados por agentes inalatórios
ambientais;
2. Fisiopatologia: mudanças de temperatura, baixa umidade, produtos
químicos ou poluentes e alérgenos presentes no ar inalado geram
resposta inflamatória na mucosa nasal. O processo inflamatório
crônico acarreta edema e consequente hipertrofia persistente da
mucosa, principalmente dos cornetos inferiores, cursando com
obstrução nasal persistente;
3. Quadro clínico: caracteriza-se por congestão nasal (Figura 5.17 -
B), rinorreia do tipo aquosa, prurido nasal e espirros. Também pode
estar presente prurido ocular nas orelhas e na faringe. Esses sintomas
podem ser perenes ou transitórios e geralmente têm relação com
exposição prévia a tais irritantes nasais;
4. Diagnóstico: a rinite alérgica caracteriza-se, imunologicamente, por
uma reação tipo 1 de Gell-Coombs e é mediada por IgE específica, ou
seja, o correto diagnóstico depende de positividade no teste alérgico.
Clinicamente, o mais utilizado é o RAST (radioallergosorbent)
sanguíneo, podendo ser usado também o prick test (teste cutâneo). A
citologia nasal pode demonstrar eosinofilia. No sangue periférico,
observa-se eosinofilia, bem como a elevação de IgE;
5. Classificação: segundo recomendação da iniciativa Allergic Rhinitis
and Its Impact on Asthma e da Organização Mundial da Saúde, a
classificação da rinite alérgica deve levar em consideração a duração,
intermitente ou persistente, e a gravidade dos sintomas, incluindo
aspectos de qualidade de vida (Figura 5.16);
6. Tratamento: fundamenta-se na gravidade do quadro, de acordo
com o Quadro 5.3.

Figura 5.16 - Classificação da rinite alérgica


Fonte: adaptado de Allergic Rhinitis and Its Impact on Asthma, 2016.

A rinite alérgica é a mais comum, a irritativa é a


segunda mais comum e há, ainda, a gestacional,
a induzida por exercício e a do idoso.

Em relação ao tratamento, deve ser feito seguindo a graduação de


gravidade da doença, de acordo com o Quadro 5.3
Quadro 5.3 - Tratamento

Com relação aos corticoides nasais, deve-se evitá-los em menores


de 2 anos. Geralmente, opta-se pelo uso dos cromoglicatos.
A imunoterapia pode ser bastante eficaz, desde que seja realizada
por profissional capacitado.
Quanto aos casos em que há grande refratariedade ao tratamento
clínico, pode-se considerar o tratamento cirúrgico, com a exérese do
excesso mucoso de conchas inferiores e eventualmente, da concha
média, e a correção de ocasionais desvios septais que contribuam
para a diminuição do fluxo nasal.
Figura 5.17 - Rinoscopia
Legenda: (A) fossa nasal esquerda mostrando mucosa normal; (B) em paciente com rinite
– notar o edema, a palidez e o estado hipersecretivo da mucosa.

5.5 DESVIOS SEPTAIS


O septo nasal é formado, anatomicamente, por uma cartilagem em
sua porção anterior e pela fusão dos ossos vômer, maxilar e etmoidal
na sua porção.
Na região da fusão condro-óssea pode haver, durante o crescimento
nasofacial, um “descompasso” de crescimento, com consequente
desvio do septo nasal para uma das fossas (Figura 5.18 - B). Esse
desvio pode, também, ser secundário a traumas.
Figura 5.18 - Principais tipos de desvio septal
Legenda: (A) cartilaginoso; (B) e (C) na junção condro-óssea; (D) misto.
Fonte: Ilustração Claudio Van Erven Ripinskas.

Em alguns indivíduos, o grau de desvio pode ser intenso a ponto de


gerar obstrução nasal total ou parcial. Nesses, a avaliação da
obstrução nasal deve considerar outros fatores associados, sendo a
rinite, com hiperplasia dos cornetos inferiores, a mais comumente
relacionada.
O diagnóstico baseia-se na queixa clínica, além das avaliações
tomográfica e ou nasofibroscópica (Figuras 5.19 e 5.20).
Figura 5.19 - Tomografia computadorizada coronal mostrando desvio septal, indicado por
seta, para a direita

Figura 5.20 - Endoscopia da fossa nasal direita mostrando o contato do septo nasal com o
corneto inferior
O tratamento deve levar em consideração a presença de sintomas
obstrutivos relacionados ao desvio septal. A simples existência de
desvio septal não indica a cirurgia. Esse procedimento é executado
quando o desvio contribui significativamente para a ocorrência de
obstrução nasal.
5.6 CISTO DE RETENÇÃO MUCOSO
Trata-se de obstrução inflamatória de glândulas seromucinosas,
causando retenção de muco, que afeta principalmente o seio maxilar
(Figura 5.21).
Acomete de 10 a 15% da população geral e seu diagnóstico
geralmente é feito por meio de radiografia de face ou radiografias
panorâmicas de mandíbula. Normalmente, não possui
sintomatologia, devendo-se apenas fazer seguimento. Caso haja
sintomatologia, realiza-se exérese.
Figura 5.21 - Tomografias computadorizadas coronais de seios da face mostrando cistos
maxilares, indicados por setas
5.7 EPISTAXES
A epistaxe é definida como o sangramento proveniente da mucosa
nasal. Estima-se que cerca de 60% da população adulta já
apresentou ao menos 1 episódio, em sua maioria autolimitado. Mais
de 90% decorrem de sangramento na região septal anterior –
Kiesselbach ou Little.
5.7.1 Fatores etiológicos
1. Sistêmicos: hipertensão arterial é principal causa de epistaxe
severa, além de coagulopatias e doenças hematológicas. O uso de
medicamentos anticoagulantes e antiagregantes plaquetários também
pode ocasionar epistaxe;
2. Locais: traumas como fraturas nasais ou manipulação digital,
infecções de vias aéreas superiores, inalação de ar frio e seco,
quadros alérgicos nasais, introdução de corpos estranhos na fossa
nasal, perfuração ou desvio septal. Em pacientes idosos a fragilidade
dos pequenos vasos sanguíneos pode facilitar seu rompimento em
picos hipertensivos.

Nos casos de epistaxe recorrente, é extremamente importante a


investigação com nasofibrolaringoscopia para descartar a presença
de tumores, em especial o nasoangiofibroma nos jovens e os
carcinomas nos pacientes acima dos 60 anos. Também deve-se
investigar coagulopatias.
O tratamento das epistaxes recorrentes ou severas pode envolver
cauterização química ou elétrica, tamponamento anterior e ou
posterior, ou ligadura videoendoscópica da artéria esfenopalatina ou
das artérias etmoidais anterior e posterior (Figura 5.22).
Figura 5.22 - Tratamento das epistaxes
Fonte: elaborado pelos autores.

5.8 FRATURAS NASAIS


As fraturas da pirâmide nasal são muito frequentes, e
aproximadamente 39% das fraturas maxilofaciais são nasais. O pico
de incidência ocorre dos 15 aos 25 anos, com predominância no sexo
masculino de 2:1.
5.8.1 Fisiopatologia
Os tipos de fraturas nasais e suas sequelas dependem de alguns
fatores:
1. Idade do paciente – flexibilidade das estruturas;
2. Intensidade e direção da força aplicada;
3. Natureza do instrumento causador do trauma.

Os impactos laterais provocam mais fraturas do que os frontais.


5.8.2 Diagnóstico
O diagnóstico baseia-se na história de trauma nasal associado a
mudança na aparência ou obstrução nasal, epistaxe, dor e edema –
geralmente de evolução rápida.
Ao exame físico, observa-se laceração ou ruptura da mucosa nasal.
Equimose e hematoma intenso sugerem fratura. À palpação, nota-se
instabilidade dos ossos nasais.
A pesquisa radiológica é feita com incidência em perfil para
visualização do osso próprio do nariz e de Waters para avaliação do
septo ósseo, da pirâmide dorsal e das paredes nasais laterais. TC
deve ser considerada em casos especiais e de dúvida diagnóstica
(Figura 5.23).
5.8.3 Tratamento
As indicações de tratamento são para fraturas que apresentem
desvios e ou instabilidade. A maioria dos autores considera que a
redução da fratura nasal deve ser feita nos primeiros dias após o
trauma e, se possível, em até 15 dias.
Figura 5.23 - Esquema de fraturas nasais
Fonte: adaptado de Management of Nasal Fractures, 2015.

5.9 TUMORES DE NARIZ E SEIOS DA


FACE
Apresentam sintomatologia inespecífica, pois crescem em cavidade,
permanecendo silenciosos até infiltrarem alguma estrutura vizinha,
como um par craniano, a órbita, a base do crânio, causando
sintomas. Geralmente apresentam-se como quadro de sinusopatia
crônica rebelde a tratamento.
Os principais sintomas são a obstrução nasal unilateral,
sangramento, rinorreia purulenta e cacosmia. O exame endoscópico
é fundamental para a caracterização da massa tumoral e das
alterações decorrentes da sua presença. A TC permite avaliar tanto
estruturas ósseas, como partes moles e tumorais, quanto a sua
extensão e invasão de estruturas adjacentes, sendo geralmente o
método de escolha para diagnóstico e estadiamento dos tumores.
5.9.1 Tumores benignos
5.9.1.1 Papilomas de cavidade nasal – papiloma schneideriano

1. Definição: tumorações benignas, derivadas de epitélio


schneideriano, únicas ou múltiplas, comumente encontradas no nariz e
nos seios paranasais, consistindo em massa fibrovascular coberta por
epitélio sem atipia celular;
2. Classificação: são 3 categorias distintas – simples fungiforme,
simples cilíndrica ou invertido, ou uma combinação destas. Embora
sejam reconhecidas como clinicamente diferentes, todas apresentam
histologias semelhantes. Também são semelhantes às papilomatoses
recorrentes encontradas no trato respiratório;
3. Etiologia: os subtipos 6, 11, 16 e 18 de HPV têm sido implicados
em sua gênese;
4. Quadro clínico: o sintoma mais comum é a obstrução nasal
unilateral, frequentemente associada à rinorreia mucopurulenta,
epistaxe, hiposmia, dor facial e deformidades;
5. Diagnóstico: na endoscopia nasal observa-se tumor polipoide,
irregular, lobulado e que pode ser distinguido de pólipos inflamatórios
devido à sua aparência grosseira e mais firme (Figura 5.24). O
diagnóstico só é confirmado pela biópsia. TC mostra destruição óssea
e erosão da parede lateral nasal com alargamento do meato médio
(Figura 5.25);
6. Tratamento: apesar de ser considerado benigno, o papiloma
schneideriano é localmente agressivo e existe a possibilidade de
malignização. Dessa forma, o tratamento desse tumor é cirúrgico, com
remoção completa, para evitar recidivas.

Figura 5.24 - Papiloma da cavidade nasal

Nota: observar a irregularidade da lesão e ausência de translucidez.


Fonte: Pocket Atlas of the Nose and Paranasal Sinuses.
Figura 5.25 - Tomografia computadorizada de papiloma invertido à direita
Legenda: (A) processo uncinado rebatido medialmente, indicado por setas vermelhas; (B)
rebatimento medial do processo uncinado e da bula etmoidal, indicado por seta vermelha;
(C) alargamento do infundíbulo etmoidal e velamento dos seios maxilar e etmoidal.

5.9.1.2 Nasoangiofibroma

1. Definição: o nasoangiofibroma (NAFJ) é um tumor que acomete


quase exclusivamente jovens do sexo masculino. Apesar de ser
considerada neoplasia benigna, pois não apresenta invasão de tecidos
periféricos, o NAFJ não é encapsulado, cursa com destruição de
tecidos locais e tem alta taxa de recorrência. Histologicamente, é
composto por elementos miofibroblásticos e vasogênicos, portanto
trata-se de um tumor altamente vascularizado. O principal suprimento
sanguíneo é pela artéria maxilar e emerge principalmente do forame
esfenopalatino – região posterior do nariz – e pode estender-se até a
fossa craniana média;
2. Quadro clínico: pacientes na puberdade ou na pré-puberdade, do
sexo masculino, com queixa de obstrução nasal uni ou bilateral,
epistaxes recorrentes e cujo exame clínico revela a presença de tumor
na fossa nasal ou rinofaringe;
3. Diagnóstico: a avaliação da extensão tumoral é feita por meio da
TC e da RNM; a primeira para mostrar alterações ósseas, e a
segunda, para mostrar a extensão aos tecidos moles. A biópsia não é
um procedimento de rotina devido ao alto risco de sangramento;
4. Tratamento: atualmente, a cirurgia parece ser a melhor forma de
tratamento dos NAFJs. Outros métodos, como a hormonoterapia, a
radioterapia e a quimioterapia são hoje modalidades terapêuticas
usadas ocasionalmente como tratamentos complementares.

5.9.2 Tumores malignos


Os tumores malignos de nariz e seios paranasais representam cerca
de 3% dos cânceres de cabeça e pescoço, e 0,8% de todos os cânceres
humanos. A predominância é no sexo masculino.
Obstrução e sangramento nasal, dor facial e infecção nasossinusal
representam as queixas iniciais.
O tipo histológico mais frequente é o carcinoma espinocelular,
seguido dos tumores de sistema nervoso e dos sarcomas.
5.9.2.1 Carcinoma espinocelular

Os carcinomas epidermoides correspondem a mais de 80% de todos


os casos de neoplasias malignas nasossinusais. A TC e a RNM são
técnicas bem estabelecidas e úteis para avaliar a extensão do tumor
para áreas adjacentes. O tratamento geralmente é feito com
abordagem cirúrgica mantendo margens, seguida de radioterapia.
5.9.2.2 Estesioneuroblastoma

É um tumor de origem provável no epitélio olfativo. Tem pico de


incidência entre os 40 e os 70 anos. Os sintomas são inespecíficos e
resultam do crescimento tumoral.
Os principais locais de metastização são os gânglios cervicais e, em
seguida, os pulmões e os ossos.
Os tratamentos de ressecção craniofacial e radioterapia parecem
associar-se aos melhores resultados. A quimioterapia é geralmente
reservada a tumores localmente avançados, inoperáveis, recidivas
ou doença metastizada.
Quadro 5.4 - Principais tipos de tumores relacionados aos seios da face
Uma paciente de 4 anos e
um paciente de 14 anos,
ambos apresentam
obstrução nasal e rinorreia
espessa em fossa nasal
direita há 30 dias. Podemos
sugerir o mesmo
diagnóstico para ambos os
pacientes?
Quadro de obstrução unilateral em paciente do sexo
feminino, de 4 anos, sugere corpo estranho em fossa nasal.
Já o quadro de paciente do sexo masculino, adolescente,
com epistaxe, é sugestivo de tumor de fossa nasal direita,
possivelmente nasoangiofibroma. Para a confirmação dos
diagnósticos, podemos realizar nasofibroscopia em ambos
os casos. Na suspeita de tumor de fossa nasal direita, não
se deve realizar biópsia devido ao risco de sangramento.
Paciente do sexo masculino,
72 anos, trabalhador rural,
tabagista de 50 maços-ano,
etilista regular, com lesão
verrucosa hiperemiada em
borda lateral esquerda da
língua há 40 dias. Pode-se
iniciar tratamento empírico
com antibioticoterapia oral
e corticoterapia tópica?

6.1 GLÂNDULAS SALIVARES


As glândulas salivares apresentam-se em 2 grupos:
1. Glândulas salivares maiores: parótidas, submandibulares e
sublinguais (Figura 6.1);
2. Glândulas salivares menores: estimadas entre 600 e 1.000, são
pequenas, independentes e encontradas praticamente em toda a
cavidade oral, nasal, faringe, no polo superior das amígdalas –
glândulas de Weber – e nos pilares amigdalianos (Figura 6.2).
A saliva produzida pelas glândulas salivares tem funções digestiva e
bactericida e facilita a gustação, além de função imunológica e de
controle de cálcio e fósforo nos dentes.
Figura 6.1 - Observe as glândulas salivares maiores e seus ductos de drenagem

Fonte: Ilustração Claudio Van Erven Ripinskas.

Figura 6.2 - Observe a distribuição das glândulas salivares menores por toda a mucosa
palatal e faríngea
Fonte: adaptado do site earthslab.com.

6.1.1 Infecções virais


6.1.1.1 Caxumba ou parotidite epidêmica

As informações a seguir estão em acordo com as orientações do


Ministério da Saúde.
1. Definição: doença viral aguda caracterizada por febre, dor e
aumento de volume de 1 ou mais glândulas salivares maiores, com
predileção pelas parótidas e, secundariamente, pelas sublinguais ou
submandibulares;
2. Epidemiologia: ocorre primariamente no escolar e no adolescente,
com pico de incidência entre 4 e 6 anos. Adultos são pouco
acometidos;
3. Etiologia: é causada por um RNA vírus, da família Paramyxoviridae,
gênero Paramyxovirus, endêmico na comunidade;
4. Transmissão: é transmitido por perdigotos, secreção nasal e urina;
5. Período de incubação: de 2 a 3 semanas – em média, 18 dias;
6. Período de transmissibilidade: varia entre 6 e 7 dias antes das
manifestações clínicas e até 9 dias após o surgimento dos sintomas;
7. Quadro clínico: seus primeiros sintomas são febre, calafrios,
astenia, dores de cabeça e musculares ao mastigar ou engolir. Evolui
com dor e aumento da região parotídea uni ou bilateral (Figura 6.3), de
rápida progressão, com deslocamento do pavilhão auricular e
apagamento do ângulo da mandíbula. A dor piora estímulos de
alimentos que provocam contração da glândula, por exemplo,
alimentos ácidos;
8. Exame físico: as glândulas salivares acometidas apresentam-se
edemaciadas, permitindo sua palpação, que é geralmente dolorosa. A
abertura do ducto de Stensen pode estar edematosa, porém não se
encontra secreção purulenta. Cerca de 30% das infecções podem não
apresentar aumento aparente das glândulas;
9. Diagnóstico: o diagnóstico da doença é eminentemente clínico-
epidemiológico, não sendo necessária a realização de testes
laboratoriais em pacientes sem sinais de complicações. Quando
solicitados, os exames laboratoriais podem demonstrar anticorpos
contra antígenos S e V da caxumba. O vírus pode ser isolado da urina
entre 6 dias antes e 13 dias depois do aumento parotídeo. Nenhum
estudo de imagem específico é diagnóstico para a infecção da
caxumba. Estudos de imagem podem ser necessários para avaliar
certas complicações, como encefalite. A ultrassonografia escrotal deve
ser realizada quando a orquite é clinicamente suspeita;
10. Tratamento: é sintomático, com hidratação, repouso e cuidados da
dieta para minimizar a secreção salivar, e a vacinação deve ser feita
em todas as crianças a partir de 12 meses e adultos não imunizados,
em dose única. O paciente deve ser afastado de suas atividades de
trabalho ou escola por 5 dias após o início dos sintomas, que é o
período de maior transmissibilidade;
11. Outros locais de infecção viral:
a) Sistema nervoso central: o envolvimento é a complicação
extraglandular mais comum da caxumba. Sua apresentação é
mais frequente como meningite asséptica do que como uma
verdadeira encefalite. Ocorre em cerca de 10% dos pacientes
com história de parotidite e tem bom prognóstico, geralmente com
recuperação completa;
b) Gastrintestinal: outra manifestação clínica de caxumba é a
pancreatite aguda, possível em 20% dos casos. A pancreatite
apresenta dor e distensão do abdome, febre – tipicamente baixo
grau –, náuseas e vômitos. Um valor elevado de lipase sérica
favorece esse diagnóstico;
c) Geniturinário: em homens adultos, pode ocorrer
orquiepididimite em cerca de 20 a 30% dos casos. Geralmente
aparece durante a primeira semana de parotidite. Meninos pré-
púberes podem desenvolver orquite, mas é incomum em meninos
menores de 10 anos. A ooforite ocorre em cerca de 7% das
mulheres pós-púberes.
12. Notificações: a caxumba não é um agravo de notificação
compulsória, no entanto a ocorrência de surtos (mais que 2 casos em
grupo de 200 indivíduos) deve ser notificada.

6.1.1.2 Sialadenite por outros vírus

1. Agentes etiológicos: Coxsackievirus, que causa aumento


parotídeo e gengivite, echovírus e Epstein-Barr;
2. Fisiopatologia: infecção por via hematogênica ou via retrógrada
pelo ducto;
3. Quadro clínico: abaulamento unilateral da glândula, na maioria dos
casos, de rápida evolução, com leve dor local e febre. Os achados de
exame físico são semelhantes aos da caxumba;
4. Diagnóstico: também é clínico; ultrassonografia evidencia edema
difuso da glândula, com ectasia vascular. Os testes laboratoriais mais
úteis são hemograma – geralmente normal ou com linfocitose por
monocitose, elevação de amilase e proteína C reativa;
5. Tratamento: sintomáticos, compressas de calor local. O paciente
deve também ser afastado de suas atividades por 5 dias após o início
dos sintomas quando não há certeza do agente etiológico.

Figura 6.3 - Parotidite à esquerda: observar o edema da glândula


A diferenciação entre parotidites virais e a
caxumba baseia-se no teste sorológico.

6.1.2 Infecções bacterianas


6.1.2.1 Sialadenite supurativa aguda

Acomete principalmente as glândulas parótidas e submandibulares.


A infecção purulenta das glândulas salivares é mais comum em
situações de imunossupressão, como diabetes, em idosos ou
distúrbios eletrolíticos, especialmente em pós-operatórios de
cirurgias abdominais. A infecção do parênquima salivar dá-se,
geralmente, pela migração de bactérias provenientes da cavidade
oral por meio do ducto da glândula.
Clinicamente, há aumento agudo da glândula, pele avermelhada e
dor (Figura 6.4). O quadro pode ser acompanhado de febre, calafrios,
prostração, toxemia e leucocitose com neutrofilia. Ao exame físico,
pode-se notar à palpação bimanual e à expressão da glândula saída
de secreção purulenta pelo orifício do ducto, ou que indica
fortemente a presença de bactérias como agente etiológico.
Os organismos mais encontrados são o Staphylococcus aureus – de 50
a 90% – e o Streptococcus viridans. Os Gram negativos, como E. coli,
Klebsiella e Pseudomonas, são menos frequentes.
O tratamento constitui-se da administração empírica de antibióticos
com espectro para germes Gram positivos, principalmente
Staphylococcus aureus. Pode-se optar por cefalexina ou clindamicina
oral e, nos casos mais graves, por internação e administração de
oxacilina intravenosa.
Figura 6.4 - Indivíduo com parotidite à direita
Fonte: site Glândula Parótida.

6.1.3 Sialolitíase
A sialolitíase é uma condição em que uma massa calcificada, ou
sialolito, se forma dentro de uma glândula salivar; 80% dos cálculos
aparecem na glândula submandibular, 19% na parótida e 1% na
sublingual, e 75% dos pacientes estão entre a quinta e a oitava
década de vida; entretanto há casos entre crianças.
A causa da formação de cálculos é desconhecida, mas alguns fatores
podem estar associados, como estase salivar ou lesão e inflamação
do epitélio do ducto.
Na sialolitíase, 80% dos cálculos aparecem na
glândula submandibular, e são fatores de risco
desidratação, estenose de ducto, estase salivar
e processos inflamatórios crônicos do epitélio
do ducto.

O quadro clínico compõe-se de dor em cólica pós-prandial e


aumento da região da glândula. Ao exame físico, observam-se
edema de papila ductal, hiperemia e possível drenagem de secreção
purulenta – infecção secundária. À palpação bimanual, podem-se
notar ducto e ou cálculo no assoalho da boca, na glândula
submandibular.
O diagnóstico baseia-se em radiografia com contraste intraductal –
sialografia – e ultrassonografia nas quais podem-se ver os cálculos
associados à dilatação proximal do ducto (Figura 6.5), além do
esvaziamento retardado.
6.1.3.1 Tratamento

1. Sialadenite aguda:
a) Tratamento clínico: hidratação, antibióticos oral ou parenteral,
compressas quentes e massagem, sialogogos;
b) Tratamento cirúrgico: incisão e drenagem ou excisão da
glândula em casos refratários a antibióticos; considerar drenagem
em casos com formação de abscesso.
2. Cálculos salivares:
a) Tratamento clínico: hidratação, compressão e massagem,
antibióticos para a glândula infectada;
b) Tratamento cirúrgico: canulação do ducto com remoção de
pedra, excisão da glândula em caso recorrente.
Figura 6.5 - Tomografia computadorizada de pescoço evidenciando cálculo – seta azul –
em ducto de glândula submandibular esquerda – seta vermelha

Fonte: site Learning Radiology.

6.1.4 Neoplasias de glândulas salivares


As neoplasias das glândulas salivares representam 6% de todos os
tumores de cabeça e pescoço. As lesões malignas tipicamente
ocorrem após os 60 anos, enquanto as lesões benignas acima de 40
anos. As neoplasias benignas são mais frequentes nas mulheres do
que nos homens, enquanto os tumores malignos são distribuídos
igualmente entre os sexos. A maior parte dos casos ocorre na
parótida, seguida pela submandibular, salivares menores e
sublinguais, com predomínio das neoplasias benignas. A
sintomatologia é semelhante tanto para tumores benignos como
para malignos, sendo o principal achado uma massa indolor de
crescimento lento.
6.1.4.1 Diagnóstico

Utiliza-se principalmente a ultrassonografia, seguida de Punção


Aspirativa por Agulha Fina (PAAF).
Tanto a Tomografia Computadorizada (TC) quanto a Ressonância
Magnética (RM) têm boa definição para avaliar a massa tumoral e
devem ser feitas em qualquer lesão de aspecto benigno ou maligno.
6.1.4.2 Neoplasias benignas de glândulas salivares

Geralmente há massa indolor de crescimento lento. A queixa de dor é


rara e não sugere malignidade. São sinais e sintomas típicos:
1. Tumores parotídeos: massas indolores na periferia da glândula;
2. Tumores submandibulares e sublinguais: aumento indolor de
toda a glândula;
3. Tumores menores da cavidade oral: massas submucosas
indolores geralmente no palato e no assoalho da boca;
4. Tumores menores nos seios paranasais: sintomas de sinusite por
obstrução da drenagem desses seios; obstrução nasal ou sintomas
que simulam quadros de rinite.

a) Adenoma pleomórfico – tumor misto benigno

Trata-se de um tumor benigno de crescimento lento que


corresponde a 2 terços de todas as neoplasias de parótida e o mais
frequente em todas as demais glândulas salivares (Figura 6.6).
Macroscopicamente, tem cápsula fibrosa bem definida e,
histologicamente, pode-se observar cápsula incompleta (Figura
6.7). Em 90% dos casos é superficial ao nervo facial na parótida e,
em 10%, tende a ser profundo ao nervo ou acometer o espaço
parafaríngeo (Figura 6.8).
Figura 6.6 - Adenoma pleomórfico da glândula parótida esquerda

Fonte: Otolaryngology Houston.

Figura 6.7 - Histologia de adenoma pleomórfico


Nota: a seta indica a cápsula do tumor.
Fonte: Pathpedia.

Figura 6.8 - Adenoma pleomórfico à ultrassonografia


Fonte: Wikiwand.

Em até 7% dos casos, observa-se degeneração maligna para o


carcinoma ex-adenoma pleomórfico. Dessa forma, o tratamento do
adenoma pleomórfico consiste na ressecção total do tumor com
margem cirúrgica de glândula normal. O uso da radioterapia não está
bem estabelecido.
6.1.4.3 Tumor de Warthin

Segundo tumor benigno mais frequente na parótida – de 6 a 10% –,


também conhecido como cistoadenoma papilar linfomatoso. Trata-
se de um tumor composto por linfócitos com organização de centros
germinativos. Ocorre quase exclusivamente na parótida (Figura 6.9).
O tratamento é cirúrgico, com parotidectomia superficial, e a taxa de
recorrência é de 5%. O tumor de Warthin tende a ser bilateral em
10% dos casos e é geralmente encontrado nas glândulas maiores.
6.1.4.4 Hemangioma
É o tumor das glândulas salivares mais comum entre crianças, com
raras exceções. A conduta é expectante, porém os tumores grandes
ou os que apresentam complicações como sangramento ou
coagulação intravascular disseminada devem ser tratados com
corticosteroides intravenosos e, em casos não responsivos,
interferona. A ressecção cirúrgica raramente é necessária.
6.1.4.5 Neoplasias malignas de glândulas salivares

a) Quadro clínico

Independentemente da localização do tumor, os malignos tendem a


ter sintomatologia semelhante à dos benignos. Geralmente,
apresentam-se como massa de crescimento lento e indolor, mas
alguns sinais estão associados a alta chance de malignidade da lesão,
como o envolvimento do nervo facial, a observação de massa aderida
a planos profundos e a presença de metástases locais e a distância.
b) Estadiamento

Em 1997, o American Joint Committee on Cancer (AJCC) estabeleceu


um estadiamento para os tumores malignos das glândulas salivares.
1. Tumor primário (T):
a) Tx: tumor que não pode ser acessado;
b) T0: sem evidências de tumor primário;
c) T1: tumor < 2 cm sem extensão extraparenquimatosa;
d) T2: tumor entre 2 e 4 cm sem extensão extraparenquimatosa;
e) T3: tumor entre 4 e 6 cm ou com extensão
extraparenquimatosa sem envolvimento do nervo facial;
f) T4: tumor que invade a base do crânio, nervo facial e ou > 6
cm.
2. Envolvimento linfonodal (N):
a) Nx: linfonodos não avaliados;
b) N0: sem acometimento de linfonodos;
c) N1: metástases em apenas 1 linfonodo ipsilateral ao tumor com
até 3 cm;
d) N2: linfonodo de 3 a 6 cm;
e) N2a: apenas 1 linfonodo ipsilateral;
f) N2b: múltiplos linfonodos ipsilaterais;
g) N2c: linfonodos bilaterais ou contralaterais;
h) N3: linfonodos > 6 cm.
3. Metástases (M):
a) Mx: metástases não avaliadas;
b) M0: sem metástases;
c) M1: metástases a distância.

A partir desse estadiamento, os tumores podem ser classificados em


estádios:
1. Estádio 1: T1, T2, N0 e M0;
2. Estádio 2: T3, N0 e M0;
3. Estádio 3: T1, N1, T2, N1 e M0;
4. Estádio 4: T4, N0, T3, N1 e qualquer T com N2, N3 ou M1.

6.1.5 Carcinoma mucoepidermoide


O Carcinoma Mucoepidermoide (CM) é o tumor maligno mais
comum da parótida – 15% dos tumores de parótida – e é o segundo
mais frequente de submandibular e de glândulas salivares menores.
Dos CMs, 50% são na parótida, e o palato é o segundo local mais
frequente. Histologicamente, compõe-se por 2 tipos de células: as
mucosas e as epidermoides.
O tratamento é cirúrgico, com excisão ampla, margem cirúrgica e
preservação do nervo facial se não há envolvimento. A sobrevida em
5 anos está estimada em 80%.
Figura 6.9 - Carcinoma mucoepidermoide da glândula parótida direita indicado por seta
Fonte: Radiopaedia.

6.1.6 Carcinoma Adenoide Cístico (CAC)


Segundo tumor maligno mais comum de glândulas salivares,
corresponde de 5 a 10% das neoplasias das glândulas salivares
maiores e a 35% das neoplasias malignas das glândulas menores.
Origina-se das células ductais e mioepiteliais provenientes dos
ductos intercalares.
Tipicamente, o CAC é uma neoplasia de crescimento lento;
clinicamente, manifesta-se como a maioria dos tumores, além de
ser assintomático. A alta porcentagem de acometimento do nervo
facial é uma característica típica desse tumor, que evolui com
invasão perineural precoce. Metástase a distância é comum,
particularmente tardia, sendo o pulmão o sítio mais comumente
afetado.
O tratamento de escolha para o CAC é a excisão cirúrgica e a
radioterapia adjuvante é utilizada.
6.2 NEOPLASIAS DE CAVIDADE ORAL
E OROFARINGE
6.2.1 Quadro clínico
Os tumores mais comuns são o carcinoma de células escamosas e a
neoplasia de glândulas salivares menores – carcinoma cístico
adenoide. O quadro clínico não é característico. Em geral, a lesão
inicial apresenta evolução lenta e indolor. O paciente pode se queixar
de incômodo e globus faríngeo, bem como abaulamento ou
espessamento da mucosa jugal. Com a evolução da lesão, pode haver
queixa de odinofagia, disfagia, hemorragia, hemoptise e trismo. A
otalgia é um sintoma recorrente. Ao exame físico, observam-se
lesões leucoplásicas, vegetantes, que em geral são o estágio mais
comum de apresentação do quadro ou ulceradas. Pode haver infecção
secundária (Figura 6.10).
A lesão mais frequente é uma úlcera endurecida.
Figura 6.10 - Lesão tumoral, com áreas de ulceração e necrose na gengiva superior
Fonte: Sol Silverman, Jr., DDS, 2006.

Os principais fatores de risco são etilismo, tabagismo, idade acima


de 40 anos, imunodeficiências, deficiência nutricional e trauma
repetitivo de mucosa, em geral por oclusão dentária inadequada ou
próteses mal alocadas.
Deve-se suspeitar de lesões que não regridem em 2 ou 3 semanas,
devendo-se realizar biópsia. Os tumores epiteliais comumente se
iniciam como lesões leucoplásicas ou eritroplásicas. Deve ser
realizada biópsia, e caso não evidencie lesão maligna, devem ser
acompanhadas com bastante atenção.
Lesões leucoplásicas e eritroplásicas merecem
ser consideradas lesões pré-malignas.

6.2.2 Diagnóstico
É feito por meio de biópsia. Uma vez confirmado o diagnóstico,
deve-se realizar o estadiamento da lesão primária e avaliar a
possibilidade de metástases a distância.
6.2.2.1 Estadiamento
Segundo o manual da AJCC (1992), a classificação do carcinoma
espinocelular (CEC) de cavidade oral segue os critérios relacionados
a seguir.
1. Tumor primário (T):
a) Tx: in situ, diagnosticada;
b) T1: ≤ 2 cm;
c) T2: 2 a 4 cm;
d) T3: > 4 cm;
e) T4: invasão de estruturas adjacentes.
2. Nódulos linfonodais (N):
a) Nx: não diagnosticada;
b) N0: sem adenopatia;
c) N1: único ipsilateral de até 3 cm;
d) N2a: único ipsilateral de 3 a 6 cm;
e) N2b: nódulos múltiplos ipsilaterais de até 6 cm;
f) N2c: nódulos bilaterais ou contralaterais de até 6 cm;
g) N3: nódulos > 6 cm.
3. Metástases (M):
a) Mx: não diagnosticada;
b) M0: sem metástases;
c) M1: metástase a distância.

6.2.2.2 Exames complementares

Devem-se realizar TC e ou RM para avaliar a extensão da doença


para planos profundos e metástases a distância.
6.2.2.3 Metástases a distância

Seus sítios mais frequentes são pulmão, ossos e fígado.


6.2.2.4 Tratamento

O tratamento pode ser cirúrgico, radioterápico ou associado. Tanto a


radioterapia quanto a cirurgia têm sucesso no controle de lesões T1
na cavidade oral. O tratamento deve ser escolhido de acordo com
diversos fatores, como o sítio da lesão, as condições gerais do
paciente, suas condições socioeconômicas e a experiência da equipe
médica.
6.3 NEOPLASIAS DE LÁBIO
6.3.1 Definição
O CEC de lábio é o tumor maligno mais frequente na cavidade oral.
Ocorre, com maior frequência, em homens de fototipo baixo,
associado a tabagismo e antecedente de exposição prolongada à
radiação solar. Apresenta um dos maiores índices de cura, pois o
tumor é visível e seu diagnóstico em geral é precoce.
6.3.2 Quadro clínico
Lesão labial ulcerada que não cicatriza e forma crostas com eventual
sangramento. Linfonodos submentonianos e submandibulares
podem estar acometidos em 10% dos indivíduos.
A maioria dos tumores de lábio ocorre no
inferior; acredita-se que devido à maior
exposição solar.

6.3.3 Diagnóstico
É feito por biópsia local. Radiografia panorâmica de mandíbula pode
mostrar infiltração neoplásica.
6.3.4 Tratamento
O tratamento é eminentemente cirúrgico, com exérese da lesão,
removendo margens. Deve-se considerar manter a função de
vedamento labial, bem como da fala. Está indicada radioterapia após
cirurgia.
6.4 NEOPLASIAS DE NASOFARINGE
O carcinoma de nasofaringe (nasopharyngeal carcinoma – NPC)
apresenta um dos piores prognósticos entre os tumores malignos de
cabeça e pescoço. As razões para isso são a proximidade da base do
crânio e de outras estruturas vitais, a natureza invasiva do tumor, o
crescimento em espaço vazio – rinofaringe –, causando sintomas
em estágios avançados. O início se dá tipicamente em torno da tuba
auditiva – na fosseta de Rosenmüller – ou no rebordo da cóana,
desenvolvendo-se em sentido à própria tuba e em direção à base do
crânio. O crescimento para a orofaringe e as fossas nasais é raro.
Figura 6.11 - Lesão tumoral na nasofaringe vista em endoscopia (seta)

É mais comum em homens (2:1) e apresenta 2 picos de incidência:


20% ocorrem entre jovens com menos de 30 anos; e o restante em
indivíduos entre a quarta e a quinta décadas de vida, com diagnóstico
ao redor dos 50 anos.
6.4.1 Quadro clínico
O sintoma inicial mais comum é o aparecimento de massa cervical
assintomática localizada no ângulo da mandíbula ou inferiormente à
ponta da mastoide. Setenta por cento têm acometimento ganglionar
no momento do diagnóstico.
Hipoacusia condutiva em adultos deve ser investigada, pois os NPC
levam a disfunção tubária, otite média serosa e perda condutiva.
O acometimento dos nervos intracranianos resulta da extensão
superior ou posterior do tumor.
6.4.2 Fatores de risco
São semelhantes aos de câncer da orofaringe, como o consumo de
nitrosaminas, presentes em condimentos alimentares,
hidrocarbonetos policíclicos e condições precárias de vida. Há forte
associação com a infecção pelo vírus Epstein-Barr.
6.4.3 Diagnóstico
A TC – base do crânio e nasofaringe – é o exame de escolha para o
diagnóstico, o estadiamento e o acompanhamento terapêuticos.
Cerca de 25% desses tumores têm invasão da base do crânio (Figura
6.12).
A RM é melhor para avaliar o grau de extensão intracraniana, a
recorrência e as partes moles.
A biópsia fecha o diagnóstico. Devem ser realizadas biópsias das
lesões presentes e de fragmentos da fosseta de Rosenmüller.
Figura 6.12 - Neoplasia de rinofaringe com invasão do espaço parafaríngeo e extensão
lateral à direita
Fonte: Nasopharyngeal Carcinoma: The Role of the Epstein-Barr Virus, 2008.

6.4.3.1 Estadiamento

Há os estadiamentos feitos pelo AJCC e pela Union for International


Cancer Control (UICC), que são semelhantes. É utilizado o sistema de
classificação TNM.
1. Tumor primário (T):
a) Tx: sem definição;
b) Tis: tumor in situ;
c) T1: tumor confinado à nasofaringe;
d) T2: tumor que se estende à cavidade nasal e à orofaringe;
e) T2a: tumor sem comprometimento do espaço parafaríngeo;
f) T2b: tumor com comprometimento do espaço parafaríngeo;
g) T3: tumor que invade estruturas ósseas ou seios paranasais;
h) T4: tumor com extensão intracraniana e/ou envolvimento dos
pares cranianos, hipofaringe, fossa infratemporal e órbitas.
2. Acometimento linfonodal (N):
a) Nx: sem definição quanto a gânglios;
b) N0: ausência de gânglios palpáveis;
c) N1: adenopatia homolateral menor do que 6 cm acima da fossa
supraclavicular;
d) N2: adenopatia contralateral ou bilateral menor do que 6 cm
acima da fossa supraclavicular;
e) N3: adenopatia maior do que 6 cm ou na fossa supraclavicular.
3. Metástases a distância (M):
a) Mx: sem definição quanto às metástases;
b) M0: ausência de metástases;
c) M1: presença de metástases.

6.4.4 Tratamento
O tratamento primário consiste na radioterapia, com bons
resultados tardios, porém com índice relativamente alto de
complicações, como xerostomia, otite serosa e hiposmia.
Paciente do sexo masculino,
72 anos, trabalhador rural,
tabagista de 50 maços-ano,
etilista regular, com lesão
verrucosa hiperemiada em
borda lateral esquerda da
língua há 40 dias. Pode-se
iniciar tratamento empírico
com antibioticoterapia oral
e corticoterapia tópica?
Quadro de lesão verrucosa hiperemiada em borda lateral
esquerda, com características proliferativas, em paciente
com fortes fatores de risco, é indicativo de CEC de cavidade
oral. Apesar do diagnóstico diferencial de doença
infecciosa, devemos inicialmente proceder à biópsia da
lesão descrita antes de planejar o tratamento.

Você também pode gostar